pathology

Réussis tes devoirs et examens dès maintenant avec Quizwiz!

Which of the following is/are true of dilatation of the heart: 1) may be a consequence of myocarditis 2) right heart dilatation can be associated with left heart failure 3) typically occurs in Down`s syndrome 4) other options are NOT true

1) May be a consequence of myocarditis 2) Right heart dilatation can be associated with left heart failure -DS: mostly septal defects in these people

Which of the following is/are true of pulmonary embolism: 1) its source is a deep vein thrombosis 2) its complication is always pulmonary thrombosis 3) if untreated, it always leads to patient death 4) it occurs frequently in obstructive jaundice

1) its source is a deep vein thrombosis

Which of the following is/are primary skin tumors? 1) malignant melanoma 2) squamous cell carcinoma 3) carcinoid (neuroendocrine) tumors 4) basiloma

1) malignant melanoma 2) squamous cell carcinoma 4) basiloma On Jame´s document it says that only #3 should be correct.. While on pubmed it says: "Primary malignancies of the skin are uncommon in childhood. A low index of suspicion may lead to a delay in diagnosis. This paper reviews the most common pediatric primary skin tumors: melanoma, squamous cell carcinoma, and basal cell carcinoma."

Which of the following valves is the most frequently affected in rheumatic fever? 1) mitral 2) tricuspid 3) aortic 4) other options are NOT true

1) mitral

Cystic mass in the tail of the pancreas is found in a 50 years old chronic alcoholic with acites exam for epigastric intermittent pain. Which of the following is most likely to be found on a microscopic examination of the mass? 1) post necrotic pseudocyst 2) acute pancreatitis 3) insulinoma 4) adenocarcinoma of the pancreas

1) post necrotic pseudocyst

Which of the following is/are NOT the microscopic criteria of malignancy? 1) the shift in nucleo/cytoplasmic ratio towards cytoplasma 2) polymorphism of nuclei 3) Anizocytosis 4) Anisokaryosis

1) the shift in nucleo/cytoplasmic ratio towards cytoplasma Anisocytosis is a medical term meaning that a patient's red blood cells are of unequal size. anisokaryosis = Variation in the size of the nuclei of cells.

Which of the following is a complication of acute myocardial infarction 1. Cardiac pericardial 2. Rupture of papillary muscles 3. Fibrinous pericarditis 4. Development of acute aneurysm of the left ventricle

1. Cardiac pericardial 2. Rupture of papillary muscles 3. Fibrinous pericarditis 4. Development of acute aneurysm of the left ventricle

Excessive mucus production is described in 1. Catarrhal inflammation 2. Phlegmone 3. Mucinous carcinoma 4. Tuberculosis

1. Catarrhal inflammation 3. Mucinous carcinoma

Basillary meningitis is 1. Caused by mycobacterial infection 2. Caused by herpes virus infection 3. A disease with an acute clinical course 4. Exudative inflammation

1. Caused by mycobacterial infection 4. Exudative inflammation

Choose the disease with an incresed risk of developing a malignant tumor 1. Cirrhosis 2. Celiac spue 3. Ulcerative colitis 4. Hepatitis B infection

1. Cirrhosis 2. Celiac spue 3. Ulcerative colitis 4. Hepatitis B infection

Which of the following conditions are observed in terminal chronic right-sided heart failure? 1. Cyanotic induration of the spleen 2. Oedema of the lower extremities 3. Ascites 4. Pleural effusion, nutmeg liver

1. Cyanotic induration of the spleen 2. Oedema of the lower extremities 3. Ascites 4. Pleural effusion, nutmeg liver

Which the following is/are characteristic of multiple myeloma 1. Damage to kidney 2. Pathologic fractures 3. AA amyloidosis 4. The proliferation of neoplasric plasma cells

1. Damage to kidney 2. Pathologic fractures 4. The proliferation of neoplasric plasma cells

Which of the following is primary cutaneus neoplasm 1. Dermatofibroma 2. Malignant melanoma 3. Squamous cell carcinoma 4. Basilioma

1. Dermatofibroma 2. Malignant melanoma 3. Squamous cell carcinoma 4. Basilioma

Gangreen 1. Dry gangreen is found preferentially on lower limbs 2. Wet gangreen is described in diabetics 3. Gas gangreen is caused by yest infection 4. Abscess is an example for dry gangreen

1. Dry gangreen is found preferentially on lower limbs 2. Wet gangreen is descrided in diabetics

Clostridia are the causative agents of 1. Gas gangreen 2. Typhus abdominalis 3. Botulism 4. Tetanus

1. Gas gangreen 3. Botulism 4. Tetanus

Select the valid claims about granulation tissue: 1. it occurs in the base of peptic ulcers 2. it is formed by fibroblasts and capullaries in loose connective tissue 3. it is the final stage of wound healing 4. granulation tissue within the alveoli is the cause of grey hepatisation

1. it occurs in the base of peptic ulcers 2. it is formed by fibroblasts and capullaries in loose connective tissue

Hepatic metabolism impairment is typically associated with: 1) Gilbert syndrome 2) Dubbin Jonson syndrome 3) Down syndrome. 4) Rotor Syndrome.

1) Gilbert syndrome 2) Dubbin Jonson syndrome 4) Rotor Syndrome. Tonje´s quizlet said it was nr 3., but james´document had 1, 2 and 4 so I changed it!

Epitheloid cells in TB granulomas arise from? 1) Histiocytes and monocytes. 2) Epithelial cells and lymphocytes. 3) Endothelial cells and epithelial cells. 4) Lymphocytes and plasmocytes.

1) Histiocytes and monocytes.

Bowen`s disease is a term for: 1) In situ squamous cell carcinoma of the skin 2) in situ skin basiloma 3) in situ melanoma of the skin 4) in situ primary cutaneous lymphoma

1) In situ squamous cell carcinoma of the skin skin basiloma = a basal cell carcinoma.

Which of the following is/are sources of tuberculosis? 1) Infected cows milk. 2) Open pulmonary TB. 3) Transplacental transmission. 4) Droplet infection.

1) Infected cows milk. 2) Open pulmonary TB. 3) Transplacental transmission. 4) Droplet infection.

Acute myocardial infarction: a) Is always caused by the stenosis of the coronary artery with an atherosclerotic plaque b) Its course is typically worse in younger patients c) Morphologically is fully developed in 72 hours d) Coagulative necrosis is histologically described

b, d

Chronic inflammation can cause: a) AL amyloidosis b) Cachexia c) Pickwick syndrome d) Anaemia

b, d

Diphtheria: a) Is caused by gram-negative bacteria b) The myocardium is typically affected c) Spreads through contaminated food d) Causes fibrinous purulent infection

b, d

Hematoidin: a) This pigment is exogenous b) It has a similar structure as bilirubin c) Perk prussian blue reaction is positive d) Is almost always stored extracellularly

b, d

Inherited metabolic disorders of glycogen turnover in the cells (glycogenoses) a) Never cause organomegaly b) Might affect liver, myocardium, skeletal muscle or neurons c) Are never caused by deficiency of a lysosomal dehydrogenase d) Might manifest with a combination of hypoglycemia, hyperlactacidemia and hyperuricemia

b, d

Libman-Sacks endocarditis: a) Affects the aortic valve b) The inflammatory infiltrate contains neutrophils, lymphocytes and histiocytes c) Arises in rheumatoid arthritis d) Is associated with systemic lupus erythematosus

b, d

Ochronosis: a) Occurs in phenylketonuria b) Affects the joints c) Ochronic acid accumulation is described d) Homogentisic acid accumulation is present

b, d

Select the valid statement(s): a. Ixodes ricinus is the carrier of rabies b. The causative agent of rabies belongs among viruses c. The causative agent of anthrax belongs among viruses d. The causative agent of plague belongs among bacteria

b, d

Which of the statements about gout don`t apply a. It`s caused by increased levels of uric acid b. The afflicted suffers from repeated attacks of acute arthritis c. In chronic gout more joints are normally affected d. The characteristic lesion are gouty tophi e. Precipitated urate crystals breach the lysosomal membranes

C is wrong, it's usually just one joint.

Which of the following are examples of lipomatosis a. Aggregations of cholesterol in macrophages b. Aggregations of neutral fat in hepatocytes during alcoholic hepatitis c. Presence of adipocytes between cardiomyocytes of the right ventricle d. Digestion of peritoneal adipose tissue during acute necrosis of the pancreas e. Fat accumulation in 'granule cells' of encephalomalacia

C, presence of adipocytes between cardiomyocytes of right ventricle

Which of the following statements about atrophy are correct a. Its only detectable microscopically b. Its only detectable macroscopically c. Its usually evident both macro- and microscopically d. Atrophy can occur physiologically e. It cause a decrease in cell size

C,D,E It is visible macro + micro, it can be physiologically and it decreases cell size

What is the origin of amyloid deposits in medullary carcinoma of the thyroid a. AA amyloid b. Beta-amyloid c. Beta-2-microglobulin d. AL amyloid e. AE amyloid

C- cells causes increase of Calcitonin deposition according to pathoma p 13... what what type of amyloid??

Histocytes A. Their cytoplasm contains histamine B. They can be characterized as macrophages C. They belong to the infiltrate of acute inflamation D. Rank among antigen presenting cells

B. They can be characterized as macrophages D. Rank among antigen presenting cells

Which of the following are not true of theneuroendocrine tumor of the gastrointestinal tract A. May produce hormonally active substances B. Thy are always benign C. May cause episodes of hypoglicemia

B. Thy are always benign

Which of the following is caused by a meningococcal infection D. Severe acute sepsis

D. Severe acute sepsis

What is the best description of hyaline a. Eosinophilic amorphous material present intra- or extracellularly b. Eosinophilic amorphous material present only extracellularly c. Refractive material with Sudan black d. Granular material connected with necrotic tissue e. Light-colored cells as a result of increased content of water

a Eosinophilic amorphous material present intra- or extracellularly

Starvation is associated with a decrease in size of the a. Fat stores b. Heart c. CNS d. Liver e. Bones

a Fat stores b Heart d Liver

Which substances accumulates in organs in Gaucher disease a. Glucocerebrosides b. Galactocerebrosides c. Gangliosides d. Sphingomyelin e. Unknown lipopigment

a Glucocerebrosides (No (lady) GaGa, but (lady) GaGlu)

Which of the following cellular changes are reversible a. Glycogen depletion b. Karyorrhexis c. Autolysis d. Defect plasma membrane e. High density of mitochondrial matrix

a Glycogen depletion

During which of the following conditions can metastatic calcification arise a. Hyperparathyroidism b. Vitamin D deficiency c. Tissue necrosis d. Acute renal disease e. Calcified atheromatous emboli

a Hyperparathyroidism Metastatitc-high calicum leves

Which of the following statements about steatosis are correct a. It can lead to injury of the mitochondria, ER, and lysosomes b. HDL ensures transport of lipids to cells c. Steatosis of the liver usually (obvykle -> ''most frequently?'') originates from defect of lipoprotein synthesis d. Steatosis based on lysosomal dysfunction is always due to congenital defects e. Steatosis originates from accumulations of triglycerides and/or fatty acids

a It can lead to injury of the mitochondria, ER, and lysosomes e Steatosis of the liver usually (obvykle -> ''most frequently?'') originates from defect of lipoprotein synthesis

What is the most common cause of death in systemic AA amyloidosis a. Kidney failure b. Liver failure c. Intractable diarrhea with metabolic disorders d. Respiratory failure e. None of the answers are correct

a Kidney failure

All the following items are markers for necrosis except a. Lipofuscin b. Pyknosis c. Karyolysis d. Karyorrhexis e. Rupture of cell membranes

a Lipofuscin normal pigment, present in any old cell

In which of the following diseases is there a characteristic accumulation in macrophages a. Niemann-Pick disease b. Fabry disease c. Tay-Sachs disease d. Cereoidlipofuschinosis e. Pompe disease

a Niemann-Pick disease #he has "picked" macrophages#

Which of the mentioned changes is (not) present in the early phase of shock a) pulmonary collapse b) anuria c) pulmonary edema d) acidosis in the peripheral bloodstream e) caused by insufficient flow in the terminal venules

a Pulmonary collapse The question sheet has "not" in it.. and the answer sheet does not have "not" in the statement..

Neurogenic atrophy can occur in a. Skeletal muscle b. The brain c. The spinal cord d. Diencephalon e. Myocardium

a Skeletal muscle

Which of the following organs typically undergoes simple necrosis a. Skin b. Heart c. Liver d. Brain e. Kidneys

a Skin I think simple necrosis is a undergroup of coagulative, but I'm not sure!

To execute Virchow`s (macro)reaction we need a. Sulphuric acid b. Congo red c. Thioflavin T and T d. Lugol solution e. Hydrochloric acid

a Sulphyric acid d lugol solution

What is not affected in Niemann-Pick disease a. Endothelia b. Liver c. Lymph nodes d. Lungs e. None of the answers above is correct

a c (it accumulates in spleen, liver, lungs, bone marrow, and brain)

Choose the statements(s) valid for Weil´s disease: a) It prevalently damages the liver and the kidneys b) is caused by a virus c) its causative agent can be stained by silver impregnation d) capillary damage is described

a c d

Which of the following pair of organs is due to the dual blood supply least affected by infarction? a) Liver and lungs b) Liver and kidney c) Lungs and spleen d) Spleen and kidneys e) Heart and lungs

a liver and lungs

RS cells in Hodgkin lymphoma usually express: a) CD 30 b) CD 117 c) CD 15 d) CD 20 e) they cannot be detected immunohistochemically

a) CD 30 c) CD 15 ( d was corrected as right, but I think not. pathoma says no CD20 and wiki says Sternberg cells are CD30 and CD15 positive, usually negative for CD20 and CD45.)

Philadelphia chromosome is present in: a) CML b) Hodgkins disease c) Burkitts lymphoma d) Down`s disease e) Apoptoticky zanikajících monocytû

a) CML

For which variant of Hodgkin lymphoma is the characteristic of lacunar variant cell? a) Nodular sclerosis b) mixed cellularity c) lymphocyte depleted d) Lymphocyte rich e) eosinophilic granuloma

a) Nodular sclerosis

Pseudomembranous infiammation 1. occurs in post-antibiotic clostridial colitis 2. the mucosal epithelium remains intact 3.its subtypes include crupois and phlegmonous infiammation 4. its subtypes include diphtheric and ulcerative/necrotizing infiammation

1. occurs in post-antibiotic clostridial colitis 4. its subtypes include diphtheric and ulcerative/necrotizing infiammation

Most common type of Hodgkin lymphoma is: a) Nodular sclerosis b) Mixed cellularity c) Lymphocyte depleted d) Lymphocyte rich e) myelosarcoma

a) Nodular sclerosis

Unconjugated hyperbilirubinemia: a) Occurs in massive hemolysis b) The cause of neonatal jaundice c) Is accompanied by increased levels of bile acids in the blood d) Is a consequence of liver disease

a) Occurs in massive hemolysis b) The cause of neonatal jaundice d) Is a consequence of liver disease - Unconjugated hyperbilirubinemia: can result from increased production, impaired conjugation, or impaired hepatic uptake of bilirubin, a yellow bile pigment produced from hemoglobin during erythrocyte destruction.

Which statement is true of syphilis? a) Untreated usually involves three stages b) The first stage is characterized by the "ulcus molle" c) Gumma is a manifestation of the third stage d) When neurosyphilis occurs in the spinal cord, motor neurons are destroyed

a) Untreated usually involves three stages c) Gumma is a manifestation of the third stage - Primary: A red, oval sore, called a chancre (ulcus durum) develops at the site where the bacteria entered the body Elise: wiki says that syphilis should have a "hard chancre", whereas the statements speaks about a a "soft chancre" Molle = soft , Derfor er denne feil! -Secondary: condyloma latum, a cutaneous condition characterized by wart like lesions on the genitals -Latent: having serologic proof of infection without symptoms of disease. -Tertiary: formation of chronic gummas, which are soft, tumor-like balls of inflammation which may vary considerably in size. They typically affect the skin, bone, and liver, but can occur anywhere

Which of the following conditions does not have an increased tendency to thromboses? a) Use of Acylpyrin b) Atherosclerosis c) Increased blood viscosity d) Carcinoma of pancreas e) Intravascular catheter

a) Use of Acylpyrin

What is true about the role of cells in the immune process: a) macrophages serve mainly to the destruction of antigens b) interdigiting dendritic cells present antigen to T lymphocytes c) the lymphocytes remain in the node in which they arose d) NK cells are part of the acquired immune response

b) interdigiting dendritic cells present antigen to T lymphocytes

Prehepatic jaundice: 1. results from the lysis of red blood cells 2.occurs in fetal erythroblastosis 3. is a symptom of viral hepatites 4. is caused by elevated levels of conjugated bilirubin in the blood

1. results from the lysis of red blood cells 2.occurs in fetal erythroblastosis

Selecet the valid statement(s): 1.Crigler-Najjar syndrome is a congenital heperbilirubinemia 2. Gilbert's syndrome belongs among porphyrias 3. brown atrophy of the liver lead to hepatic failure 4. intracellular accumulation of lipofuscin is a feauture of hernochromatosis

1.Crigler-Najjar syndrome is a congenital heperbilirubinemia

Select the correctly paired statement(s): 1.hermosiderosis=hermosiderin accumulation in macrophages due to haemolysis 2.hermosiderin=identical composition as bilrubin 3.hermosiderin=predominantly intracellular accumulation 4.hermosidem=identical chemical composition as ceroid

1.hermosiderosis=hermosiderin accumulation in macrophages due to haemolysis 3.hermosiderin=predominantly intracellular accumulation

Which statement is/are true of syfilis 1.in neurosyphilis spinal dorsal roots are damaged 2. The first stage is characterized by ulcus durum 3. Aortic aneurysm develops early in the disease 4. Gummas are described in secondary syphilis

1.in neurosyphilis spinal dorsal roots are damaged 2. The first stage is characterized by ulcus durum

an ulcer heals: 1. ad integrum with regeneration of the affected tissue 2. with a scar 3. always with pseudocyst formation 4. it usually does not heal

2

choose the correct statement(s) about healing crupous pnemonia: 1. pleural adhesions never develop 2. lung parenchyma can undergo carnification 3. a late consequence can be the development of small cell carcinoma 4. granulation tissue is not observed during the healing process

2

choose the correct statement(s) about scrufolosis 1. it affects mostly mediastinal lymph nodes 2. fistula formation is described 3. the spread of the infection is hematogenous 4. an increased risk for the development of squamous cell carcinoma is present

2

choose the correct statement(s) for necrosis 1. caseous necrosis is always caused by ischemia 2. colliquative necrosis is described in acute pancreatitis 3. the type of necrosis depend only on the causative agent 4. necrosis is not accompanied by an inflammatory reaction

2

fat embolism: 1. its most frequent cause is retroperitoneal fatty tissue necrosis following acute pancreatitis 2. it accompanies fractures of long bones 3. it affects mostly the corporal circulation 4. it usually becomes symptomatic after 1-2 weeks

2

for the skin leasion of herpes simplex is typical: 1. the presence of Warthin-finkeldey inclusions 2. acantholysis in the stratum spinosum of the epidermis with blister formation 3. gram+ microorganism can be cultivated from the lesion 4. the inflammatory infiltrate in the dermis contain mostly granulocytes

2

granulomas are not a morphologic manifestation of: 1. tuberculosis 2. asbestosis 3. sarcoidosis 4. crohn disease

2

in lysosomal system 1. necessary alkaline ph is gradually increasing from early endosome to late endosome and lysosome 2. functioning acid hydrolases are highly glycosylated and transported from golgi apparatus to late endosome in transport vesicles 3. necessary alkaline ph is maintained by ATP dependent proton pump in lysosomal membranes 4. functioning acid hydrolases are highly glycosylated and transportet from cytosol to early endosome.

2

myocardial infarction 1. is typically caused by embolisation 2. the affected tissue heals with a fibrous scar 3. its typical late complication is the occurence of life-threatening arrythmias 4. myocardial damage develops initially subepicardially and can be followed by pericarditis epistenocardica

2

which staining method(s) can be used to highlight asbestos bodies? 1. alcian blue 2. perls prussian blue 3. PAS 4. trichromes

2

herpes zoster: 1. is caused by the measles virus 2. varicella is caused by the same virus 3. it is caused by the herpes simplex virus 4. the virus survives latently in the cortical motor neurones

2 (it survives in the dorsal root ganglia)

We describe microscopically in tuberculosis the following: 1) Langerhans cells 2) basophilic cellular debris in central caseous necrosis 3) acidoresistant (acid-fast) cocci 4) inclusions in Orth cells

2)

During its evolution hematoma can undergo: 1) metastatic calcification 2) abscess formation 3) hygroma formation 4) neoplastic transformation

2) 3)

Mallory hyaline: 1) is described in hyaline-degeneration of kidney 2) occurs in hepatocytes 3) is a manifestation of chronic alcoholism 4) it is an eosinophilic inclusion in neurons

2) 3)

Select the valid statement(s) about Hodgkin's lymphoma: 1) it typically forms osteolytic foci in the calva 2) it affects mainly young adults 3) is characterised by swelling of the lymph node with the presence of Reed-Sternberg cells 4) has for morphological types with different prognosis

2) 3) 4)

Tubular adenoma of the colon: 1) does not undergo malignant transformation 2) is an example of intestinal polyps 3) it may occur sporadically or in a familiar setting 4) is characterised by dysplastic changes

2) 3) 4)

Select the valid statement(s) about malignant melanoma: 1) it occurs exclusively in the skin 2) it may arise from congenital nevus 3) it can completely regress 4) prognostic features include the microscopic evaluation according to Clark

2) 3) 4))

Select the valid statement(s): 1) melanocytic nevus is a malignant skin tumor 2) the poorest prognosis among the variants of malignant melanoma is ascribed to the nodular type 3) the basal cell carcinoma is a benign tumor 4) malignant melanoma belongs to the neuroectodermal tumors

2) 4)

Steatosis is 1) the presence of lipids in fat cells 2) the accumulation of lipids in parenchymal cells 3) extracellular lipid accumulation 4) the presence of intracellular lipid vacuoles e.g. in liver or myocardium

2) 4)

Tay-Sachs disease: 1) is a prionopathy 2) is a lipidosis 3) is caused by a congenital defect of amyl-1,6-glucosidase 4) it affects the central nervous system

2) 4)

Morbus bowen is 1. Squamous cell carcinoma in situ of the cervix 2. A lesion induced mostly by UV radiation 3. Carcinoma in situ of the sweat glands 4. Pseulymphoma of the skin

2. A lesion induced mostly by UV radiation

Oose the correct statement 1. Apoptosis is a form of programmed cell death described only tumors 2. Apoptosis can be induced by cells of the immune system 3. Necrosis does not elicit an inflammatory response 4. The apoptotic bodies are phagocytized

2. Apoptosis can be induced by cells of the immune system 4. The apoptotic bodies are phagocytized

Which of the following mechanism is/are basic mechanism of steatosis? a) disorder of lipoprotein synthesis b) disorder of beta oxidation in mitochondria c) increased permeability of cellular membrane d) lysosomal dysfunction

a) disorder of lipoprotein synthesis b) disorder of beta oxidation in mitochondria d) lysosomal dysfunction

Which of the following tumors is the women's equivalent of testicular seminoma in men: a) dysgerminoma of ovary b) dermoid cyst of ovary c) choriocarcinoma d) yolk sac tumor e) malignant teratoma

a) dysgerminoma of ovary

Listeriosis: a) Is a bacterial infection b) Can lead to spontaneous abortion c) Can cause meningitis in weakened individuals d) Spreads through aerosol droplets

a, b, c

Choose the statement valid for lobar pneumonia 1. It is a viral infection 2. The inflammation is typically purulent 3. It is frequently associated with secondary pleural inflammation 4. Affects the entire lobe

2. The inflammation is typically purulent 3. It is frequently associated with secondary pleural inflammation 4. Affects the entire lobe

Which changes is/are characteristic of anthracosis 1. Significant interstitial lung fibrosis 2. The macrophages contain amorphous black pigment 3. Is described only in coal mine workers 4. Is followed by acute respiratory distress syndrome

2. The macrophages contain amorphous black pigment

Select the valid statement(s): a. Amyloid stains positive with Congo red b. Transthyretin (prealbumin) can form amyloid deposits c. Amyloid can be demonstrated macroscopically with the Virchow reaction d. Amyloidosis is among the complications of celiac sprue

a, b, c

Choose the statement(s) valid for Hashimoto thyroiditis: a) It clinically manifests with a struma b) Modified follicular (so-called Hurtle) cells are present c) Lymphoid follicles with germinal centre formation are described in the thyroid parenchyma d) The inflammatory infiltrate consists mostly of lymphocytes and plasma cells

a, b, c, d

Disorders of substrate degradation in lysosomal system might be caused by a) Deficiencies of several lysosomal enzymes b) Deficiencies of lysosomal proteins functioning as activators of some lysosomal enzymes c) Defects in a transport of lysosomal hydrolases from Golgi apparatus to late endosomes d) Defects of lysosomal membrane proteins controlling functions of lysosomal system

a, b, c, d

Urate stones can occur in: a) Dehydration b) Urinary tract infections c) Hyperparathyroidism d) Vitamin D poisoning

a, b, c, d

Actinomycosis: a) Causes an inflammation with abscess formation b) The inflammation is fibroproductive c) Is a precancerous lesion d) The causative agent can be stained in Grocott

a, b, d

Among staphylococcal infections we include: a) Hidradenitis b) Paronychia c) Erysipelas d) Puerperal mastitis

a, b, d

Cyanosis: a) Easily occurs in anemia b) Occurs in congenital heart defects with right-to-left shunt c) Is a light pink color of mucous and cutaneous surfaces d) Can also occur in healthy humans

a, b, d

HPV infection can be typically present in the following diseases: a) Squamous papilloma of the tongue b) Koilocytosis of the squamous epithelium of the exocervix c) Endometroid adenocarcinoma d) Squamous carcinoma of the endocervix

a, b, d

Mycobacterium avium: a) Causes granulomatous inflammation b) Belongs among acid-fast (acidoresistant) bacteria c) Ziehl-Neelsen staining can be used for its detection d) Only people with concurrent M. tuberculosis infection are at risk for infection

a, b,c

Choose the statement(s) valid for corpora amylacea: a) They are PAS positive b) They are eosinophilic c) They occur in the prostate d) They occur in the lungs

a, b,c, d

Among pseudotumours we include: a) Pseudoepiteliomatous hyperplasia of the epidermis b) Ovarian cystadenoma c) Retention cyst of sebaceous glands d) Xanthoma

a, c

Apoptosis: a) Activation of the Bax-Bak channel is described b) In malignant tumours apoptosis is intensified c) The FaS protein plays a major role during the initiation of apoptosis via the intrinsic pathway d) Is the basic mechanism of simple atrophy

a, c

Beta 2 microglobulin: a) May be elevated in cancer b) It participates in the development of Alzheimer's disease c) Can cause amyloidosis d) Is increased in chronic respiratory insufficiency

a, c

Choose the neuroectodermal tumour(s) a) Pheochromocytoma b) Chondrosarcoma c) Neurinoma d) Chondroma

a, c

Choose the statement(s) true for uterine leiomyoma: a) Its growth is influenced by hormones b) Has no effect on fertility c) We may encounter bizarre cells d) It may undergo metastatic calcification

a, c

Choose the statement(s) valid for ependymoma: a) It occurs most often in IV. ventricle of the brain b) It is a malignant tumour c) It may cause hydrocephalus d) There is histologically rosette formation around the blood vessels

a, c

Select the valid statement 1. Amyloid stains positive with red trichhrome 2. Transthyretin (prealbumin) can form amyloid deposits 3. Amyloid can be demonstrated macroscopically with the Virchow reaction 4. Amyloid does not form deposits in the heart

2. Transthyretin (prealbumin) can form amyloid deposits 3. Amyloid can be demonstrated macroscopically with the Virchow reaction

Choose the cell belonging into the inflammatory infiltrate of leptosy 1. Touton multinuclear giant cells 2. Virchow cells 3. Reed-sternberg cells 4. Mikulicz cells

2. Virchow cells

SChiller-duval bodies are typically present in 1. Embryonal carcinoma 2. Yolk sac tumor 3. Choriocarcinoma 4. Semicoma

2. Yolk sac tumor

Select the valid statement(s): 1. nevus is characterized by hyperpigmentation without increase in the number of melanocytes 2. an increase in nmber of metanocytes under the influence of sunlight is called a solar lentigo 3. malignant metanoma is an example for melanodermia 4. leukoderma is an example for a pigments nevus

2. an increase in nmber of metanocytes under the influence of sunlight is called a solar lentigo

Mucin production and exudation with large numbers of neutrophils occur typically in: 1. crupous infiammation 2. catarrhal purulent infiammation 3. infection by pyogenic bacteria 4. viral infections

2. catarrhal purulent infiammation 3. infection by pyogenic bacteria

Granulomatous infiammations do NOT includes: 1. rheumatoid arthritis 2. infctious mononucleosis 3. sarcoidosis 4. Crohn's disease

2. infctious mononucleosis

Selecet the valid statement(s): 1. cholesterolosis always affects hepatocytes 2. steatosis typically affects hepatcocyte 3. cholesterolosis is an example for microvesicular steatosis 4. the basis for hepatic steatosis is theaccumulation of foamy

2. steatosis typically affects hepatcocyte 3. cholesterolosis is an example for microvesicular steatosis

Choose the statement(s) valid for typhus abdominalis (typhoid fever): a) Sepsis is typically present b) The infection remains latent for a long time in the stomach c) The infection remains latent for a long time in the gallbladder d) It most commonly affects the jejunum

a, c

Pneumoconioses 1. they belong to granulomatous infiammation 2. the lead to a reduction of respiratory volume 3. silicosis of the lungs is a example 4. they are typically occupational diseases

2. the lead to a reduction of respiratory volume 3. silicosis of the lungs is a example 4. they are typically occupational diseases

Select the valid statement(s) about pigments: 1.they are soluble substances which lead to staining of cells by diffusion 2. thei rigin may be exogenous or endogenous 3, cause the coloration cells in unstained samples 4. are not detectable marcoscopically

2. thei rigin may be exogenous or endogenous 3, cause the coloration cells in unstained samples

Calcifications are positively stained with: 1.luxol bluer 2.von kossa stain 3.Peris Prussia blue reaction 4.nissl staining

2.von kossa stain

Select the correctly paired statement(s) 1. fresh blood in stool= myeloma (or it says melona?) 2. digested blood in the stool =enterorrhagia 3. bleeding into the lung parenchyma= apoplexy 4. spotty hemorraghe in skin, mucosa or in serosa= petachia

3,4

Select the valid statement(s): 1. medulloblastoma is a tumor arising in the bone marrow of young children 2. glioblastomas are among the rarest braintumors of adults 3. pilocytic astrocytoma occurs primarly in childhood 4. meningioma is usually a benign tumour

3,4

choose the correct combinations 1. neutrophils-histamine 2. extravasal erythrocytes-hematin 3. neutrophils-purulent inflammation 4. eosinophils-allergic reaction

3,4

choose the stament(s) NOT belonging into the triad of Horner: 1. miosis 2. ptosis of the eyelids 3. damage to the innervation of the oculomotoric nerve 4. exophtalmus

3,4

crohn disease: 1. is frequently complicated by toxic megacolon 2. is a granulomatous infection with suppurative granuloma formation 3. is associated with fistula formation 4. may affect the stomach

3,4

mark the complication(s) of tuberculosis 1. aneurysm of the ascending aorta 2. AL amyloidosis 3. meningitis 4. AA amyloidosis

3,4

which staining method(s) can be used to highlight reticulin fibers? 1. blue trichrome 2. sudan IV 3. silver impregnation 4. PAS

3,4

choose the incorrect statements: 1. mural thrombus could be seen by naked eye 2. red thrombus is caused by stagnation of blood 3. mural thrombi are fomed in capillaries 4. in the heart thrombi arise only in atrium

3,4 (red thrombus is a venous thrombi... red because it tends to contain more enmeshed red cells robbins p 88)

Epithelioid cells in tuberculous granulomas arise from 1. Degenerated pneumocytes 2. Mycobacteria-infected specialized endothelial cells 3. Activated macrophages 4. Multinucleated cells of Langerhans type

3. Activated macrophages

Which of the following is/are NOT example of acquired immunosuppression 1. Glucocorticoid therapy 2. Advenced chronic myeloid leukemia 3. Agammaglobulinemia of Bruton 4. HIV infectiob

3. Agammaglobulinemia of Bruton

Icterus 3. Describe a brown discoloration of the skin and organs

3. Describe a brown discoloration of the skin and organs WRONG

Which of the following are true of sarcoma 3. Is a histological subtype of rhabdomyosarcoma

3. Is a histological subtype of rhabdomyosarcoma

Metastatic calcification 1. Can be caused by adenoma of the thyroid gland 2. Is seen in fibrinous scars 3. Is dependant on the levels of blood calcium 4. It affects the kidneys

3. Is dependant on the levels of blood calcium 4. It affects the kidneys

Senile amyloidosis 1. Is caused by light chain deposits 2. Causes pancreatic insufficiency 3. Is typical descibed in the myocardium 4. Is made of deposits of proteinaceous material

3. Is typical descibed in the myocardium 4. Is made of deposits of proteinaceous material

Mallory hyaline 1. Is described in hyaline degeneration of kidney 2. Occurs in HPV infected kearatinocytes 3. It can be manifested of chronic alcoholism 4. It is an eosinophilic inclusion in hepatocytes

3. It can be manifested of chronic alcoholism 4. It is an eosinophilic inclusion in hepatocytes

Which statement is/are true of classic Hodgkin's lymphoma 1. It can be associated with HPV infection 2. It is a T-cell lymphoma 3. It is typically described in youg adults 4. Langhans cells are diagnostic for it

3. It is typically described in youg adults

Melanin 1. Is synthetized by melanophages 2. Defends the cell against the effects of infrared radiation 3. Its production is indirectly regulated by adrenal hormones 4. Albinism is described only in animal not in humans

3. Its production is indirectly regulated by adrenal hormones

Herpes simplex virus is usually related to 1. Merkel cell carcinoma 2.gastric MALT lymphoma 3. None of the mentioned neoplasias 4. Carcinoma of the uterine cervix

3. None of the mentioned neoplasias

Russel bodies 1. They are made of phagocytized material in macrophages 2. They are positive in special stains for iron 3. Occur in the cytoplasm of plasma cells 4. Contain immunoglobulins

3. Occur in the cytoplasm of plasma cells 4. Contain immunoglobulins

Subarachinoidal bleeding is most frequentky caused by 3. Rupture of the medial meningeal artery

3. Rupture of the medial meningeal artery - false

The inflammatory pseudomembrane in the course of pseudomembranous inflammation 3. The inflammatory infiltrate is composed mainly of macrophages

3. The inflammatory infiltrate is composed mainly of macrophages WRONG

Which of the following is characterized of gout 1. Calcification in the CNS 2. Presence of Criggler-Najjar-type giant cells 3. Tophus formsation 4. Hydroxyapathite crystal deposition in the articular synovium

3. Tophus formsation

In which of the following polyps of the gastrointestinal tract do we describe malignant change the most frequently 1. Tubular adenoma 2. Hyperplastic polyp 3. Villous adenoma 4. Carcinoid polyp

3. Villous adenoma

Posthepatic (mechanical) jaundice typically occurs in: 1. viral hepatites 2. occurs in fetal weythiroblastosis 3. carcinoma of the pancreatic head 4. heaptocellular carcinoma

3. carcinoma of the pancreatic head 4. heaptocellular carcinoma

Serious infiammation is characterized by: 1.dense granuloctuc infiltrate 2. accumlation of transudate 3. congetion of serous membranes 4. production of fluid with watery appearance

3. congetion of serous membranes 4. production of fluid with watery appearance

In autoimmune vasculitis which type(s)f necrosis is(are) tipically vessel wall? 1.cascous 2.liquefactive 3. fibrinoid 4. numerical

3. fibrinoid

Asbestosis: 1. gives rise to hepatocellular carcinoma 2. fiborus asbest is stored in tissues in the so-called Askenazy 3. is associated with fibroproduction 4. is usually localized in the lungs

3. is associated with fibroproduction 4. is usually localized in the lungs

Select the valid statement(s) about vitilgo: 1. belongs among the so-called metanodermias 2. it is the same as a freckle 3. it is probably an autoimmune disorder 4. it is characterized by limited blurred hyperpigmentation

3. it is probably an autoimmune disorder

SElect the valid statemenst(S) about dystrophic calcifcation: 1. it may be a manifestatio of parathyroid adenoma 2. ia a sympton of calcitronin overprodcution 3. occurs e.g. in the Ghon complex 4. it can be distinguished from metastatic calcifaciont with the Warthin-Starry stain

3. occurs e.g. in the Ghon complex 4. it can be distinguished from metastatic calcifaciont with the Warthin-Starry stain

Fungalpneumonia 3.In malignancies they oftenleat to death of the patient

3.In malignancies they oftenleat to death of the patient

a malignant epithelial tumor is with prevailing cellular components over the stroma is called: 1. mucinous carcinoma 2. scirrhous carcinoma 3. diffuse carcinoma 4. medullary carcinoma

4

a pyogenic membrane is typically described in: 1. phlegmon of the lower limb 2. bacterial meningitis 3. tuberculosis 4. chronic abcesses

4

choose the correct statement(s) about balzer necrosis 1. the necrotic areas undergo saponification due to precipitaion of plasmatic iron 2. the unleashed pancreatic enzymes digest the duodenal wall 3. it accompanies chronic pancreatitis 4. it affects the peripancreatic adipose tissue

4

choose the correct statement(s) about the cells of neuroendocrine tumors: 1. nuclear atypias are prominent even by low magnification 2. they have optically clear nuclei 3. mucin production is typically present 4. secretory granules are found in their cytoplasm

4

choose the statements characterizing urothelial carcinoma: 1. koilocytosis is present in the superficial cells 2. the neoplastic cells are always spindle shaped 3. we typically describe psammomatous bodies 4. the tumor has usually a papillary growth pattern

4

choose the valid statement(s) about precanceroses: 1. they evolve in all cases into invasive lesions 2. uterine leiomyomas belong into precancerous lesions 3. an example is adenomyomatous hyperplasia of the prostate 4. erythroplasia of Queyrat of the penis is a precancerous lesion

4

choose the valid statement(s) about prostatic adenocarcinoma: 1. it is characterized by osteochondrotic bone metastases 2. is accompanied by an elevation of serum AFP levels 3. is caused by HPV infection of the prostate 4. it originates from the peripheral part of the prostate

4

cyanotic induration of the organs: 1. is caused by left sided heart failure 2. is usually caused by chronic anemia 3. histologically it is a combination of venostasis and an increase in elastic fibers 4. histologically it is a combination of venostasis and and increase in reticulin fibers

4

glioblastoma is: 1. is a well differentiated glioma 2. the most frequent brain tumor in children 3. is frequently followed by extracranial metastases 4. is a badly differentiated glial neoplasm

4

select the valid statement(s) 1. adenomatous prostatic hyperplasia is a precancerous lesion 2. there are NO urothelial papillomas 3. dysgerminoma is a benign tumor 4. leiomyosarcoma is a malignant tumor arising from smooth muscle

4

select the valid statement(s) about hepatic steatosis: 1. it macroscopically leads to green coloration of the liver 2. it is the deposition of fat in adipocytes located in the portal field 3. it is the result of impaired beta oxidation of amino acids 4. common causes include hypoxia or toxic effects

4

the gleason score is: 1. a score used in the staging of prostatic adenocarcinoma 2. a score used in the grading of renal adenocarcinoma 3. it represents the depth of cervical squamous adenocarcinoma invasion 4. essential for establishing the prognosis of patients with prostatic adenocarcinoma

4

Choose the statements NOT valid for necrosis: a) Is the basic mechanism of numerical atrophy b) One of its first manifestations is pyknosis of the nuclei of affected cells c) Typically affects isolated cells in the necrotic tissue d) In brain tissue necrotic foci gradually develop into a pseudocyst

a, c

Select the feature(s) of infectious mononucleosis: a. Amplification of immunoblasts in reticulo-endothelial system b. Cytomegalovirus inclusions in the neurons of the cerebral cortex c. Hepatosplenomegaly d. Purulent tonsilitis

a, c

Select the valid statement(s) about measles: a. It is associated with the risk of viral encephalitis b. Is characterized by exanthema and diarrhea c. Warthin-Finkeldey giant cells are microscopically described d. It frequently occurs in unvaccinated individuals

a, c

The so-called Reiter syndrome consists of: a) Urethritis b) Polyarthritis c) Conjunctivitis d) Gastritis

a, c

T-lymphoblastic leukemia often affects the following organ(s): a) CNS b) Mediastinum c) Testes d) None of the above mentioned

a, c (can anyone find a source on this.. because in Patoma he only mentiones that it presents as a mediastinal(thymic) mass called lymphoma)

Among the diseases with a higher incidence of cancer we include: a) Lynch syndrome b) Crouzon syndrome c) Morbus von Recklinghausen d) Li Fraumeni syndrome

a, c, d

Choose the statement(s) valid for Weil's disease: a) It prevalently damages the liver and the kidneys b) Is caused by a virus c) Its causative agent can be stained by silver impregnation d) Capillary damage is described

a, c, d

Choose the statement(s) valid for lobar pneumonia: a) It is a bacterial infection b) The inflammation is granulomatous c) Is frequently associated with fibrinous pleural inflammation d) Affects the entire lobe

a, c, d

Choose the statement(s) valid for seminoma: a) The cytoplasm of tumour cells is PAS positive b) It occurs only in the gonads c) Ovarian dysgerminoma has same microscopic structure d) It may be part of mixed germ cell tumours

a, c, d

Fungal pneumonia: a) Arises more frequently in immunocompromised patients b) It has mostly a harmless course c) In haematological malignancies they often lead to the death of the patient d) The fungal mycelia are PAS-positive

a, c, d

. Choose the statement(s) NOT true for mucoviscidosis: a) It is an autosomal dominant disease b) The affected males are often infertile c) A high risk of sudden death is described in children d) The endocrine portion of the pancreas is mostly affected

a, c,d

Choose the correct statement(s) for the primary stage of syphilis: a) Ulcus durum develops at the point of entry of infection b) Ulcus molle develops at the point of entry of infection c) Plasma cells predominate in the inflammatory infiltrate d) In women it usually arises in the vulva

a, c....Ulcus durum is the same as hard chancre... regarding d i only found this: In women, chancres can develop on the outer genitals or on the inner part of the vagina. A chancre may go unnoticed if it occurs inside the vagina or at the opening to the uterus (cervix). The sores are usually painless and are not easily seen.

Choose the valid statement(s) for amyloidosis: a) It may be inherited b) It is never found in the colon c) It is never located in the small intestine d) It may accompany the diabetic foot

a, d

Cold abscess: a) Typically occurs in the vertebrae b) Is of non-infectious origin c) Arises in rheumatoid arthritis d) Is associated with a systemic mycobacterial infection

a, d

Gonococcal infections cause: a. Purulent inflammation b. Nonpurulent inflammation c. Chancroid of the genital d. In some cases purulent tonsilitis

a, d

Peroxisomal biogenesis a) Is dependent on a regulated transport of peroxisomal structural proteins and enzymes synthetized on cytosolic ribosomes to peroxisomes b) Is independent on nuclear DNA (gDNA) because peroxisomes are semiautonomous organelles c) Is restricted to specialized cell types, e.g. hepatocytes, neurons, oligodendroglia and cortical adrenal cells d) Is dependent on cytosolic proteins (peroxines) recognizing signal amino acid sequences in proteins destined for peroxisomal membranes and matrix

a, d

Pulmonary embolism: a) May manifest as sudden death b) Causes acute left ventricular failure c) When the foramen ovale is patent, the thrombotic embolus usually travels from the large to the small circulation d) Cancer can increase its incidence

a, d

Symptoms characteristic for serum sickness, a type III hypersensitivity reaction are: a) Urticaria b) Rhinitis, bronchial constriction and cough c) Haemolysis and anaemia d) Fever, joint pain and a skin rash

a, d

Select the valid statement(s) about amyloid: a. It is amorphous proteinaceous matter b. It can be deposited in the kidney during chronic inflammation in the organism c. It is deposited intracellularly d. It is a fatty substance

a,b

Select the valid statement(s) about cystic fibrosis: a. It affects exocrine glands b. It leads to malabsorption c. Its complications include insulinopenic diabetes mellitus d. The inheritance is gonosomal recessive

a,b,c

Which of the following are not characteristics for multiple myeloma a. AA Amyloidosis B. Skeleton involvment

a. AA Amyloidosis

Which of the following process is an example of dysplasia? a. Actinic keratosis b. Chronic cystitis c. Chronic bronchitis d. Ulcerative colitis e. Pigmented nevus

a. Actinic keratosis Also called "solar keratosis" and "senile keratosis". It is a pre-cancerous patch of thick, scaly, or crusty skin. These growths are more common in fair-skinned people and those who are frequently in the sun. Dysplasia=the enlargement of an organ or tissue by the proliferation of cells of an abnormal type, as a developmental disorder or an early stage in the development of cancer.

Which of the following infectious agents can cause myectomas in the lung a. Aspergillus fumigatus b. Candida albicans c. Mycoplasma pneumonia d. Histplasma capsulatum e. Cryptosporidium parvum

a. Aspergillus fumigatus

Marfan syndrome is a. Autosomal-dominant defect of the glycoprotein fibrillin b. Genetic determined defect of collagen type III c. Defect of maturation of collagen type I d. Hyalinosis of the wall of coronary vessels e. One of the glycogenoses

a. Autosomal-dominant defect of the glycoprotein fibrillin

Deficiency of which vitamin manifests itself as heart failure: a. B1/Thiamine b. B12 c. Folic acid d. Niacin e. Vitamin E

a. B1/Thiamine In mammals, deficiency results in Korsakoff's syndrome, optic neuropathy, and a disease called beriberi that affects the peripheral nervous system (polyneuritis) and/or the cardiovascular system

Which of the following diseases are not caused by exotoxins a. Brucellosis b. Staphylococcal scalded skin syndrome c. Pertussis d. Scarlet fever e. Diphtheria

a. Brucellosis

The tumor characterized by reciprocal chromosomal translocations leading to amplification of c-myc oncogene is called: a. Burkitt's lymphoma b. Neuroblastoma c. Chronic myeloid leukemia d. Fibrosarcoma e. Small cell carcinoma

a. Burkitt's lymphoma A cancer of the lymphatic system, particularly B lymphocytes found in the germinal center. All types of Burkitt's lymphoma are characterized by disregulation of the c-myc gene by one of three chromosomal translocations

Tumor promoters acts by: a. Changes of cellular differentiation b. DNA mutation c. Activation of apoptosis d. Dephosphorylation of Rb protein e. Formation of oxygen radicals

a. Changes of cellular differentiation

What is the most common cause of deficiency of thiamine (Vitamin B1) in the developed world: a. Chronic alcoholism b. Malabsorption c. Gastroenteritis d. Parenteral nutrition e. Chronic dialysis

a. Chronic alcoholism Thiamine deficiency commonly presents subacutely and can lead to metabolic coma and death. A lack of thiamine can be caused by malnutrition, a diet high in thiaminase-rich foods (raw freshwater fish, raw shellfish, ferns) and/or foods high in anti-thiamine factors (tea, coffee, betel nuts) and by grossly impaired nutritional status associated with chronic diseases, such as alcoholism, gastrointestinal diseases, HIV-AIDS, and persistent vomiting

Which of the following diseases don't have a viral etiology a. Condylomata lata b. Condylomata accuminata c. Molluscum contagiosum d. Common cold (Rhinitis) e. Rabies

a. Condylomata lata Condylomata lata is a cutaneous condition characterized by wart like lesions on the genitals. They are generally symptoms of the secondary phase of syphilis, caused by the spirochete, Treponema pallidum

Which of the following agents cause Q fever a. Coxiella burnetti b. Rickettsia rickettsii c. Chlamydia psittaci d. Chlamydia trachomatis e. Rickettsia prowazekii

a. Coxiella burnetti

Lesch-Nyhan syndrome belongs to a. Defects of nucleoprotein metabolism b. Defects of protein metabolism c. Defects of polysaccharide metabolism d. Defects of beta-oxidation of fatty acids e. Defects of lipid metabolism

a. Defects of nucleoprotein metabolism

Niacin deficit manifests like a. Dementia b. Hemorrhagic diathesis c. Diarrhea d. Dermatitis e. Night blindness

a. Dementia c. Diarrhea d. Dermatitis

Which of the following sentences don`t relate to hydropic cell swelling a. Increased number of cytoplasmic organelles b. Dilated cisterns of ER c. Injury to the cell volume regulatory mechanisms d. Influx of sodium ion to the cell e. Efflux of potassium out of the cell

a. Increased number of cytoplasmic organelles

Which of the following is the LEAST characteristic feature of a malignant tumor? a. Increased proliferation of cells b. Nuclear atypia c. Distant metastasis d. Penetrating or invasive to adjacent tissue e. Invasion to blood or lymphatic vessels

a. Increased proliferation of cells

Which virus can cause necrotizing bronchitis and diffuse hemorrhagic necrotizing pneumonia in adults sometimes in the form of epidemics a. Influenza virus A b. CMV c. Rhinovirus d. Norwalk-like virus e. Rotavirus

a. Influenza virus A

Which of the following statements applies to Amyloidosis a. It can occur in chronic inflammatory conditions b. It occurs in medullary carcinoma of the thyroid gland c. It acts like a basophilic substances which stains with Congo red d. It weakens the wall of blood vessels e. It cause renal failure

a. It can occur in chronic inflammatory conditions b. It occurs in medullary carcinoma of the thyroid glande. e .It causes renal failure (its eonsinophilic?)

Which of the following is NOT true about neuroblastoma? a. It's a tumor for middle-aged adults b. May arise in the medulla c. May arise in the posterior mediastinum d. Can secret catecholamines e. Histologically rosette formation is present

a. It's a tumor for middle-aged adults

What is the most common finding of patients with trypanosomiasis during autopsy a. Meningoencephalitis b. Massive hepatic necrosis c. Lung abscesses d. Cardiac abscesses e. Endocarditis

a. Meningoencephalitis

Which of the following types of cells have a connection with infectious processes: a. Mikulicz cells b. Warthin-Finkeldey cells c. Virchow cells d. Orthovy cells e. Kultschitský cells

a. Mikulicz cells (klebsiella rhinoscleroderma) b. Warthin-Finkeldey cells c. Virchow cells (leprosy) d. Orthovy cells A = (Rhino)Scleroma, is a chronic granulomatous bacterial disease of the nose that can sometimes infect the upper respiratory tract B = A type of giant multinucleate cell found in hyperplastic lymph nodes early in the course of measles and also in HIV-infected individuals C = The cavities in osseous tissue containing bone cells?? D = Orth cells = macrophages containing mycobacteria (NOTE: This is a specific term used ONLY in meningitis. Kultschitský cells - can´t find anything online.....

For determining the prognosis of the tumor we use: a. Mitotic index b. Necrosis c. Degree of differentiation d. Spreading to the lymph nodes e. Number of deposits of dystrophic calcifications

a. Mitotic index c. Degree of differentiation d. Spreading to the lymph nodes

Which type of cell serve as a reservoir for HIV: a. Monocytes b. Fibroblasts c. T-lymphocytes d. Epithelial cells e. B-lymphocytes

a. Monocytes

Hunter and Hurler`s disease belongs to a. Mucopolysaccharidoses b. Mucoviscidoses c. Gangliosidoses d. Glycogenoses e. A and B are correct

a. Mucopolysaccharidoses (lysosomal storage diseases) Hurler syndrome: mucopolysaccharidosis type I Hunter syndrome: mucopolysaccharidosis type II

Which of the following are examples of a ganglion a. Myxoid modulation of connective tissue b. Transparent modulation of connective tissue c. Fibrinoid modulation of connective tissue d. Hyaline modulation of connective tissue e. Proliferative changes

a. Myxoid modulation of connective tissue

Which of the following pathogens have fimbriae as a virulence factor, allowing them to adhere to the mucosal surface of the host organism a. Neisseria gonorrhea b. Vibrio cholera c. Campylobacter jejuni d. Yersinia enterocolitica e. Bacillus anthracis

a. Neisseria gonorrhea

To DNA oncogenic viruses belong: a. Papillomavirus b. Myxovirus c. Papovavirus (they probably mean the polyomavirus since the papillomavirus is a papovavirus) d. Rhabdovirus e. Picornavirus

a. Papillomavirus

Which of the following protein encoding genes are inactivated by HPV? a. Rb b. Ras c. Myc d. Bcl-2 e. WT-1

a. Rb

Which of the following infectious agents may induce formation of bladder cancer? a. Schistosoma hematobium b. Trichinella spiralis c. Trichophyton d. Candida albicans e. Ascaris lumbricoides

a. Schistosoma hematobium - a flatworm that develope squamous cell carcinoma of the bladder

Cholera cause a. Serous inflammation b. Fibrinous inflammation c. Phlegmon d. Suppurative inflammation e. Catarrhal inflammation

a. Serous inflammation

To Neuroendocrine tumors belongs: a. Small cell carcinoma of lung b. Squamous cell carcinoma of cervix c. Pancreatic insulinoma d. Appendix carcinoid e. Neuroma

a. Small cell carcinoma of lung c. Pancreatic insulinoma d. Appendix carcinoid

Which of the following condition predisposes to the emergence of squamous cell carcinoma? a. Solar keratosis b. Bowen's disease/dermatosis c. Chronic eczema d. Chronic ulcer e. Keratoacanthoma

a. Solar keratosis b. Bowen's disease/dermatosis d. Chronic ulcer

Which of the following infectious agents often exhibit acute suppurative inflammation, and abscess production a. Staphylococcus aureus b. Mycobacterium tuberculosis c. Blastomyces dermatitidis d. Coccidioides immitis e. Cryptococus neoformans

a. Staphylococcus aureus

Which of the following infectious agents cause scarlet fever a. Streptococcus pyogenes b. Burkholderia mallei c. Clostridium botulinum d. Clostridium tetani e. Klebsiella pneumonia

a. Streptococcus pyogenes Scarlet fever (also called scarlatina in older literature) is an infectious disease which most commonly affects children. Signs and symptoms include sore throat, fever, and a characteristic red rash. Scarlet fever is usually spread by inhalation. There is no vaccine, but the disease is effectively treated with antibiotics. Most of the clinical features are caused by erythrogenic toxin, a substance produced by the bacterium Streptococcus pyogenes (group A streptococcus) when it is infected by a certain bacteriophage

Which of the following conditions belong among the potentially lethal complications of meningococcal infections: a. Waterhouse-Friedrichsen syndrome b. Myocarditis c. Hepatitis d. Glomerulonephritis e. Endometritis

a. Waterhouse-Friedrichsen syndrome Also called hemorrhagic adrenalitis or Fulminant meningococcemia: it is defined as adrenal gland failure due to bleeding into the adrenal glands, caused by severe bacterial (or rarely viral) infection (most commonly the meningococcus Neisseria meningitidis

Tumor associated with congenital deletion on chromosome 11 is? a. Wilm's tumor b. Cervical carcinoma c. Mammary gland carcinoma d. Colorectal carcinoma e. Yolk sac tumor

a. Wilm's tumor (or nephroblastoma) is cancer of the kidneys that typically occurs in children, rarely in adults

goblet cells a. they are stained positively with the PAS method b. they occur in interstitial metaplasia in the stomach c. characteristically occur in endocrine tumors d. are synonymous to Kultschitsky cells

a. they are stained positively with the PAS method b. they occur in interstitial metaplasia in the stomach

Hemosiderosis a) is not a disease b) pigment is stored in the reticuloendothelial cells c) is a disease caused by increased absorption of Fe ions d) is followed by the so-called bronze diabethes

ab

Which of the followings is/are true of pathological immune response type 2 reactions? a) occurs in myasthenia gravis b) it cal lead to urticaria c) this type of reaction can occur in mismatched transfusions d) typical examples are immuno-complex diseases (eg. glomerulonephritis)

ab (In type II hypersensitivity (or cytotoxic hypersensitivity)[1] the antibodies produced by the immune response bind to antigens on the patient's own cell surfaces. The antigens recognized in this way may either be intrinsic ("self" antigen, innately part of the patient's cells) or extrinsic (adsorbed onto the cells during exposure to some foreign antigen, possibly as part of infection with a pathogen). These cells are recognized by macrophages or dendritic cells, which act as antigen-presenting cells. This causes a B cell response, wherein antibodies are produced against the foreign antigen.)

Choose the statements(s) valid for molluscum contagiosum: a) it is a viral infection b) the skin is affected c) it is more frequent in children d) it is a precencerosis

abc (molluscum contagio´sum a common, benign, usually self-limited viral disease of the skin marked by the formation of firm, rounded, translucent, crateriform papules containing caseous matter and intracytoplasmic inclusions (molluscum bodies), which contain replicating virions. The disease is spread by contact and is common in young children. In adults, lesions in the pubic area indicate sexual transmission.)

Choose the statements(s) valid for Hashimoto thyroiditis: a) it clinically manifests with a struma b) modified follicular (so-called Hurtle) cells are present c) lymphoid follicles with germinal centre formation are described in the thyroid parenchyma d) the inflammatory infiltrate consist mostly of lymphocytes and plasma cells

abcd

choose the statement(s) valid for corpora amylacea a) they are PAS positive b) thaéy are eosinophilic c) they are in the prostate d) they occur in the lungs

abcd

Choose the statements(s) NOT true for mucoviscidosis: a) it is an autosomal dominant disease b) the affected males are often infertile c) a high risk of sudden death described in chilren d) the endocrine portion og the pancreas is mostly affected

acd

Which of the following is/are true of radicular cysts? a) bone cyst mostly involving the radius b) is caused by epithelization of chronic abscess in a dead tooth c) is a cyst resulting from the inflammation of meninges of spinal roots d) is usually associated with periodontitis

b) is caused by epithelization of chronic abscess in a dead tooth

Which of the following represents a viral skin infection? a) molluscum contagiosum b) pemphigus vulgaris c) verruca vulgaris d) herpes zoster

b) pemphigus vulgaris It must be which of the following does not represent a viral skin infection, as all the other 3 are viral skin infections

Choose the statement(s) valid for sarcomas: a) They are tumours arising from the ectoderm b) Occur more often in a younger age c) All sarcomas have rapid progression and a fatal outcome d) Typical for them are implantation metastases

b

During the degradation of haemoglobin the globin chains are: a) Changed by macrophages into biliverdin b) Split by macrophages into aminoacids c) Are responsible for the colour of the stool d) Changed by macrophages into bilirubin

b

Lipofuscin : a) Is found in the adipocytes of patients with diabetes mellitus b) Is the cause of the brownish discoloration of organs in atrophia fusca c) It accumulates on the bottom of peptic ulcers d) Its accumulation in cardiomyocytes leads to the so-called senile chronic heart failure

b

Papanikolau stain: a) Is an obsolete staining method b) Is used routinely in cytology c) Is an excellent stain for the examination of the cervix conisation specimen for suspected dysplastic changes d) Using this technique you need longer times for formalin fixation of the examined material

b

Select the infection NOT caused by Chlamydia a. Psittacosis b. Ulcus molle c. Lymphogranuloma venereum d. Trachoma

b

Which of the following statement(s) is NOT true for the inflammation of serous membranes: a) No chronic inflammation is described on serous membranes b) There is often abscess formation c) Viral infections can have a serous character d) Fibrinous pericarditis occurs in uraemia

b

select the valid statements a) nevus is characterized by hyperpigmentation without increase in the number of melanocytes b) an increase in the nr of melanocytes under the influence of sunlight is called solar lentigo c) malignant melanoma is an example for melanodermia d) leukoderma is an example for a pigmented nevus

b

Select the valid statement(s) 1. an example of granulomatous infiammation is diphtheria 2 the so-called Celsus signs of infiammation dscribe infiammation microscopically 3.in the case of vasculitis we usually do NOT see fibrinoid necrosis of the affected blood vesel walls 4. fibrous adhesions develop from fibrinous pericarditis

4. fibrous adhesions develop from fibrinous pericarditis

Russel bodies 1. they are composed of viral inclusions 2. they are PAS positive 3. occur in the cytoplasm of mast cells 4. occur in the granular endoplasmic reticulum of plasma cells

4. occur in the granular endoplasmic reticulum of plasma cells

Selecet the valid statement(s) about hematin: 1. it has the same chemical composition as bilirubin it is the basis of the yellow colour of older haemorrhages 2. it is a modure of hermosiderin and ceroid 3. it may be demostrated by reaction to trivalent ... 4. occurs in peptic ulcers

4. occurs in peptic ulcers

Select the valid statement(s): 1.caseous necrosis is caused by ischemia 2.liquefactive necrosis occurs on in CNS 3.the type of necrosis depends only on the causative agent 4.renal infarction is an example of coagualative necrosis

4.renal infarction is an example of coagualative necrosis

Which of the following is/are not true of pancreatic adenocarcinoma? a) often affects the cauda/head of pancreas b) the prognosis is very poor c) jaundice is frequent d) tumors cells produce mucous

?

Zenker degeneration/Wax degeneration is a variant of ... necroses occurring in .... During ... which is correct? :) a. Coagulation - Skeletal muscle - Infectious processes (For example typhoid fever) b. Caseous - Skeletal muscle - Infectious processes (For example typhoid fever) c. Coagulation - Brain - Acute pancreatitis d. Liquefactive - Brain - Pancreatitis e. Fibrinoid - Heart - Anaphylactic shock

A Zenker degeneration is a variant of coagulative necrosis ocurring in the skeletal muscle during infectious processes /typhoid fever

How does apoptosis differ from necrosis a. Morphologically necrotic cells b. Nuclei of apoptotic cells exhibit karyorrhectic disintegration c. The causes of apoptosis are mainly external factors d. Apoptotic cells show pyknotic nuclei e. Apoptosis occurs only in the prenatal period

A & D Morphologically necrotic cells Apoptotic cells show pyknotic nuclei I though that necrotic cells also had pyknotic nuclei?? hmmm

Granulomatous infiammation develops: A. Around surgical sutures B. In the case of sarcoidosis C. In the ghon's complex D. In ulcerative colitis

A b c

Amyloidosis arising from chronic inflammatory conditions are responsible for the increase in serum levels of which of the following proteins a. Immunoglobulins b. SAA protein (Serum amyloid A) c. Transthyretin d. Beta-2-microglobulin e. Calcitonin

b SAA protein (Serum amyloid A)

After stainin with congo red: With which of the following characteristics of proteins are the presence of birefringence connected a. The ability to bind to acid b. Tertiary structure of beta-pleated sheets c. Electrophoretic mobility d. Content of hydroxyproline in the protein e. Molecular weight of the protein

b Tertiary structure of beta-pleated sheets

For which of the following purposes are immunohistochemistry inappropriate a. To identify histogenetic causes of neoplasms b. To identify enzyme activity c. Identification of tissue-specific markers of neoplasms d. Detection of hormones and their receptors e. Detection of auto-antibodies

b To identify enzyme activity

For which of the following diseases are caseous necrosis typical a. Acute myocardial infarct b. TBC c. Acute pancreatitis d. Cerebral infarct e. Pulmonary pneumoconiosis

b Tuberculosis

Choose the statements(s) valid for polyarteritis nodusa: a) is a granulomatous infection b) is a necrotizing infection c) the kidneys are typically affected d) monocytes are predominant in the inflammatory infiltrate

b and c

Gelatinous atrophy is a. A type of numerical atrophy b. A type of atrophy of adipose tissue c. Normal and commonly occurs as part of the aging process d. Characteristic for the bone marrow

b atrophy of adipose tissue

In a patient that died from sepsis during bacterial endocarditis, it was during autopsy of the parenchyma of the left kidney and spleen found yellow wedge-shaped foci of inflammation under the capsule. It most likely represents a. Liquefactive necrosis b. Coagulation necrosis c. Caseous necrosis d. Hemorrhagic necrosis e. Fibrinoid necrosis

b coagulative necrosis

Which of the following gene products stimulates apoptosis a. C-erb-B2/HER-2/neu b. P53 c. Myc d. Ras e. Bcl-2

b p53 Myc - is a regulator gene that codes for a transcription factor. The protein encoded by this gene is a multifunctional, nuclear phosphoprotein that plays a role in cell cycle progression, apoptosis and cellular transformation. Ras - involved in transmitting signals within cells (cellular signal transduction) Bcl-2 - regulate cell death (apoptosis), by either inducing (pro-apoptotic) it or inhibiting it (anti-apoptotic). Bcl-2 is specifically considered as an important anti-apoptotic protein and is thus classified as an oncogene. C-erb-B2/HER-2/neu - protoncogene, tyrosine kinase

Which of the following tumor is most likely the primary source of metastasis to the liver? a) Cervical carcinoma in situ b) Adenocarcinoma in colon c) Clear cell carcinoma of the right kidney d) Testicular germ cell tumor e) Pleomorphic adenoma of the parotid gland

b) Adenocarcinoma in colon

Does not belong among the rhabdomyosarcomas: a) Sarcoma botryoides b) Alveolar sarcoma c) Juvenile rhabdomyosarcoma d) Wilms tumor e) Rhabdomyoma

b) Alveolar sarcoma d) Wilms tumor e) Rhabdomyoma

Necrosis liquefactive: a) occurs only in the CNS b) arises from the action of proteolytic enzymes c) may be the result of purulent inflammation d) never occurs in TB

b) Arises from the action of proteolytic enzymes c) May be the result of purulent inflammation d) Never occurs in TB -purulent: full of, containing, forming, or discharging pus; suppurating -Liquefactive necrosis: transformation of the tissue into a liquid viscous mass. Often associated with focal bacterial or fungal infections. The affected cell is completely digested by hydrolytic enzymes, resulting in a soft, circumscribed lesion consisting of pus and the fluid remains of necrotic tissue. -Can also occur in lungs and pancreas

Which of the following substances is not a proven carcinogen? a) Asbestos b) Aspartame c) Vinyl chloride d) Benzpyrene e) Aflatoxin

b) Aspartame

Which of the following does not represents viral skin infection? a) molluscum contagiosum b) pemphigus vulgaris c) verruca vulgaris d) herpes zoster

b) pemphigus vulgaris Pemphigus vulgaris is a chronic blistering skin disease with skin lesions that are rarely pruritic, but which are often painful. It is classified as a type II hypersensitivity reaction, with the formation of anti-desmosome antibodies formed.

Among the primary proliferative inflammation we include A. Palmar fascitis B. Penile fibromatosis C. Dupuytren's disease D. Dermatofibroma

A. Palmar fascitis B. Penile fibromatosis C. Dupuytren's disease

The complex of reumathic fever does not include A. Raynaudphenomenon B. Myocarditis

A. Raynaudphenomenon

Granulomas are not a morphological manifestation of A. Silicosis

A. Silicosis

It is typical for polyclonal antibodies that they: a) recognize only one antigen epitope b) recognize more antigen epitopes c) never show cross-reactivity d) can be produced by so-called hybridomas e) are used to monitor the entire antigen

b) recognize more antigen epitopes c) never show cross-reactivity

Right ventricular hypertrophy: a) persists as long standing concentric hypertrophy b) results from hypertension in the pulmonary circulation c) the result is a brown induration of lung d) may occur in left heart failure

b) results from hypertension in the pulmonary circulation d) may occur in left heart failure

Diarrhea in children is most commonly caused by: a) Salmonella b) rotavirus c) Entamoeba histolytica d) S. enterotoxicosis

b) rotavirus

The most serous complication of myocardial infarction: a) aneurysm b) rupture c) pulmonary edema d) pericardial adhesions

b) rupture

The most important risk factors for urothelial bladder cancer include: a) HPV infection b) smoking cigarettes c) exposure to ... d) Schistosoma infection

b) smoking cigarettes d) Schistosoma infection Urothelial carcinoma is a prototypical example of a malignancy arising from environmental carcinogenic influences. By far the important cause is cigarette smoking, which contributes to approximately half of the disease burden. Schistosomiasisis a disease caused by parasitic worms of the Schistosoma type. It may infect the urinary tract or intestines. Signs and symptoms may include abdominal pain, diarrhea, bloody stool, or blood in the urine. In those who have been infected a long time, liver damage, kidney failure, infertility, or bladder cancer may occur. The disease is spread by contact with water that contains the parasites. These parasites are released from freshwater snails that have been infected

For abnormal findings on cytological examination (PAP) cervical biopsy was performed in 28 years old female. The endocervical squamous epithelium was regular, both on the surface and in glands. Diagnosis? a) squamous dysplasia b) squamous metaplasia c) endocervical atrophy d) squamous hyperplasia

b) squamous metaplasia Endocervix is usually made from glandular epithelium, not squamous

Acute coronary syndromes do not include: a) myocardial infarction b) stable angina pectoris c) sudden death d) chronic ischemic heart disease

b) stable angina pectoris d) chronic ischemic heart disease

Which of the following tumors consists of epithelioid and spindle components? a) rhabdomyosarcoma b) synovial sarcoma c) malignant melanoma d) liposarcoma

b) synovial sarcoma a - mesenchymal c- neuroectoderm d - no epithelial component

Which of the following statements about fat embolism is/are true? a) the most common cause is bruising of subcut. tissue b) tends to accompany multiple fractures of long bones c) apply mainly in the pulmonary bloodstream and CNS d) syndromes appear within one week of trauma

b) tends to accompany multiple fractures of long bones c) apply mainly in the pulmonary bloodstream and CNS A fat embolism is a type of embolism that is often caused by physical trauma such as fracture of long bones, soft tissue trauma, and burns.

hypovolemic shock: a) the assumption is always extensive bleeding b) the cause may be an extensive bleeding c) the cause is hypovolemia in patients with edema d) the cause may be severe dehydration in patients with cholera

b) the cause may be an extensive bleeding d) the cause may be severe dehydration in patients

Which of the following finding is most characteristic for tile carcinoma: a) neurosecretory granule b) the corner beads c) intracellular mucus d) complex glandular formation e) tumor rosettes

b) the corner beads

Choose the statement(s) valid for the sentinel lymph node: a) It is always only one lymph node b) It is examined in the case of breast fibroadenoma c) It is examined in advanced melanoma d) Its assessment is conducted in adenocarcinomas of the rectum

c

For well-differentiated liposarcoma applies: a) Rapid disease progression leads to the death of the patient b) Usually manifests as cachexia c) Often develops in the retroperitoneum d) Arises in children in the brown adipose tissue

c

In acute occlusion of left main coronary artery through embolism: a) Myocardial infarction develops b) Manifests with recurrent retrosternal pain c) Sudden death occurs d) Necrosis occurs in the posterior wall of the left ventricle

c

Inflammatory pseudoxanthoma: a) Is a benign tumour composed of cells with watery clear cytoplasm, with an inflammatory background b) Is a malignant tumour composed of cells with watery clear cytoplasm, with an inflammatory background c) Is a non-neoplastic lesion d) Leads to atrophy in the surrounding tissue

c

Select the valid statement(s) about atrophy: a. It's always a localized process b. It's histological hallmark is the intracellular lipid deposition c. Physiological atrophy is the same as involution d. In senile atrophy is deposition of senile amyloid in the brain is described

c

The TNM classification: a) Is based only on histology b) Is determined for all cancers c) Relates to the prognosis of the given tumour d) Is based mainly on clinical findings

c

The most serious complication of myocardial infarction is: a) Development of acute aneurysm b) Development of chronic aneurysm c) Rupture of the ventricular wall d) Pulmonary oedema

c

Which organ(s) is affected in Tay-Sachs disease? a. Endothelia b. Liver c. Brain d. Spleen e. Kidneys

c

among the staphylococcal infection do NOT include: a. Suppurative bronchopneumonia b. Carbuncle c. Erysipelas d. Suppurative osteomyelitis

c

Mycosis fungoides a) Is a fungal skin disease b) Phlegmonous type of inflammation is described, with formation of Pautrier abscesses in the basal layer of the epidermis c) This disease has a chronic course d) it is a lung infection of immunosuppressed patients

c (b is wrong because this condition causes localized skin rashes, plaques and nodules, no phlegmonous inflammation) Mycosis fungoides is the most common type of Cutaneous T-cell lymphoma.

Systemic hypertension leads to: a. Hypertrophy of the right ventricle b. Brown induration of the lung c. Chronic dilatation of the aortic bulb d. The progression of atherosclerosis

c ,d

Eosinophilic substance present in the extracellular space, made of fibrillary material structured like beta-pleated sheets from beta-2-microglobulin is a. Basal lamina b. Lipofuscin c. Amyloid d. Collagen e. Keloid

c Amyloid

What is the origin of amyloid deposits in senile amyloidosis of the heart a. AA amyloid b. Beta-amyloid c. Amyloid derived from transthyretin d. AL amyloid e. AE amyloid

c Amyloid derived from transthyretin

Which type of amyloid deposits inside neurons in familial amyloid polyneuropathy a. AA amyloid b. Beta-2-microglobulin c. Amyloid derived from transthyretin d. AL amyloid e. AE amyloid

c Amyloid derived from transthyretin

Does not increase the risk for thrombosis: a) Dehydration b) Thrombocytosis c) Anemia d) Diabetes e) Venostasis

c Anemia

During autopsy it`s found marked reduction in the size of the kidney with numerous 1-3 mm large scars on the surface. What will be the most likely histological morphology with this finding a. Edema and neutrophilic granulocytosis b. Edema and fibrin c. Atrophy and hyaline d. Lymphocytes and fibrin e. Macrophages and edema

c Atrophy and hyaline

Which type of amyloid deposits in patients on hemodialysis a. AA amyloid b. Beta-amyloid c. Beta-2-microglobulin d. AL amyloid e. AE amyloid

c Beta-2-microglobulin

Which of the following don`t belong to simple atrophy a. So-called brown atrophy b. Senile atrophy of the skin c. Chronic atrophic cholecystitis d. Muscular atrophy during denervation e. Muscular atrophy during inactivity

c Chronic atrophic cholecystitis

Transparent connective tissue is characteristic for a. Ganglion b. Myxedema c. Cornea d. Corpus albicans e. Stroma of the renal papillary

c Cornea

What applies to glycogenosis type 1 a. Block of conversion of glycogen to maltose b. Defect of glycolysis in the muscle c. Defect of release of glucose from glycogen in the liver d. Accumulation of proteoglycans e. None of the answers are correct

c Defect of release of glucose from glycogen in the liver

What is the most common cause of steatosis of the liver a. Anemia b. Chloroform c. Ethanol d. Hypercholesterolemia e. Pre-eclampsia (EPH gestóza)

c Ethanol - meaning alcohol

So-called hyaline membrane in the lungs during ARDS are caused by aggregates of a. Aymloid b. Collagen c. Fibrin d. Material of the basal membrane e. Intermediate filaments

c Fibrin

Which of the following diseases are connected with defects in the degradation of cerebrosides a. Tay-Sachs disease b. Niemann-Pick disease c. Gaucher disease d. Wolman disease e. Tangier disease

c Gaucher disease

Which of the following lysosomal storage diseases are caused by deficiency of beta-glucocerebrosidase a. Pompe disease b. Tay-Sachs disease c. Gaucher disease d. Niemann-Pick disease e. Fucosidosis

c Gaucher disease

Which of the following are examples of simple atrophy a. Attenuation of the bone marrow b. Aplastic anemia c. Gelatinous atrophy d. Lipomatous atrophy e. Atrophy due to irradiation

c Gelatinous atrophy

Congenital metabolic disease of saccharide metabolism that is characterized by abnormal increase in concentration of hepatic glycogen of normal structure and failure of increase of serum glucose after peroral intake of fructose. Based on the information given determine which enzyme is affected in this disease a. Fructokinase b. Glucokinase c. Glucose-6-phosphatase d. Phosphoglucomutase e. UDPG-glycogen transglucosylase

c Glucose-6-phosphatase Hydrolyses Glucose-6P --> glucose (neede for both the breakdown of glycogen and the transformation of fructose into glucose)

Lipid peroxidation in cells exposed to ionizing radiation is mediated by a. Catalase b. Superoxide anions c. Hydroxyl radicals d. Hydrogen peroxide e. Glutathione

c Hydroxyl radicals

From which of the following conditions can metastatic calcification arise a. Necrosis of adipose tissue b. Bone fracture c. Hypercalcemia d. Fibrinoid necrosis e. Chronic abscess

c Hypercalcemia

Which of the following manifestations are the most reliable indicators of cell death a. Eosinophilic cytoplasm b. Granular cytoplasm c. Karyolysis d. Cell swelling e. Lipofuscin aggregates

c Karyolysis Remember: cell swelling is reverisble, lipofuscin aggregates is normal in older cells,

Hepatosplenomegaly is discovered in a 3-year-old body with mental retardation. Analysis of the tissue specimen obtained by biopsy shows unusually large amounts of glcuocerebrosides. This disease reflect a deficiency in which of the following enzymes a. Hormone-sensitive lipase b. Lipoprotein-lipase c. Lysosomal hydrolases d. Sphingolipid synthase e. Tissue phospholipases

c Lysosomal hydrolase

Fat necrosis is associated with all of the following conditions except a. Trauma of the mammary glands b. Acute pancreatitis c. Peroxidation of triglyceride esters d. Hydrolysis of triglycerides to glycerol e. Inflammatory reaction

c Peroxidation of TAGesters

A 60-year-old man is transported to the hospital with acute liver failure. He undergoes successful orthotopic liver transplantation, but the donated liver in the first 3 days produces only a little amount of bile. Dysfunction of the graft in the are ascribed to reperfusion injury. Which of the following substances are the most probable cause of reperfusion injury of the graft a. Cationic proteins b. Lysosomal acid hydrolases c. Reactive oxygen species d. Free ionic iron e. Hydrochloric acid

c ROS

Which will be the most likely finding on m. gastrocnemius of a 35 year-old lady after removal of the cast from the leg after 6 weeks of immobilization a. Conversion of the muscle to fast muscle fibers b. Reduction in the number of muscle fibers c. Reduction in the number of myofibrils in the muscle fiber d. Increase of mitochondria in muscle fibers e. Increased numbers of satellite cells

c Reduction in the number of myofibrils in the muscle fiber

What is the lethal lysosomal disease termed which manifests itself in infancy with mental retardation and blindness a. Phenylketonuria b. Spina bifida c. Tay-Sachs disease d. Osteogenesis imperfecta e. Duchenne muscular dystrophy

c Tay-Sachs disease

In which of the following diseases does hepatosplenomegaly not occur a. Leukemia b. Lysosomal enzymopathy c. Tay-Sachs disease d. Gierke disease e. Gaucher disease

c Tay-Sachs disease (disease of white matter of brain) Tay Sachs- Hexosaminidase (sex - sex)

Which of the following changes can during ischemic injury be considered manifestations of irreversible change a. Increase/Aggregate of glycogen b. Aggregation of nuclear chromatin c. Intracellular release of lysosomal enzymes d. Decreased protein synthesis e. Depletion of intracellular calcium

c The intracellular release of lysosomal enzymes

Pseudoabscess a) Is filled with serofibrinous exudate b) Is confined by a pyogenic membrane c) An example is the oviduct filled with pus d) Can occur in the brain

c d

Select the valid statement(s) about dystrophic calcification: a. It may be a manifestation of parathyroid adenoma b. Is a symptom of calcitonin overproduction c. Occurs eg. in the Ghon complex d. It can be distinguished from metastatic calcification with Warthin-Starry stain

c d

Which of the following conditions predispose to metastatic calcification a. Necrosis of adipose tissue b. Bone fracture c. Hypercalcemia d. Fibrinoid necrosis e. TBC

c hypercalcemia

Pathological immune response II: a) is mediated by IgE antibodies b) antigens involved in this reaction are only endogenous c) a typical example of this disease is hemolytic anemia d) the type of immune response is always dependent on the presence of complement

c) A typical example of this disease is hemolytic anemia -Pathological immune response II and hypersensitivity II is the same I think. At least that makes sense since hemolytic anemia is an example of it. -IgE = I -Complement = III

What is true of T and B lymphocytes: a) plasma cells arise exclusively by transformation of T-lymphocytes b) so-called natural killer cells are transformed macrophages c) CD4 lymphocytes are mainly producers of lymphokines d) CD8 positive lymphocytes are activated B cells

c) CD4 lymphocytes are mainly producers of lymphokines

Which of the following mediators activates the retinoblastoma gene product (Rb protein): a) Plasminogen activator b) Colagenase IV c) Cyclin-dependent kinase d) p53 e) Ras

c) Cyclin-dependent kinase

Which of the following intermediate filaments are found in cancer cells? a) Vimentin b) Desmin c) Cytokeratin d) Tubulin e) Glial fibrillary protein

c) Cytokeratin

Human papilloma virus HPV is involved in cervical cancer: a) Ras activation b) Bcl-2 activation c) Deactivation of p53 d) activation of myc e) Rb-1 activation

c) Deactivation of p53

Which circumstance(s) affecting the Rb gene on chromosome 13 evokes the formation of retiboblastoma? a) Chromosomal translocation b) Gene mutation c) Deletion in both copies of the gene. d) Amplification in both copies of the gene. e) Deactivation in viral infections

c) Deletion in both copies of the gene.

The general symptoms of inflammation do NOT include: a) Leukocytosis b) Hyperemia c) Fever d) Increased sedimentation

c) Fever -Leukocytosis: more WBC -Hyperemia: high blood flow to tissues -Sedimentation: sedimentation rate is common blood test that is used to detect and monitor inflammation in the body. The sedimentation rate is also called the erythrocyte sedimentation rate because it is a measure of the speed that the red blood cells (erythrocytes) in a tube of blood fall to the bottom of the tube, or sediment Redness. Joint swelling.Joint pain.Joint stiffness.Loss of joint function.

White thrombus is mainly composed of a) Erythrocytes b) Hyaline c) Granulocytes d) Fibrin and thrombocytes e) Monocytes

c) Granulocytes d) Fibrin and thrombocytes

The clinical picture of pelvic endometriosis does not include: a) infertility b) dysmenorrhea c) HPV infection d) ... menstruation

c) HPV infection A condition in which the tissue normally found inside the uterus (the endometrium) begins to grow outside of the uterus "" on the bladder, ovaries, bowel and pelvic sidewall.

All of the following statements about oncogenic RNA viruses are true, except: a) contain genes that have ancient origins in the genes of their host cells b) can transform cell effect on the host cells own genes c) HPV virus is an example d) are linked with the formation of leukemia in humans e) are linked to tumor formation in animals

c) HPV virus is an example

Which of the following is/are NOT a cause of hepatic steatosis: a) mushroom poisoning b) anemia c) hepatitis B infection d) hyperlipidemia

c) Hepatitis B was correct in one course test, but the statement was deleted.. Because HepB CAN actually cause steaotsis. - Hepatic steatosis: also known as fatty liver disease (FLD), is a reversible condition wherein large vacuoles of triglyceride fat accumulate in liver cells via the process of steatosis (i.e., abnormal retention of lipids within a cell). Despite having multiple causes, fatty liver can be considered a single disease that occurs worldwide in those with excessive alcohol intake and the obese (with or without effects of insulin resistance.

Aflatoxin B1, a chemical carcinogen produced by mold-contaminated food, evokes: a) Lung carcinoma of transitional epithelium b) Rectal adenocarcinoma c) Hepatocellular carcinoma d) Squamous cell carcinoma of the skin e) Kidney cancer

c) Hepatocellular carcinoma

Which of the following best describes fluid exudate? a) Increased sodium content b) Low specific weight c) Increased protein content d) Chronic fluid accumulation e) Presence of anasarca

c) Increased protein content

What is true about thrombosis? a) Does not occur in flowing blood b) Does not arise in the heart c) Infected thrombus is a source of pyemia d) Obturating thrombus always induces ischemic necrosis e) Only arises in the lower extremities

c) Infected thrombus is a source of pyemia

Which of the following malignant tumors have increased occurrence in AIDS patients? a) Glioma b) Cystadenoma c) Large B-Cell lymphoma d) Rhabdomyosarcoma e) Hemangioendothelioma

c) Large B-Cell lymphoma

Which of the following conditions will macroscopically present like a hemorrhagic coagulative necrosis? a) Nutmeg liver b) Lung abscess c) Lung infarction d) Kidney infarction e) Traumatic hematoma

c) Lung infarction

In which of the following organs will we not fint thromboemboli from the left ventricle? a) Brain b) Kidneys c) Lungs d) Small intestine e) Spleen

c) Lungs

Epidural bleeding is due to rupture: a) Lenticulostriate arteries b) Aneurysm of Willis circle c) Medial meningeal artery d) Cavernous hemangioma

c) Medial meningeal artery

The most common benign tumor of the mammary gland is: a) lipoma b) benign phyllodes tumor c) fibroadenoma d) leiomyoma

c) fibroadenoma Phyllodes tumor : fibroepithelial tumor, cystosarcoma

Pulmonary tuberculosis is not: a) military b) cavernous c) gangrenous d) caseous

c) gangrenous

Which of these tumors is rare in children? a) Medulloblastoma (infratentorial tumors) b) neuroblastoma c) glioblastoma d) astrocytoma (most common benign in children)

c) glioblastoma Glioblastoma multiforme (GBM) is the most common and most aggressive malignant primary brain tumor in humans, involving glial cells and accounting for 52% of all functional tissue brain tumor cases and 20% of all intracranial tumors. GBM is a rare disease

Tumors of childhoods do NOT include: a) Wilms tumors b) ALL c) glioblastoma d) invasive ductal carcinoma of the breast

c) glioblastoma d) invasive ductal carcinoma of the breast -Wilms tumor: cancer of the kidneys that typically occurs in children, rarely in adults. Most nephroblastomas are unilateral, being bilateral in less than 5% of cases, although patients with Denys-Drash syndrome mostly have bilateral or multiple tumors

icterus: a) means only an overall yellow color of the body tissues b) in total bilirubin icterus are colored in all tissues of the organism c) hematoidin has the composition as bilirubin d) icteric coloration is irreversible

c) hematoidin has the composition as bilirubin

Which of the following is/are not a cause of hepatic steatosis: a) mushroom poisoning b) anemia c) hepatitis B infection d) hyperlipidemia

c) hepatitis B infection - Hepatic steatosis: also known as fatty liver disease (FLD), is a reversible condition wherein large vacuoles of triglyceride fat accumulate in liver cells via the process of steatosis (i.e., abnormal retention of lipids within a cell). Despite having multiple causes, fatty liver can be considered a single disease that occurs worldwide in those with excessive alcohol intake and the obese (with or without effects of insulin resistance.

Which type of jaundice can develop as a consequence of severe cardiac insufficiency? a) obstructive b) hemolytic and obstructive c) hepatocellular d) there is no association with icterus

c) hepatocellular

Which of the following is/are not associated with the celsus signs of inflammation? a) calor b) tumor c) hyperemia d) rubor

c) hyperemia

A large hemorrhage is encountered in the region of the basal ganglia during the autopsy of a 56 year old man, along with penetration of blood into the brain ventricles. Which of the following findings closely relates to the emergence of bleeding in this patient? a) atrial septal defect b) calcification of aortic valve c) hypertrophy of left ventricle d) atherosclerotic plaque in right carotid e) lung embolism

c) hypertrophy of left ventricle

Which of the following statements is not true about cystic fibrosis? a) it affects exocrine glands b) recurrent bronchopneumonia c) is complicated by diabetes mellitus d) can cause intestinal perforation

c) is complicated by diabetes mellitus Cystic fibrosis (CF), also known as mucoviscidosis, is an autosomal recessive genetic disorder that affects mostly the lungs but also the pancreas, liver, and intestine. Difficulty breathing is the most serious symptom and results from frequent lung infections. Other symptoms—including sinus infections, poor growth, and infertility—affect other parts of the body. CF is caused by one of many different mutations in the gene for the protein cystic fibrosis transmembrane conductance regulator (CFTR). This protein is required to regulate the components of sweat, digestive fluids, and mucus

Embryonic carcinoma: a) is a malignant tumor affecting the embryo b) belongs among the congenital tumors c) is highly malignant d) producing high quantity of hCG e) no tumor of this name do exist

c) is highly malignant

For rheumatoid arthritis the following applies: a) it is a migratory polyarthritis b) leaves lasting effects on bones and joints c) is part of the manifestations of acute rheumatic fever d) is accompanied by formation of caseous epithelioid granulomas

c) is part of the manifestations of acute rheumatic fever (is it so that migratory polyarthritis is categorised under rheumatoid fever, not under rheumatoid arthritis?)

Which of the following is/are true of embryocarcinoma? a) malignant tumor affecting the fetus b) include congenital tumors c) it is highly malignant d) there is no such thing as embrocarcinoma

c) it is highly malignant

Which of the following is/are true of chordoma? a) spinal cord tumor b) benign tumor of cartilage c) it is usually composed of cells containing glycogen d) often located in the upper jaw

c) it is usually composed of cells containing glycogen

Which of the following is/are true of pancreatic pseudocyst? a) is lined by epithelium b) it is a congenital anomaly associated with cystic fibrosis c) its wall is formed by connective tissue lacking epithelium d) is of neoplastic origin, lined by dysplastic epithelium

c) its wall is formed by connective tissue lacking epithelium

Which of the following is least likely to change in a child dying of acute rheumatic fever? a) Aschoff nodules b) pericarditis c) large decaying vegetations on the mitral valve d) increased titer of antistreptolysin O (ASO titre)

c) large decaying vegetations on the mitral valve

Which of the following cells are not associated with Hodgkin's lmphoma? a) RS cells b) - c) lymphoblasts d) cells of Hodgkin

c) lymphoblasts

Granulation tissue consist of: a) macrophages and plasma cells and granulocytes b) lymphocytes, epithelioid cells and giant cells c) macrophages, capillaries, fibroblasts or inflammatory cells d) granuloma

c) macrophages, capillaries, fibroblasts or inflammatory cells

Which of the following assertion is true for adenocarcinoma of prostate: a) primary cause is alcohol b) caused by viral infection c) mainly metastasize to bone d) 5 year survival is about 5 % e) Most often in age group between 20-40 years

c) mainly metastasize to bone

Which of the following statements is/are true of prostate adenocarcinoma? a) the main cause is alcohol b) it is caused by viral infections c) mainly metastasizes to bone d) it is more often frequent in the age group 20-40 years

c) mainly metastasizes to bone

Which of the following tumors belong among the soft tissue tumors? a) serous carcinoma b) embryonic carcinoma c) malignant fibrous histiocytoma d) hemangioma

c) malignant fibrous histiocytoma d) hemangioma

Which of the following statements/s is/are not correct? a) the behavior of sarcomas is usually determined by the least differentiated component b) sarcomas arise from the mesoderm c) malignant mesenchymal tumors are more common than malignant tumors of epithelial origin d) non-Hodgkin lymphomas are more common in immunosuppressed patients

c) malignant mesenchymal tumors are more common than malignant tumors of epithelial origin Epithelial tumors are more often malignant!!

Which statement is incorrect: a) mural thrombus can be seen by the naked eye b) red thrombus is stagnating c) mural thrombi are formed in capillaries d) in the heart thrombi arise only in atrium

c) mural thrombi are formed in capillaries d) in the heart thrombi arise only in atrium Mural thrombi are thrombi that adhere to the wall of a blood vessel. They occur in large vessels such as heart and aorta, and can restrict blood flow but usually do not block it entirely. They appear grey-red with alternating light and dark lines (known as lines of Zahn) which represent bands of fibrin (lighter) with entrapped white blood cells and red blood cells (darker).

Acute myeloid leukemia is characterized by the presence of: a) lymphoblasts b) RS cells c) myeloblasts d) lymphohistiocytic cells

c) myeloblasts A cancer of the myeloid line of blood cells, characterized by the rapid growth of abnormal white blood cells that accumulate in the bone marrow and interfere with the production of normal blood cells. AML is the most common acute leukemia affecting adults.

.

c) neoplastic T lymphocytes Mycosis fungoides, also known as Alibert-Bazin syndrome or granuloma fungoides, is the most common form of cutaneous T-cell lymphoma. It generally affects the skin, but may progress internally over time. Characterized by "Pautriers Microabscesses", probably why a and b are wrong.. :)

Mycosis fungoides is characterized by the presence of: a) papillary microabscess b) abscesses c) neoplastic T lymphocytes d) neoplastic B lymphocytes

c) neoplastic T lymphocytes Mycosis fungoides, also known as Alibert-Bazin syndrome or granuloma fungoides, is the most common form of cutaneous T-cell lymphoma. It generally affects the skin, but may progress internally over time. Characterized by "Pautriers Microabscesses", probably why a and b are wrong.. :)

Wilms tumor is another name for: a) hepatoblastoma b) neuroblastoma c) nephroblastoma d) retinoblastoma

c) nephroblastoma Wilms' tumor, or nephroblastoma is cancer of the kidneys that typically occurs in children, rarely in adults. Most nephroblastomas are unilateral, being bilateral in less than 5% of cases. They tend to be encapsulated and vascularized tumors that do not cross the midline of the abdomen. In cases of metastasis it is usually to the lung

Deficiency of alpha-1-antitrypsin causes emphysema: a) compensatory b) congenital c) panacinar d) centroacinar

c) panacinar There are two major types of emphysema: centrilobular (centriacinar) and panlobular (panacinar). The former involves primarily the upper lobes while the latter involves all lung fields, particularly the bases Alpha 1-antitrypsin deficiency is a genetic disorder that causes defective production of alpha 1-antitrypsin, leading to decreased A1AT activity in the blood and lungs, and deposition of excessive abnormal A1AT protein in liver cells. Severe A1AT deficiency causes panacinar emphysema or COPD in adult life in many people with the condition (especially if they are exposed to cigarette smoke).

Which of the reported properties of tumor cells is involved in tumor invasiveness: a) increasing mutual cohesion of tumor cells and their increased adhesion to the substrate b) decrease in the number of laminin receporu c) production of metalloproteinases d) overexpression of bcl-2 e) the presence of the t(8:14)

c) production of metalloproteinases

Where is probably the primary cancer in a man with osteoblastic metastasis in bone: a) large intestine b) lung c) prostate d) kidney

c) prostate

Between retention cysts belongs: a) dermoid cyst b) ganglion c) ranula (mucocele) d) echinococcus cyst e) cystic spaces dilated cerebral ventricles in hydrocephalus

c) ranula (mucocele)

Which of the following pairs of pseudotumors are WRONG? a) pseudocyst- cavity without actual lining b) chocolate cyst- posthemorrhagic origin c) retention cyst- clogging of the epithelium d) parasitic cyst- chubby tapeworm e) implantation cyst- pin plug of sebaceous glands

c) retention cyst- clogging of the epithelium e) implantation cyst- pin plug of sebaceous glands

Malignant neoplasms of mesenchymal origin are referred to as: a) teratoma b) carcinoma c) sarcoma d) fibroma e) hamartoma

c) sarcoma

Which of the following substances is/are most commonly produced by intestinal carcinoid tumors? a) adrenaline b) heparin c) serotonin d) vanillylmandelic acid

c) serotonin

Which statement is true? a) inhalation of amorphous carbon and pulmonary fibrosis b) asbestosis and the development of hepatocellular carcinoma c) silicosis and silicotic nodules d) silicosis and acute cor pulmonale

c) silicosis and silicotic nodules Silicosis is a form of occupational lung disease caused by inhalation of crystalline silica dust, and is marked by inflammation and scarring in the form of nodular lesions in the upper lobes of the lungs. It is a type of pneumoconiosis

Septic shock occurs from a) excessive loss of volume of the kidneys and gastrointestinal pressure b) loss of autonomic control of heart activity c) systemic vasodilation and increased vascular permeability d) uncontrolled vasoconstriction e) sudden bleeding

c) systemic vasodilation and increased vascular permeability

Which of the following tumors belongs among germ cell tumors? a) embryonic rhabdomyosarcoma b) Wilms tumor c) teratoma d) medulloblastoma

c) teratoma

Which of the following is/are not true of the basic mechanisms of steatosis? a) lipoprotein synthesis disorders b) disorders of beta-oxidation in mitochondria c) the failure of cell membrane permeability d) lysosomal dysfunction

c) the failure of cell membrane permeability

Which is true of hypertrophy of the heart: a) there is a proliferation of cardiomyocytes b) can only be determined microscopically c) the main parameter for the diagnosis is heart weight d) may be reversible

c) the main parameter for the diagnosis is heart weight d) may be reversible

What is true of hypertrophy of the heart: a) there is a proliferation of cardiomyocytes b) can be determined only microscopically c) the main parameter for the diagnosis is the heart weight d) may be reversible

c) the main parameter for the diagnosis is the heart weight d) may be reversible

Chronic pancreatitis is characterized by: a) high levels of serum amylase b) high levels of amylase in urine c) the presence of pancreatic pseudocysts d) the presence of balser necrosis

c) the presence of pancreatic pseudocysts

What is critical for staging of tumors? a. The type of tumor b. The maturity of the tumor c. The size of the tumor d. Metastases e. The number of mitoses per 10 fields of vision at a magnification of 40x (so-called high-power field / HPF)

c. The size of the tumor d. Metastases

Which of the following statements about smallpox are correct: a. It still poses a serious medical problem in third-world countries b. It has fecal-oral transmission c. The virus cause degeneration of epidermal cells, giving rise to multilocular blisters d. The main symptom of the disease is jaundice during centrolobular necrosis of hepatocytes e. The disease cause re-activation of endogenous latent viruses which lead to serious complications

c. The virus cause degeneration of epidermal cells, giving rise to multilocular blisters

Which of the following infectious agents cause transplacental infections in the fetus a. Entamoeba histolytica b. Giardia lamblia c. Toxoplasma gondii d. Histoplasma capsulatum e. Candida albicans

c. Toxoplasma gondii parasitic protozoan that causes the disease toxoplasmosis. Acute toxoplasmosis is often asymptomatic in healthy adults. However, symptoms may manifest and are often influenza-like: swollen lymph nodes, or muscle aches and pains that last for a month or more. Rarely will a human with a fully functioning immune system develop severe symptoms following infection

Which of the following diseases are not caused by Rickettsia a. Rocky Mountain Spotted fever b. Ehrlichisois c. Typhoid fever d. Q fever e. Bacillary angiomatosis

c. Typhoid fever

Which virus are not oncogenic: a. Papilloma virus b. EBV c. Virus poliomyelitis acuta anterior d. HSV type 1 e. HSV type 2

c. Virus poliomyelitis acuta anterior d. HSV type 1 e. HSV type 2

Which vitamin deficiency manifests with cheilosis (anguli infectiosi) a. Vitamin B1 deficiency b. Vitamin B12 deficiency c. Vitamin B2 deficiency d. Vitamin A deficiency e. Vitamin E deficiency

c. Vitamin B2 deficiency Fissuring and dry scaling of the vermilion surface of the lips and angles of the mouth, a characteristic of riboflavin deficiency (Vit.B2)

Which of the following vitamins are not water-soluble: a. Biotin b. Niacin c. Vitamin D d. Thiamine (B1) e. Riboflavin (B2)

c. Vitamin D - fat soluble (ADEK)

The tumor containing, extraembryonal mesoderm, is referred to as: a. Dermoid cyst b. Choristom c. Yolk sac tumor d. Teratoma e. Hamartoma

c. Yolk sac tumor

Fibrinoid changes of connective tissue typically occurs during a. Hyperuricemia b. Diabetes c. Rheumatic fever d. Myxedema e. Chronic irritation

c. rheumatic fever

d) reaction of the bronchial mucosa to irritation by tobacco smoking

correct

choose the statement(s) valid for sialothiasis: a) the concretions contain silicate ions b) it most often affect the parotid gland c) does not affect the submandibular gland d) manifests with painful swelling og the affected gland

d

Select the valid statement(s) about tuberculosis: 1,In comparison with the disease-causing agent of measles, tuberculosis is extremely contagious 2. The transmission of infection to humans from cow's milk is not possible 3. The disease-causing agent belongs to the same genus as the agent of dysentery 4. The disease-causing agent belongs to the same genus as the agent of leprosy

d (tbc- mycobacteria tuberculosis, leprosy= mycobaterium leprae)

What is the origin of amyloid deposits in patients with B non-Hodgkins lymphoma? a. AA amyloid b. Beta-amyloid c. Beta-2-microglobulin d. AL amyloid e. AE amyloid

d AL -amyloid (amino light chain) (A-amino, L-light chain) just a way to remember it :)

What is the origin of amyloid deposits in the kidney in nephrotic syndrome from multiple myeloma a. AA amyloid b. Beta-amyloid c. Amyloid derived from transthyretin d. AL amyloid e. AE amyloid

d AL amyloid

which of the following manifestations don`t belong to certain/sure death signs a. Postmortem spots (Livores) b. Postmortem stiffness (Rigor mortis) c. Cold (Algor mortis) d. Areflexia e. Brain death

d Areflexia is not s certain sign of death

A 56-year-old obese lady (BMI = 32 kg/m2) comes in with the development of stenosis of the ureter. Which adaptive changes from chronic ischemia will be displayed by the affected kidney a. Hyperplasia b. Hypertrophy c. Neoplasia d. Atrophy e. Dysplasia

d Atrophy

What causes bleeding in DIC? - a) Intracapillary increased pressure due to slow circulation - b) Toxic hypoxic damage to the capillaries - c) Erythrocyte hemolysis - d) Consumed clotting factors - e) Inhibition of fibrinolysis

d Consumed clotting factors

Hepatocytes undergoing apoptosis from viral disease are represented like a. Barr bodies b. Fibrinoid bodies c. Apoptosomes d. Councilman bodies e. Corpora amylacea

d Councilman bodies

Cells exposed to hypoxia can have all of the following consequences except a. Damaged oxidative phosphorylation b. Decrease in ATP reserve c. Release of enzymes to the blood d. Decrease of glycolysis e. Decrease of cell volume

d Decrease of glycolysis

Amyloid precusors in the serum don't include a. SAA b. Beta-2-microglobulin c. Light-chain immunoglobulins d. Fibrinogen e. Prealbumin

d Fibrinogen

Intravascular fat particles transmitted to a remote place from their place of origin are linked to a) ascites b) decompression sickness c) lupus anticoagulant d) fracture of long bones e) deep vein thrombosis

d Fracture of long bones

Russel bodies are represented like a. Inclusions in Rabies-infected ganglion cells b. Intranuclear inclusions in epidermal cells in Varicella c. Intracytoplasmic bodies in Variola d. Gammaglobulins in plasma cells e. Inclusions of products from the ubiquitination process in old people

d Gammaglobulins in plasma cells|

In which of the following conditions can one see hydropic degeneration of the epithelia of the renal tubules a. Chronic alcoholism b. Tetrachlormethane/CCl4 poisoning c. Excessive renal loss of sodium d. Hypokalemia e. Lead poisoning

d Hypokalemia

A 62-year-old desoriented man is brought to the hospital. During physical exam there is obvious jaundice, splenomegaly, and ascites. Serum levels of ALT, AST, ALP, and GMT are increased. Liver biopsy reveals alcoholic hepatitis and Mallory hyaline/bodies. This substance is produced from aggregation of which of the following proteins a. Alpha-1-antitrypsin b. Beta-amyloid c. Alpha-synuclein d. Intermediate filaments e. Prion-related protein (PrP)

d Intermediate filaments

What is the correct term for amyloidosis of the heart based on atrial natriuretic polypeptide a. AA amyloid b. Beta-amyloid c. Beta-2-microglobulin d. Isolated atrial amyloidosis

d Isolated atrial amyloidosis

Which of the following applies to acute gout a. It can occur after a large ingestion of chicken meat in a short period b. It can be caused by taking ASA in hypersensitive individuals c. It`s a primary metabolic defect d. It can occur after therapy for hematologic malignancies e. Cholesterol ester crystals can be demonstrated

d It can occur after therapy for hematologic malignancies

In which of the following diseases are the injury mainly to glial cells a. Marfan's disease b. Fabry's disease c. Niemann-Pick disease d. Krabbe disease e. Anderson disease

d Krabbes disease

Most common complication of thrombosis of lower extremities is a) brain infarct b) kidney infarct c) myocardial infarct d) lung infarct e) intestinal infarct

d Lung infarct

Atrophy is least likely caused by a. Deinnervation b. Reduced arterial blood supply c. Aging d. Progressive hyperplasia e. Reduction of hormonal stimulation

d Progressive hyperplasia

Defect of hexosaminidase A is the cause of which disease a. Von Gierke disease b. Pompe disease c. McArdle disease d. Tay-Sachs disease e. Wilson`s disease

d Tay Sachs disease

During autopsy of a 80-year-old man without known genetic disease, there were found subtle deposits of amyloid in the heart. There were not found amyloid deposits in any of the other organs and the patient did not have a history of any chronic inflammatory conditions. What will be the most likely composition of the amyloid a. Amyloid-associated protein (AAP) b. Light-chain immunoglobulins c. Beta-2-microglobulin d. Transthyretin e. Beta-2-amyloid

d Transthyretin (it has a normal structure, it's just deposited due to old age) (senile systemic amyloidosis)

To the morphologic manifestations of neurogenic muscular atrophy belongs a. Absence of dystrophy b. Perivascular mixed inflammatory infiltrate c. Subtle dispersed necrosis of muscle fibers d. Atrophic angular muscle fibers e. Zenker degeneration

d atrophic angular muscle fibers

Which of the following is/are true of B and T lymphocytes? a) the ratio of B:T cells = 65:35 b) T lymphocytes produce IgE c) a small lymphocyte is a final stage cell, which cannot transform d) B lymphocytes provide humoral and T lymphocytes cellular immunity

d) B lymphocytes provide humoral and T lymphocytes cellular immunity

Which claim is true of retrovirus (oncogenic RNA virus) who are in no oncogene: a) Cells can not go transform b) Cells transform faster than its counterparts bearing oncogenes c) transformations is necessary for co-infection with oncogenic DNA virus d) Cells can transform, while it is advertising to its proximity to important cellular gene e) They include EBV virus

d) Cells can transform, while it is advertising to its proximity to important cellular gene

Which of the following statements is/are not true of plasmacytoma? a) it is often accompanied by amyloid deposition b) it is often accompanied by paraproteinuria c) belongs among B-NHL d) Creates osteplastic metastasis

d) Creates osteplastic metastasis

Which of the following cytoskeletal protein is present in malignant epithelial tumors in different sites of the human body? a) Glial fibrillary acidic protein (GFAP) b) Prostate-specific antigen (PSA) c) Desmins d) Cytokeratin e) Vimentin

d) Cytokeratin

Under what circumstances occur air embolism? a) In the state of weighnessless b) Carbon monoxide poisoning c) During rapid descend under the surface d) During rapid ascend to the surface e) With fast release of oxygen from blood in the form of bubbles

d) During rapid ascend to the surface

Extensive edema of the whole body is most common seen in a) Lymphatic obstruction b) Damaged capillaries c) Inflammation d) Hypoalbuminemia e) Angiogenesis

d) Hypoalbuminemia

Which of the following is a transcription factor mediator: a) Rb b) WT-1 c) Abl d) Myc e) Ras

d) Myc

Characteristics of neurogenic, septic, and cardiogenic shock: a) Excellent prognosis if treated b) Need blood transfusion c) Peripheral vasodilation in early stages d) Normal circulatory blood volume in early stages e) Increased urine output

d) Normal circulatory blood volume in early stages

What disease is not caused by exotoxin action: a) diphtheria b) tetanus c) botulism d) Pseudomonas sepsis

d) Pseudomonas sepsis Pseudomonas aeruginosa is a common bacterium that can cause disease in animals, including humans. It is citrate, catalase, and oxidase positive. It is found in soil, water, skin flora, and most man-made environments throughout the world. It thrives not only in normal atmospheres, but also in hypoxic atmospheres, and has, thus, colonized many natural and artificial environments. It uses a wide range of organic material for food; in animals, its versatility enables the organism to infect damaged tissues or those with reduced immunity. The symptoms of such infections are generalized inflammation and sepsis. If such colonizations occur in critical body organs, such as the lungs, the urinary tract, and kidneys, the results can be fata

Which of the following protein(s) is/are affected in the tumor transformation caused by HPV? a) Src b) Ras c) Bcl-2 d) Rb e) Desmin

d) Rb

In such manner do carcinomas and sarcomas metastatsize: a) often simultaneously based lymphogenous and hematogenous metastases b) Both types will first metastasize to regional lymph nodes c) perineural spread d) Sarcomas metastasize more often hematogenously and carcinomas more often lymphogenously e) Sarcomas metastasize more often lymphogenously and carcinomas more often hematogenously

d) Sarcomas metastasize more often hematogenously and carcinomas more often lymphogenously

Growth factor produced by certain tumor cells that has the same receptor as epidermal growth factor (EGF) is called: a) PDGF (platelet derived growth factor) b) TGF beta (tumor growth factor beta) c) IGF (insulin-like growth factor) d) TGF alpha (Transforming growth factor) e) TNF (tumor necrosis factor)

d) TGF alpha (Transforming growth factor)

What is true of germinal tumors? a) occur only in the gonads b) are always malignant c) includes hamartomas d) are derived from germ cells

d) are derived form germ cells - Germinal tumors: a germ cell tumor (GCT) is a neoplasm derived from germ cells. - Germ cell tumors can be cancerous or non-cancerous tumors. Germ cells normally occur inside the gonads (ovary and testis). - Despite their name, germ cell tumors occur both within and outside the ovary and testis - Hamartomas: a benign, focal malformation that resembles a neoplasm in the tissue of its origin. This is not a malignant tumor, and it grows at the same rate as the surrounding tissues. They result from an abnormal formation of normal tissue.

Which of the following direction of embolism is/are least likely to appear? a) artery - artery b) vein - vein c) heart - artery d) artery - vein

d) artery - vein

Which of the following ways of metastasis DOES NOT exist: a) porogenni b) hematogenous c) lymphogenous d) autogenous e) implantation metastasis

d) autogenous

Which of the following is/are characteristic for Grawitz renal cell carcinoma? a) other options are not correct b) it is most common in children than in adults c) it consists of very bright cells d) bleeding into the tumor tissue is frequently present

d) bleeding into the tumor tissue is frequently present

Dermatitis herpetiformis can be associated with: a) herpes simplex labialis b) primary sclerosing cholangitis c) Crohn's disease d) celiac sprue (disease of small intestine causing malabsorbtion of food in particular)

d) celiac sprue (disease of small intestine causing malabsorbtion of food in particular) It is a chronic blistering skin condition, characterised by blisters filled with a watery fluid. Despite its name, DH is neither related to nor caused by herpes virus: the name means that it is a skin inflammation having an appearance similar to herpes

Salmonella typhi abdominalis causes a) dysentery b) cholera nostras c) spotted typhus d) changes in lymphoid tissue of the small intestine

d) changes in lymphoid tissue of the small intestine

Chorionic gonadotropin is used for monitoring: a) Wilms tumor b) endometrial cancer c) seminoma d) choriocarcinoma e) embryonal carcinoma

d) choriocarcinoma

Great splenomegaly is a prominent feature, especially in? a) acute lymphoblastic leukemia b) acute myeloid leukemia c) acute myelomonocytic leukemia d) chronic myeloid leukemia

d) chronic myeloid leukemia

Which statement most accurately defines mechanism of shock a) result from bleeding b) it is an insufficiency of the circulatory system, with insufficient supply of the periphery c) circulatory insufficiency caused by heart failure d) circulatory insufficiency caused by mismatch between cardiac output and peripheral requirements e) it is a consequence of peripheral acidosis

d) circulatory insufficiency caused by mismatch between cardiac output and peripheral requirements

Which of the following changes is related to the presence of components in invasive malignant tumors: a) dysplasia b) metaplasia c) hperplasia d) desmoplasia e) anaplasia

d) desmoplasia

Which of the following factor do NOT have relationship to the process of invasion and metastasis of malignant tumors: a) tumor cell surface receptors for laminin b) properties of tumor cells to bind to platelets c) production of proteolytic enzyme d) serum lactate dehydrogenase e) angiogenesis

d) serum lactate dehydrogenase

.

d) the intracellular accumulation of lipofuscin A brownish tissue discolouration caused by lipofuscin ("ageing" pigment) deposition in certain organs—e.g., heart, liver, and others—which may occur in older individuals. Organs affected by brown atrophy are small and flabby

Which of the following is/are true of brown atrophy? a) numerical atrophy secondary to connective tissue proliferation b) accumulation of hemosiderin in the cells, leading to their atrophy c) the accumulation of lipofuscin in atrophic interstitium of damaged organs d) the intracellular accumulation of lipofuscin

d) the intracellular accumulation of lipofuscin A brownish tissue discolouration caused by lipofuscin ("ageing" pigment) deposition in certain organs—e.g., heart, liver, and others—which may occur in older individuals. Organs affected by brown atrophy are small and flabby

Which of the following is/are true of pathological immune reactions I. type? a) local contact symptoms include dermatitis b) IgE binds to eosinophils and cause their degranulation c) causes only local symptoms such as hay fever d) the main change is histamine released from mast cells

d) the main change is histamine released from mast cells

Acute lymphoblastic leukemia is characterized by: a) the proliferation of mature peripheral lymphocytes b) the proliferation of neoplastic histiocytes c) to proliferation of myeloblasts d) the proliferation of tumor elements with the phenotype of progenitor cells

d) the proliferation of tumor elements with the phenotype of progenitor cells Acute lymphoblastic leukemia (ALL) or acute lymphoid leukemia is an acute form of leukemia, or cancer of the white blood cells, characterized by the overproduction of cancerous, immature white blood cells—known as lymphoblasts. In persons with ALL, lymphoblasts are overproduced in the bone marrow and continuously multiply, causing damage and death by inhibiting the production of normal cells—such as red and white blood cells and platelets—in the bone marrow and by spreading (infiltrating) to other organs. ALL is most common in childhood.

For pulmonary embolism does not apply: a) it is usually clinically silent b) the result is cor pulmonale c) it affects women using hormonal contraception d) the pulmonary infarction results in every time

d) the pulmonary infarction results in every time Pulmonary heart disease, also known as Cor pulmonale (Latin cor, heart + of the lungs) is the enlargement and failure of the right ventricle of the heart as a response to increased vascular resistance or high blood pressure in the lungs (pulmonary hypertension)

What is true of metaplasia: a) a change in cell differentiation (Not change in differenciation, only in the epithelium) b) it is merely a change of cell proliferation c) it is a change in epithelial cell differentiation only d) the result is the formation of differentiated cells of another type than the original type

d) the result is the formation of differentiated cells of another type than the original type

Early metastasis in lymph nodes are found in: a) the hilum b) the central sinuses c) the follicles d) the subcapsular sinuses e) pouzdru

d) the subcapsular sinuses

The fragmentative bone fractures: a) there is no fat embolism b) there is no cellular embolism c) there can be a thrombotic embolism d) there can be a cellular and thrombotic embolism

d) there can be a cellular and thrombotic embolism Think C is correct as well

Known as intravital blood clot formation in the arteries or veins a) congestion b) emboli c) hemorrhage d) thrombosis e) infarct

d) thrombosis

Example of retrograde emboli is a) thombus travelled from femoral artery to cerebral artery b) loss of autonomous control of heart activity c) released from left atrium sweeping into hepatic vein d) thrombus from IVC sweeping into hepatic vein e) thrombus from femoral vein sweeping into the pulmonary artery

d) thrombus from IVC sweeping into hepatic vein In retrograde embolism, however, the emboli move in opposition to the blood flow direction; this is usually significant only in blood vessels with low pressure (veins) or with emboli of high weight

Which of the following polyps are most likely to transform into a malignant tumor? a) hyperplastic polyp b) juvenile polyp c) Peutz-Jeghers polyp d) villous adenoma

d) villous adenoma

In which polyp is the malignant transformation most often? a) tubular adenoma b) juvenile polyp c) hyperplastic polyp d) villous adenoma

d) villous adenoma Juvenile polyposis syndrome is a syndrome characterized by the appearance of multiple juvenile polyps in the gastrointestinal tract. Polyps are abnormal growths arising from a mucous membrane.

Metastasis always occurs: a) when a pathological process penetrates into blood vessels b) when a pathological process penetrates into lymph vessels c) when a pathological process spread into surrounding tissues d) when a pathological process secondarily settles in locations remote from the original site

d) when a pathological process secondarily settles in locations remote from the original site -Metastasis: is the spread of a cancer or disease from one organ or part to another not directly connected with it.

Schiller-Duvall bodies are found in: a) embryonal carcinoma b) embryonal rhabdomyesarcoma c) medulloblastoma d) yolk sac tumor e) nephroblastoma

d) yolk sac tumor

Which of the following features is most characteristic for malignant tumors? a. Encapsulated b. Expansive growth c. Large size d. Anaplasia e. Abnormal hormone secretion

d. Anaplasia a condition whereby cells lose the morphological characteristics of mature cells and their orientation with respect to each other and to endothelial cells

Cholera is characteristic as: a. Mucosal ulcerations of the small intestine b. Granulomas under Peyer's patches c. Cryptal abscesses of the large intestine d. Extensive watery diarrhea e. Peritonitis

d. Extensive watery diarrhea

Which of the following terms don`t belong to steatosis from lysosomal dysfunction a. Tigered effect on the heart b. Green Amanita/Amanita phalloides mushroom poisoning c. Phosphorous intoxication d. Gangliosidosis e. Starvation

d. Gangliosidosis

What are examples of lipomatosis a. Aggregations of cholesterol in macrophages b. Aggregations of neutral fat in hepatocytes during alcoholic liver damage c. Digestion of adipose tissue of the peritoneum during acute necrotic pancreatitis d. The presence of adipocytes between muscle fibers of the right ventricle of the heart e. Hyperlipidemia during nephrotic syndrome

d. The presence of adipocytes between muscle fibers of the right ventricle of the heart

Basement membrane and/or extracellular matrix is composed of all of the following components, except a. Collagen type IV b. Collagen type III c. Laminin d. Fibronectin e. Lamin

e : Lamin

What is the origin of deposits of amyloid in the islets of Langerhans in a patient with diabetes mellitus a. AA amyloid b. Beta-amyloid c. Amyloid derived from transthyretin d. AL amyloid e. AE amyloid

e AE amyloid deposits in the islets (I think AE amyloid= Endocrine type....AE includes both medullary carcinoma of thyroid , calcitonin, and Islets of Langerhans- Islet Amyloid Peptide)

A 28 year-old lady is confined to bedrest after a fracture of the tibia. When the cast is taken of after 6 weeks, her leg is weak and the patient notices that in comparison with the other the circumference is reduced. Which of the following terms best describes the change that has taken place in the affected musculature a. Metaplasia b. Ischemic necrosis c. Hyperplasia d. Irreversible cellular injury e. Atrophy

e Atrophy

Which of the following conditions are examples of coagulation necrosis a. Lobar pneumonia in alcoholics b. Liver abscesses in patients with amebiasis c. Pseudomembranous colitis in patients using ampicillin d. Cerebral atrophy in patients with Alzheimer`s disease e. Embolus to the superior mesenteric artery leading to colonic infarct

e Embolus to the superior mesenteric artery, leading to a colonic infarcts

Which of the following events are part of hydropic cellular changes a. Enzymatic digestion of cellular proteins b. Glycogen accumulation c. Plasma membrane rupture d. Early progression to apoptosis e. Filling of ER with water

e Filling the ER with water

Amyloid have all of the following components except a. Amorphic eosinophilic appearance in histological specimens stained with H and E b. Yellow-green birefringence after staining with Congo red c. Fibrillary structure in electron microscopy d. Stains similarly as starch with Lugol solution e. Induces an inflammatory reaction in the surroundings

e Induces an inflammatory reaction in the surroundings

Which of the following conditions can best capture the concept of passive hyperemia? - a) Reddening of the face in cold - b) Cutaneous vasodilation in a warm day - c) Congestion as a result of muscle exercise - d) Inflammation - e) Nutmeg liver

e Nutmeg liver (congested due to right heart failure)

Which of the following accumulations belongs to lysosomal dysfunction a. Osmotic nephrosis b. Nutmeg liver c. Strawberry gallbladder d. Tiger effect on the myocardium e. Post-inflammatory pesudoxanthoma

e Post-inflammatory pesudoxanthoma

Cystic dilatation of the endoplasmic reticulum is associated with a. Lipid accumulation b. Hypertrophic cells c. Intracellular dehydration d. Apoptosis e. Severe, but reversible ischemic damage

e Severe, but reversible ischemic damage

In which form are lipids accumulated in hepatocytes in hepatic steatosis a. Cholesterol ester b. Fatty acids c. Lipoproteins d. Phospholipids e. Triglycerides

e TAG

For which of the following diseases is apoptosis the main mechanism of injury a. Brown atrophy of the liver b. Alcoholic injury of the liver c. Overdose of barbiturates d. Acute pancreatitis e. Viral hepatitis

e Viral hepatitis

Which statements are correct about glycogenosis type I a. It's an acquired defect of metabolism b. It manifests until the 3rd decade of life c. We find atrophy of the brain d. We find amyloid in the cells of Langerhans e. We find marked enlargement of the liver and brain

e We find marked enlargement of the liver and brain ---> Vernerova - Not correct ?? :)

Which of the following enzymes/molecules are not involved in inactivation of free radicals a. Superoxide dismutase b. Vitamin E c. Catalase d. Glutathion peroxidase e. Xanthine oxidase

e Xanthine oxidase

Which of the following types of cells are the least sensitive to anoxia a. Neurons b. Cardiomyocytes c. Absorptive epithelia of the small intestine d. Proximal tubular cells of the kidney e. Fibroblasts

e fibroblasts

Which of the following substances/organelles are increased in so-called tumefactio turbida (Vernerova - Term don't used anymore. Used to macroscopically describe organ firm and swelled due to increased water content) a. Calcium b. ER c. Lipids d. Potassium e. Water

e water

Which of these proteins is associated with loss of intercellular adhesion of tumor: a) Ras b) c-erbB-2 c) Myc d) p53 e) DCC

e) DCC(deleted in colon carcinoma)

Which of the following mechanism is responsible for xeroderma pigmentosa patients with an unusual susceptibility to the formation of skin tumors: a) Acquired tumor suppressor gene p53 b) Activation of Ras oncogene c) Abnormal melanin synthesis d) sensitivity to infrared radiation e) Defective DNA reparation after UV damage

e) Defective DNA reparation after UV damage

In yolk sac tumors, it can be useful to monitor: a) CA 125 b) CEA(carcinoembryonal antigen) c) hCG d) alkaline phosphatase e) alpha-fetoprotein

e) alpha-fetoprotein

DOES NOT belong among mixed tumors: a) angiolipoma b) fibroadenoma c) adenosarcoma d) carcinosarcoma e) dermoid cyst of the ovary

e) dermoid cyst of the ovary

Ability of chemical carcinogen to induce tumors is influenced by each of the following characteristics EXCEPT: a) ability of cells to DNA repair b) ability to detoxify carcinogen c) ability to activate carcinogen d) The presence of tumor promoter e) extent of damage the target cell plasma membrane

e) extent of damage the target cell plasma membrane

The most important properties which determine whether or not malignant epithelial tumors, is/are: a) positive tumor markers in the blood b) number of mitoses c) nuclear hyperchromazie d) high nucleo-cytoplasmic ratio e) invasive growth

e) invasive growth

The main mechanism of tumor formation by ultraviolet radiation (UVB solar radiation) is: a) change in cellular differentiation b) inhibition of cellular interactions c) stimulate the activity of ras oncogene d) inhibits the activity of ras oncogene e) mutations of DNA

e) mutations of DNA

Which of the following statements about EBV is true? a) viral oncogene that cause cancer b) it is an oncornavirus c) induce formation of intranuclear inclusions d) induce formation of intracytoplasmic inclusions e) negatively affects the expression of bcl-2

e) negatively affects the expression of bcl-2

Which of the following statements are not true concerning carcinogens? a) are mutagens b) affects multiple target organs/tissues c) comes from the environment d) for the development of their carcinogenic potential they must be converted to electrophilic substances e) their effects on cells are irreversible

e) their effects on cells are irreversible

Which of the following manifestations are not seen in epidemic typhus a. Rickettsia in endothelial cells b. Fibrinous thrombi in small blood vessels c. Vasculitis d. Typhoid nodules from mononuclear elements e. Abscesses

e. Abscesses

Which of the following types of neoplasms are caused by arsenic during prolonged exposure: a. Skin neoplasms b. Carinoma of the breast c. Acute leukemia d. Colorectal carcinoma e. Angiosarcoma of the liver

e. Angiosarcoma of the liver Excretion occurs in the urine and long-term exposure to arsenic has been linked to bladder and kidney cancer in addition to cancer of the liver, prostate, skin, lungs, and nasal cavity

select the valid statement(s) about hematin: 1. it has the same chemical composition as bilirubin, it is the basis of the yellow color of older hemorrhages 2. gilberts syndrom belongs among porphyrias 3.brown atrophy of the liver leads to hepatic failure 4. intracellular accumulations of lipofuscin is a future of hemachromatosis

everyone is wrong

d) it is a lung infection of immunosuppresed patients

wrong

Giardiasis is a 1. Mycotic infection 2. Bacterial infection 3. Protozoal infection 4. Autoimmune mechanism

3. Protozoal infection

A typical complication of the rupture of an aneurysm of the circle of Willis is: 1. Subdural bleeding 2. Epidural bleeding 3. Subarachnoideal bleeding 4. Bleeding into the periventricular white matter

3. Subarachnoideal bleeding

A pyogenic membrane is typically described in 1. Phlegmone of the lower limbs 2. Pyothorax 3. Tuberculosis 4. Chronic abscesses

4. Chronic abscesses

AL amyloidosis is frequently described in: 1. mantle cell lymphoma 2. DLBCL 3. multiple myeloma 4. lymphoplasmocytic lymphoma

3,4

Which of the following organs are the least resistant to hypoxia a. Liver b. Heart c. Kidneys d. Spleen e. Tendon

Alwys brain, but if brain is not present it's always heart!

Cancer is a malignant tumor based on: a. Connective tissue cells b. Epithelial cells c. Lymphocytes d. Mesenchyme e. Neurons

-

At autopsy the heart was bulging in a patient that died from acute myocardial infarction. There was found a blood clot that appeared macroscopically with alternating light yellow and dark red stripes. This description corresponds to - a) Mixed (coral) thrombus - b) White thrombus - c) Thrombus containing fragments of atherosclerotic plaque - d) Embolus from lower extremity veins - e) Emboli emerged in relation to trauma, accompanied by multiple bone fractures

- a) Mixed (coral) thrombus Thrombus formed by erythrocytes and fibrin Makes lines of zahn

Ten-year boy has experienced repeated episodes of swollen upper lip and abdominal pain along with a family history of angioneurotic edema. What is causing the defect states? - a) Hypersensitivity to histamine - b) C1 esterase inhibitor deficiency - c) Excessive mast cell degranulation by C5a - d) Hypersensitivity to kinin - e) Lytic membrane damage system by complement C5-9

- b) C1 esterase inhibitor deficiency Although rare, hereditary angioedema (HAE) can have catastrophic consequences. Laryngeal edema can result in asphyxiation; abdominal attacks can lead to unnecessary surgery and delay in diagnosis, as well as to narcotic dependence due to severe pain; and cutaneous attacks are disfiguring and disabling

. Which type of shock is most likely to develop several hours after severe burns? - a) Cardiogenic shock - b) Hypovolemic shock - c) Septic shock - d) Neurogenic shock - e) Anaphylactic shock

- b) Hypovolemic shock Loss of blood volume :)

Which of the following is true of bleeding? - a) Hemarthrosis of hemophiliacs is due to a congenital disorder of collagen - b) In internal bleeding iron may be reutilized - C) Hemorrhagic diathesis indicates bleeding by a congenital disorder - D) Ecchymoses indicate bleeding in the brain - E) Petechiae are extensive bleeding into skin surfaces

- b) In internal bleeding iron may be reutilized Hemophiliacs does not have a failure of collagen, it is due to failure of some of the clotting factors Hemorrhagic diathesis indicates an increased risk of bleeding, due to hypocoagulability Ecchymoses - a subcutaneous purpura (extravasation of blood) larger than 1 centimeter or a hematoma (not necesarily caused by trauma). Petechia =1-2mm in diameter hemmorhage in skin, mucous membranes, serosal surface..

What can cause a pronounced lymphedema? - a) Recurrent attacks of erysipelas - b) Infection of Wuchereria bancrofti (filiariasis) - c) Venostasis - d) Obstruction of thoracic duct - e) Heart failure

- b) Infection of Wuchereria bancrofti (filiariasis) - d) Obstruction of thoracic duct not sure though..

Fresh blood in the stool is called - a) Epistaxis - b) Hematemesis - c) Hematochezia (enteroragie) - d) Hemoptysis - e) Melena

- c) Hematochezia (enteroragie)

In which of the following places is there typically hemorrhagic infarct? - a) Kidneys - b) Liver - c) Lungs - d) Myocardium - e) Skeletal muscle

- c) Lungs (+other spongy organs robbins p 92)

What is the alternative designation of ascites? - a) Anasarka - b) Transudate - c) Hydropericardium - d) Hydroperitoneum - e) Hydrothorax

- d) Hydroperitoneum

Which of the following statements describes the state of shock best? - a) Low volume of circulating blood - b) Decrease in venous hydrostatic pressure - c) Decrease in hydrostatic blood pressure - d) Inadequate tissue perfusion - e) Low cardiac output

- d) Inadequate tissue perfusion

What is true for thrombosis? - a) Arises only in arteries and veins - b) Has only local consequences - c) Thrombus can be any object that is trapped in the blood stream - d) Thrombi always contain platelets - e) Venous thrombi only affects lower extremities

- d) Thrombi always contain platelets

Which of the following is/are NOT true for diffuse large cell lymphoma (DLBCL): 1) a tumor derived from follicular dendritic cells 2) a common tumor derived from T-cells 3) a rare tumor derived from B- cells 4) may arise on the basis of pre-existing small cell lymphoma

-DLBCL: most common type of Non-Hodgkin´s lymphoma of adults 1,2,3 are not true! 1 - NOT true, follicular dendritic cells can be found in follicular lymphomas, but as bystander cells.. The follicular lymphoma B-cells, can transform into DLBCL. 2 - NOT true. It's B-cells 3 - NOT true. But my notes say that it is the most common type of NHL in adults.. ? 4. TRUE, because it can arise from follicular lymphoma and marginal cell lymphoma, which are both medium/small sized B-cell lymphomas..

Which of the following tumor/s is/are most commonly found in bones: 1) Ewing`s sarcoma 2) Squamous cell carcinoma 3) Yolk sac tumor 4) medullary carcinoma

1) Ewing`s sarcoma Ewing's sarcoma or Ewing sarcoma is a malignant small, round, blue cell tumour. It is a rare disease in which cancer cells are found in the bone or in soft tissue. The most common areas in which it occurs are the pelvis, the femur, the humerus, the ribs and clavicle (collar bone). Endodermal sinus tumor (EST), also known as yolk sac tumor (YST), is a member of the germ cell tumor group of cancers.

Which of the following states represents purulent inflammation: a) abscesses in the liver b) portal pyemic c) thrombophlebitis of appendicular veins d) ulcerphlegmonous appendicitis

.a) abscesses in the liver b) portal pyemic c) thrombophlebitis of appendicular veins d) ulcerphlegmonous appendicitis

AL amyloid is found in: 1. multiple myeloma 2. chronic inflammation 3. tumors made of C cells 4. familial mediterranean fever

1

Select the valid statement(s) 1. multiple fibrin thrombi are formed in disseminated intravascular coagulopathy 2. amniotic fluid embolism is an example of tumor metastasis 3. gangrene of the intestine does NOT arise due to vascular closure 4. bone marrow embolism is a complication of myeloid leukemia

1

choose the statement(s) NOT belonging into the triad of virchow: 1. changes in the composition of haemotopoietic bone marrow 2. change in the coagulative qualites of blood 3. change in the qualities of the vessel wall 4. disruption in the laminar flow of the blood

1

select the valid statement(s): 1. crigler-najjar syndrom is a congenital hyperbilirubinemia 2. gilberts syndrom belongs among porphyrias 3. brown atrophy of the liver leads to hepatic failure 4. intracellular accumulations of lipofuscin is a feature of hemochromatosis

1

thrombosis: 1. may be infected and represent a source of sepsis 2. the thrombus is in every case completely dissoluted 3. the recanalisation of the thrombi occurs within 24-48 hours 4. it never develops in adequately flowing blood

1

select the valid statement(s) 1. paradoxical embolism may be succesive 2. pulmonary infarction is typically haemorrhagic 3. embolism means intravital blood clotting in blood vessels 4. thrombosis typically occurs outside the vessel lumen

1 2

Which of the following is/are NOT true for diffuse large cell lymphoma (DLBCL): 1) a tumor derived from follicular dendritic cells 2) a common tumor derived from T-cells 3) a rare tumor derived from B- cells 4) may arise on the basis of pre-existing small cell lymphoma

1 2 3 -DLBCL: most common type of Non-Hodgkin´s lymphoma of adults 1 - NOT true, follicular dendritic cells can be found in follicular lymphomas, but as bystander cells.. The follicular lymphoma B-cells, can transform into DLBCL. 2 - NOT true. It's B-cells 3 - I don't know if it's rare.. ?! But my notes say that it is the most common type of NHL in adults.. ? 4. TRUE, because it can arise from follicular lymphoma and marginal cell lymphoma, which are both medium/small sized B-cell lymphomas..

For the diagnosis of papillary thyroid carcinoma which of the following finding(s) is necessary: 1) ground-glass nuclei 2) follicles 3) metastases 4) tumor amyloid

1)

Clinical manifestations of glycogenoses include: 1) hepatomegaly 2) muscle weakness 3) mental retardation 4) episodes of hyperglycemia

1) 2)

Select the valid statement(s): 1) Fabry disease is an inborn disorder of metabolism 2) thesaurismoses are often accompanied by morphological changes in lysosomes 3) hypertriglyceridemia is an example of thesaurismosis 4) Wilson disease is sympatomatic in the first months of life

1) 2)

Select the valid statement(s): 1) viral inflammation can have serous character 2) fibrinous pericarditis occurs in uremia 3) phlegmone is an example of surface inflammation 4) pseudoabsces is suppurative interstitial inflammation

1) 2)

Which of the following process(es) can be described during the evolution of an abscess: 1) the formation of fistulas and sequestration 2) the development of an inflammatory pseudoxantoma 3) development of Ghon complex 4) development of a specific granuloma

1) 2)

Select the valid statement(s): 1) epithelioid histiocytes are found in tuberculosis 2) mycobacterial infection leads to the activation of T-lymphocytes 3) the Mantoux-test is based on immune complex-mediated reaction 4) the causative agent of TBC is demonstrated in Gram staining

1) 2) A standard dose of 5 tuberculin units is injected intradermally and read 48 to 72 hours later. A person who has been exposed to the bacteria is expected to mount an immune response in the skin containing the bacterial proteins. The reaction is read by measuring the diameter of induration (palpable raised, hardened area M. tuberculosis has an unusual, waxy coating on its cell surface (primarily due to the presence of mycolic acid), which makes the cells not able to stain by Gram staining. The Ziehl-Neelsen stain, or acid-fast stain, is used instead.

Select the valid statement(s): 1) the carbuncle belongs among staphylococcal infections 2) erysipelas belongs among streptococcal infections 3) staphylococci are pyogenic bacteria 4) streptococci are NOT pyogenic bacteria

1) 2) 3)

Steatosis caused by: 1) altered lipoprotein (VLDL) synthesis in endoplasmic reticulum manifests in hepatocytes and enterocytes 2) inherited disorders of mitochondrial fatty acids beta-oxidation might be coupled with encephalopathy and brain edema (role of toxic intermediary metabolites) 3) inherited mitochondrial beta-oxidation deficiencies manifests in organs with a high degree of mitochondrial fatty acids beta-oxidation (myocardium, skeletal muscle, hepatocytes, renal tubular epithelium) 4) excessive lipid endocytosis manifests in histiocytes with accumulation of cholesterol esters in cytosol

1) 2) 3) 4)

Cervical intraepithelial neoplasia: 1) belongs among precancerous lesions 2) is characterised by nuclear atypic in squamous epithelium 3) belongs among pseuodotumors 4) results from human papillomavirus (HPV) infection

1) 2) 4)

Identify the correct pairing: 1) chronic inflammation infiltration by macrophages 2) bacterial inflammation = infiltration by neutrophilic granulocytes 3) crupous inflammation = phlegmonous inflammation 4) xanthogranuloma = presence of foamy cells

1) 2) 4)

Lysosomal system in the cell: 1) represents a major center for regulated intracellular digestion of high molecular compounds 2) represents a center for degradation of altered poly-ubiquitylated proteins by neutral proteases 3) degrades substrates which entry via endocytosis, autophagy or a direct transport of specific molecules from cytosol 4) is not involved in regulatory mechanisms being solely a place of degradation

1) 3)

Mental retardation and skeletal deformities are typically described in: 1) some mucopolysaccharidoses 2) some glycenoses 3) Hurler syndrome 4) Gaucher disease

1) 3)

Identify the correctly paired statements: 1) chronic gastritis = lymphoplasmocellular infiltrate in the gastric mucosa 2) peptic ulcer in the stomach = liquefactive necrosis 3) catarrhal bronchopneumonia = alveoli filled with exudate and neutrophilic granulocytes 4) allergic inflammation = infiltration with numerous foamy macrophages

1) 3) peptic ulcer=fibrinoid necrosis

Choose the predisposing factor(s) of thrombosis: 1) endothelial defects of the vascular wall 2) thrombocytopenia 3) foreign body in the vascular system 4) slowing of the blood flow

1) 3) 4)

Select the correctly paired statement(s): 1) Rb1 = tumor suppressor gene 2) p53 = oncogene 3) APC = a tumor suppressor gene 4) MYC = protooncogene

1) 3) 4)

Select the valid statement(s) about empyema: 1) pyothroax is an example of empyema 2) it is bordered by suppurative interstitial inflammation 3) it can be a complication of catarrhal-purulent cholecystitis 4) pyometra is an example of empyema

1) 3) 4)

select the valid statement(s): 1) neuroendocrine tutors often detected in the appendix 2) atypical cardinoid is NOT a neuroendocrine tumor 3) cardinoid syndrome may affect the right side of the heart 4) an example of neuroendocrine tumor is a pancreatic insulinoma

1) 3) 4)

Which of the following is/are true of medullary tumors: 1) Are formed by numerous tumor cells and small number of stromal cells. 2) Are formed by in-numerous tumor cells and a large number of stromal cells. 3) They arise in the bone marrow. 4) Never metastasizes.

1) Are formed by numerous tumor cells and small number of stromal cells - Medullary carcinomas: epithelial tumor cells predominate- breast, thyroid gland (vernerova lecture) -Mostly found in breast and thyroid gland -Often metastasis -Medullary tumors are always with little or no stroma! Stromal cells: CT cells

Which of the following tumor/s is/are most commonly found in ovaries? 1) Brenner tumor. 2) Choriocarcinoma. 3) Immature teratoma. 4) Yolk sac tumors.

1) Brenner tumor. 2) Choriocarcinoma. 3) Immature teratoma. 4) Yolk sac tumors. -Brenner tumor:part of the surface epithelial-stromal tumor group of ovarian neoplasms -Choriocarcinoma: a malignant, trophoblastic cancer, usually of placenta. Also classified as a germ cell tumor and can arise in the testis or ovary. -Immature teratoma: a rare malignant (cancerous) germ cell tumor (type of tumor that begins in the cells that give rise to sperm or eggs). -Yolk sac tumors: also known as Endodermal sinus tumor (EST), a germ cell tumor group of cancers. Most common testicular tumor in children under 3 (infantile embryonal carcinoma). EST of the ovary are particularly aggressive

Which of the following is/are true for mesothelioma? 1) Can arise in connection with asbestos exposure. 2) It is a highly malignant neoplasm. 3) Usually affects the pleural surface. 4) It is a benign tumor.

1) Can arise in connection with asbestos exposure 2) It is a highly malignant neoplasm. 3) Usually affects the pleural surface. -Mesothelioma: a rare form of cancer that develops from cells of the mesothelium, the protective lining that covers many of the internal organs of the body. Mesothelioma is most commonly caused by exposure to asbestos. It says that they can be beningn as well, although it is rare...!?

Select the possible complication(s) of myocardial infarction: 1) myocardial rupture with cardiac tamponade 2) ventricular fibrillation 3) thrombosis of heart cavities 4) marantic endocarditis

1) Cardiac tamponade, also known as pericardial tamponade, is an acute type of pericardial effusion in which fluid, pus, blood, clots, or gas[1] accumulates in the pericardium (the sac in which the heart is enclosed), resulting in slow or rapid compression of the heart 2) 3)

Which of the following is/are true of amniotic fluid embolism? 1) May cause symptoms of pulmonary embolism. 2) Its often a clinically insignificant complication of childbirth. 3) Its leads to a massive increase in circulating blood volume and to circulation overload. 4) Serious complications of aminotic fluid embolism do not include DIC (disseminated intravascular coagulation) and bleeding.

1) May cause symptoms of pulmonary embolism. -A rare obstetric emergency in which amniotic fluid, fetal cells, hair, or other debris enters the mother's blood stream via the placental bed of the uterus and trigger an allergic reaction. This reaction then results in cardiorespiratory (heart and lung) collapse and coagulopathy. -DIC: a pathological process characterized by the widespread activation of the clotting cascade that results in the formation of blood clots in the small blood vessels throughout the body. This leads to compromise of tissue blood flow and can ultimately lead to multiple organ damage. In addition, as the coagulation process consumes clotting factors and platelets, normal clotting is disrupted and severe bleeding can occur from various sites. DIC does not occur by itself but only as a complicating factor from another underlying condition

Which of the following can distinguish a leiomyosarcoma from rhabdomyosarcoma? 1) Only in rhabdomyosarcoma do we find striations in the cytoplasm. 2) Leiomyosarcoma is found mostly in young ages. 3) Leiomyosarcoma is found only in the uterus, while rhabdomyosarcoma is in the heart only. 4) Rhabdymyosarcoma never forms distant metastases.

1) Only in rhabdomyosarcoma do we find striations in the cytoplasm. -Leiomyosarcoma (LMS): smooth muscle CT tumor (malignant) -Rhabdomyosarcoma (RMS): cancer cells from sk.muscle prog,. cancer of CT -RMS can metastase -LMS is found in uterus, intestines, sk.muscle etc.

Adenoma of the liver is associated with: 1) oral contraceptives 2) polycythemia vera 3) hepatatis A 4) hepatitis B

1) Oral contraceptives. -90% -Benign tumor

Which of the following is/are complications of acute myocardial infarction? 1) pericardial tamponade 2) cardiac arrhythmias 3) the emergence of an acute aneurysm of the chamber wall/s 4) the development of amyloidosis

1) Pericardial tamponade 2) Cardiac arrhythmias 3) The emergence of an acute aneurysm of the chamber wall/s -Pericardial tamponade: pericardial effusion (pus, fluid, blood clott, gas) in pericardium that compresses the heart (slow or rapid) -Amyloidosis is not acute, it takes time!

Which of the following is/are NOT true for catarrhal inflammation? 1) The accumulation of macrophages in tissue 2) Overproduction of mucus 3) mucosal oedema 4) the presence of granulocytes in the exudate

1) The accumulation of macrophages in tissue catarrhal inflammation, is a disorder of inflammation of the mucous membranes in one of the airways or cavities of the body.It can result in a thick exudate of mucus and white blood cells caused by the swelling of the mucous membranes in the head in response to an infection.

Which statement is/are true: 1) The main feature distinguishing benign and malignant tumors are differentiation and the mode of growth. 2) The main criterion for the biological behaviour of a tumor is its size. 3) Benign tumors never transforms into malignant tumors. 4) Other options are NOT true.

1) The main feature distinguishing benign and malignant tumors are differentiation and the mode of growth.

Which of the following is/are true of RS cells: 1) they are cancer diagnostic elements for hodgkin lymphomas 2) they are cancer diagnostic elements for non-Hodgkin`s lymphomas 3) they are tumor elements which are diagnostic of sarcidosis 4) they are cancer diagnostic elements in leukemia

1) They are cancer diagnostic elements for hodgkin lymphomas -RS cells: Reed-Sternberg cells (also known as lacunar histiocytes for certain types) are different giant cells found with light microscopy in biopsies from individuals with Hodgkin's lymphoma (aka Hodgkin's disease; a type of lymphoma). They are usually derived from B lymphocytes, classically considered crippled germinal center B cells, meaning they have not undergone hypermutation to express their antibody. Seen against a sea of B cells, they give the tissue a moth-eaten appearance.

Which of the following is/are true of teratomas? 1) They belong among germinal tumors. 2) They are always malignant. 3) They are always benign. 4) They arise only in the ovary or testis.

1) They belong among germinal tumors.

Which of the following is are true of pathological immune response type III reactions?: 1) a typical example of immuno-complex disease is glomerulonephritis 2) disease caused by immune complex always have a chronic course 3) this type of reaction occurs only under assumption that immune complexes circulate in the blood 4) a systemic disease of this type include hives

1) a typical example of immuno-complex disease is glomerulonephritis Type III hypersensitivity occurs when antigen-antibody complexes that are not adequately cleared by innate immune cells accumulate, giving rise to an inflammatory response and attraction of leukocytes. Hence, vasculitis, glomerulonephritis and arthritis are commonly associated conditions as a result of type III hypersensitivity responses

Which of the following brain tumors is/are most common in adults: 1) astrocytoma 2) oligodendroglioma 3) medulloblastoma 4) glioblastoma

1) astrocytoma remember that medulloblastoma are most common in children!

Which of the following is/are of septic shock 1) can be accompanied by Waterhouse Friderichsen syndrome 2) Other options are NOT true 3) Never accompanies suppurative leptomeningitis 4) is associated with staphylococcal infections

1) can be accompanied by Waterhouse Friderichsen syndrome Waterhouse-Friderichsen syndrome (WFS) or hemorrhagic adrenalitis or Fulminant meningococcemia, is defined as adrenal gland failure due to bleeding into the adrenal glands, caused by severe bacterial (or rarely viral) infection (most commonly the meningococcus Neisseria meningitidis)

In which of these infections do not involve invasion of the causative agent into host tissues? 1) cholera 2) skarlatina 3) typhus exanthematicus 4) typhus abdominalis

1) cholera cholera: Transmission occurs primarily by drinking water or eating food that has been contaminated by the feces (waste product) of an infected person, including one with no apparent symptoms.

Which of the following is/are true for neuroblastoma 1) it is a tumor formed of small blue cells 2) its presence in retroperitoneum is rare 3) never metastasize 4) its occurrence in childhood is rare

1) it is a tumor formed of small blue cells most common extracranial solid cancer in childhood and the most common cancer in infancy. It most frequently originates in one of the adrenal glands, but can also develop in nerve tissues in the neck, chest, abdomen, or pelvis.

which of the following is true of gangrene 1) wet gangrene is caused by putrefactive bacteria 2) never occurs in children 3) it not associated with necrosis 4) never affects internal organs

1) wet gangrene is caused by putrefactive bacteria the act or process of putrefying; the anaerobic decomposition of organic matter by bacteria and fungi that results in obnoxiously odorous products; rotting. Dry gangrene is actually a form of coagulative necrosis

which of the following diseases is an immunodeficiency? 1. bruton agammaglobulinemia 2. sjøgrens syndrom 3. goodpastures syndrom 4. wiskott aldrish syndrom

1, 4

Paradoxial embolism 1. may cause myocardial infarction 2. results from pulmonary embolism and patent foramen ovale 3. results from pulmonary embolism and closed foramen ovale 4. is venous thrombus embolization into the hepatic veins due to increased intrathoracic pressure

1,2

in sarcoidosis we microscopically describe 1. asteroid inclusions 2. epitheloid granulomas 3. fungal microorganisms 4. foci of caseous necrosis

1,2

in the follicular carcinoma of the thyroid we typically describe: 1. angioinvasion 2. capsular invasion 3. clearing of the nuclear chromatin 4. psammomatous bodies

1,2

steatosis might be: 1. caused by an altered lipoprotein synthesis in endoplasmatic reticulum of hepatocytes 2. caused by mitochondrial fatty acids beta-oxidation disorders 3. severe in brain neurons causing sudden infant death syndrome (SIDS) 4. a manifestation of a genetic defect

1,2

the klatskin tumor is: 1. a cause of painless obstructive icterus 2. a malignant tumor arising from the extrahepatic bile ducts in the proximity of the hepatic hilus 3. a malignant tumor of the renal pelvis 4. a malignant hepatic tumor

1,2

which of the following is an example for steatosis: 1) nutmeg liver 2) myocardial changes in chronic hypoxia 3) proliferation of fatty tissue in lipomatosis 4) cholesterolosis of the gallbladder

1,2

Addison's disease: 1. is associated with the overproduction of ACTH 2. is associated with the production melanotropin 3. its symptom is patchy hypopigmentation 4. occurs due to insuficiency of the thyroid

1,2 Addisons, decreased cortisol--> increaced POMC,--> increased melanotropin, ACTH and endorphins, ie. hyperpigmentation. Occurs due to adrenal glands do not produce sufficient steroid hormones (like f.eks cortisol)

The possible outcome(s) during the evolution of a venous thrombus include: 1) propagation 2) embolisation 3.organization 4. recanalization

1,2, 3, 4

Choose the disease(s) caused by immunocomplexes: 1. post-streptococcal glomerulo nephritis 2. serum sickness 3. SLE systemic lupus erythematodes 4. Graves disease

1,2,3

choose the correct statement(s) for phlegmon 1. it is frequently observed in the lower limbs of diabetic patients 2. an example is erysipelas 3. the infection spreads diffusely 4. the infectious process is well circumscribed

1,2,3

choose the diseases with an increased risk of developing a malignant tumor. 1. celiac disease 2. chronic atrophic gastritis 3. ulcerative colitis 4. hepatitis A infection

1,2,3

choose the option(s) where a purulent iflammation is present: 1. hepatic abcess 2. acute pyelonefritis 3. bacterial meningitis 4. catarrhal appendicitis

1,2,3

choose the valid statement(s) about adenomyomatous hyperplasia of the prostate: 1. it is not a precancerosis 2. it is associated with nycturia and dysuria 3. it can be a cause of inflammation of the urinary tract 4. it is prevalent among young adults

1,2,3

reparation is: 1. the replacement of the original tissue with functionally an morphologically inferior tissue 2. a healing process, during which scar tissue is formed 3. described during the healing of an ulcer 4. reparation does not occur in bones

1,2,3

ascitic fluid in the peritoneal cavity can be sign of: 1. right sided heart failure 2. infectious peritonitis 3. portal hypertension 4. carcinomatous peritonitis

1,2,3,4

the histological structure of an epithelial tumor may be: 1. solid 2. glandular 3. trabecular 4. solid alveolar

1,2,3,4

HPV infection does not cause: 1. condylomata lata 2. ulnar epicondylitis 3. condylomata accuminata 4. high-grade bronchial intraepithelial neoplasia

1,2,4

streptococcal angina is NOT characterized by: 1. koplik spots 2. skin rash 3. swelling of the tonsils 4. pauci-immune glomerulonephritis

1,2,4

Select the correct paired statement(s) 1.hemosiderosis= hemosiderin accumulation in macropahges due to haemolysis 2. hemosiderin= identical chemical composistion as bilirubin 3. hemosiderin= predominately intracellular accumulation 4. hemosiderin= identical chemical composition as ceroid

1,3

choose the cell(s) NOT belonging into the inflammatory infiltrate of ieprosy: 1. touton multinuclear giant cells 2. virchow cells 3. mikulicz cells 4. lymphocytes and plasma cells

1,3

choose the correct statement(s) about miliary tuberculosis 1. it spreads hematogenously 2. it affects mostly the cervical lymph nodes 3. a great number of tiny caseous granulomas can be observed microscopically 4. it is described mostly in secondary TBC

1,3

choose the risk factors for artherosclerosis: 1. cigarette smoking 2. hypotension 3. hypercholesterolemia 4. cardiovascular disease in the family history

1,3

select the histologic changes characteristic for acute pancreaitis 1. massive destruction of the acinal secretory parenchyma 2. dilated pancreatic are filled with calcified concretions 3. balzer necroses of the surrounding fatty tissue 4. diffuse fatty atrophy of the secrtory acinar parenchyma

1,3

choose the valid statement(s) about left ventricular hypertrophy: 1. no proliferation of cardiomyocytes is evident 2. the hyperthrophic left ventricle is 11 mm thick or more 3. an important parameter for its diagnosis is the weight of the heart 4. it is always irreversible

1,3 2 was marked as wrong, but the regular thickness of the left ventricle is 0.6-1.1, and the statement should therefore be correct

choose the valid statement(s) about Bence-Johnson protein: 1. accumulate in the renal tubules 2. consist of heavy immunoglobulin chains, mostly of the IgD class 3. is produced by multiple myeloma 4. consist of Tamm-Horsfall protein

1,3 (Bence Johnson protein is free light chains circulating in urine and deposits in renal tubules, leads to risk of renal failure(myeloma kidney) ) Bence Johnson does not consist of Tamm Horsfall.... TH is a protein that can be present in multiple myeloma... its a glycoprotein, the most abundant protein excreted in ordinary urine

alpha-1 antitrypsin deficiency is characterized by: 1. panlobular emphysema 2. centrilobular emphysema 3. hepatal failure with cirrhosis 4. chronic lung insufficiency

1,3,4

choose the INvalid statement(s) about lyme: 1. it is caused by rickettsia 2. neurologic symptoms can be present 3. the typical dermal lesion is called erythema nodosum 4. its vector is the body louse

1,3,4

choose the valid statement(s) about toxoplasmosis: 1. it is protozoal infection 2. transplacental spread is not possible 3. it may cause microphtalmia 4. it is zoonosis

1,3,4

specific granulation tissue is decribed in: 1. leprosy 2. a bony callus 3. syphilis 4. pott abcess

1,3,4

where can be renal amyloid deposition present? 1. in the wall of small vessels 2. in the lumina of tubules 3. in the glomerular mesangium 4. in the interstitium

1,3,4

ulcerous colitis: 1. may be accompanied by erythema nodosum 2. the infection of the bowel wall is transmural 3. it may affect the appendix 4. it is an infection with mucosal pseudopolyp formation

1,3,4 (Erythema nodosum (EN) is an inflammatory condition characterised by inflammation of the fat cells under the skin, resulting in tender red nodules or lumps that are usually seen on both shins)

in basal cell carcinoma of the skin we typically describe: 1.local recurrences 2. metastasis and early invation in lymphatic vessels 3. peripheral palisading of the tumor cells 4. ulceration of the tumor surface

1,3,4 (in weathers its states that recurrence is rare... from 1-50%, in robbins it says 40% recurrence, it also says that it MAY ulcerate.. so the term typically basically fuges us over)

Masson fontana stain 1. belongs to the impregnation teqniques 2. can't be used on parafin embedded stainings 3. stains amyloid while exhibiting dichroism in polarized light 4. is used for detection of melanin

1,4

amyloid deposition can be verified with: 1. immunohistochemistry/immunoflourescence 2. hematoxylin-eosin 3. yellow birefringence after cresyl violet staining 4. green birefringence after congo red staining

1,4

choose the correct statement(s) about the nuclei of papillary thyroid carcinoma: 1. they are optically clear, with nuclear membrane grooves 2. bizarre nuclear atypias are typically observed 3. they almost completely resemble the nuclei of non-neoplastic thyreocytes 4. we frequently describe nuclear inclusions

1,4

choose the correct statement(s) for primary syphilis: 1. in the affected tissues a lymphoplasmocytic infiltrate is found 2. treponemes are not microscopically and serologically detectable anymore 3. the infection is generelized 4. the capillaries are affected by the inflammation (endarteritis)

1,4

choose the corrects statement(s) about mucosal inflammation: 1. a catarrhal type of infection is usually described 2. mucosal inflammation always transitions into chronic inflammation 3. abcesses are usually described 4. chronic inflammation can lead to mucosal hyperplasia

1,4

choose the tumor(s) belonging into the family of neuroectodermal tumors: 1. malignant melanoma 2. ependymoma 3. leiomyosarcoma 4. pheochromocytoma

1,4

epidural hematoma is caused by: 1. the rupture of the middle meningeal artery 2. damage to the bridging veins 3. rupture of the lenticostriate artery 4. cranial trauma followed by a loss of conciousness

1,4

Select the valid statement(s) about myeloid leukemias: 1. they are clonal disorders of hematopoetic stemcells 2. swollen lymphnodes are typical, spleen and liver are normal size 3. hand-schuller christian disease is an example 4. they often arise in the background of of myelodysplastic syndrom

1,4 (hand schuller christian disease is a langerhans cell histiocytosis)

Cholelithiasis 1. A risk factor is hypercholesterolemia 2. A risk is sialolithiasis 3. Possible complications include stenosis of the common bile duct 4. Can lead to biliary colic

1. A risk factor is hypercholesterolemia 3. Possible complications include stenosis of the common bile duct 4. Can lead to biliary colic

Histologically the following is/are described in the Aschoff's nodule in rheumatic myocarditis 1. Aschoff's cells 2. Central caseus necrosis 3. Schaumann's giant cells with astroid-bodies 4. Central fibroid necrosis

1. Aschoff's cells 4. Central fibroid necrosis

Histolocally the followingfinding(s) is (are) descibed in the Aschoff's nodule in rheumatic myocarditis: 1. Aschoff's cells, fibrunoid necrosis 2. colones of Gram+ streptococci 3. Schaumann's giant cells with asteroid-bodies 4. Anitschkow cells

1. Aschoff's cells, fibrunoid necrosis 4. Anitschkow cells

Which statement is/are true of pulmunary embolism 1. Can be successive 2. Develops typically in the superficial veins of the upper extremity 3. Is a complication of prolonged immobilisation 4. Pulmunary infarction always develops

1. Can be successive 3. Is a complication of prolonged immobilisation

Meningioma 1. Can behave in a malignant fashion 2. Typically contains areas of necrosis and haemorrhage 3. Arises exclusively intracranially 4. Is more frequently found in women

1. Can behave in a malignant fashion 4. Is more frequently found in women

Listeriosis 1. Can spread trough contaminated food 2. Can lead to spontaneus abortion 3. Can cause meningitis in weakened individuals

1. Can spread trough contaminated food 2. Can lead to spontaneus abortion 3. Can cause meningitis in weakened individuals

which statement(s) do not belong to the whole-body manifestations of inflammation? 1. leukocytosis 2. decrease in the level of plasmatic cytokines 3. fever 4. decreased erythrocyte sedimentation rate

2,4

Which of the following is/are NOT morphological patterns seen in early congenital syphilis: 1. Gumma. 2. Osteochondritis. 3. Hudchinsonova triad. 4. Pneumonia alba.

1. Gumma. -Gumma: a soft, non-cancerous growth resulting from the tertiary stage of syphilis. It is a form of granuloma. -Osteochondritis:a painful type of osteochondrosis where the cartilage or bone in a joint is inflamed -Hudchinsonova triad: a common pattern of presentation for congenital syphilis. Consists of three phenomena: interstitial keratitis (inflamed eye), Hutchinson incisors (smaller and widely spread incisors), and eighth nerve deafness. -Pneumonia alba: often seen in neonates with congenital syphilis. The lung may be firm and pale, owing to the presence of inflammatory cells and fibrosis in the alveolar septa.

Urolithiasis can manifest with 1. Haematuria 2. Urinary tract inflammation 3. Hydronephrosis 4. Adenomyomatous hyperplastic reaction of the prostate

1. Haematuria 2. Urinary tract inflammation 3. Hydronephrosis

Which of the following are not examples of acquired immunosuppression 1. Hirshprung disease

1. Hirshprung disease

Dystrophic calcification is typically described 1. In the leaflets of a stenotic aortic valve 2. In the wall of a postencephalomalatic pseudocyst 3. In the kidneys in nephrocalcinosis 4. In atherosclerotic plaques

1. In the leaflets of a stenotic aortic valve 4. In atherosclerotic plaques

Benign neoplasms 1. In the uterus they are more numerous than malignant neoplasms 2. They usually have an expansive growth pattern 3. Histologically we usually see necrosis and an incresed mitotic rate 4. Melanoma of the eye is an example

1. In the uterus they are more numerous than malignant neoplasms 2. They usually have an expansive growth pattern

Condyloma accuminatum 1. Is a genital wart 2. It develops n treponema pallidum infection 3. It develops in HSV type 2 infection 4. It develops in HPV infection

1. Is a genital wart 4. It develops in HPV infection

The inflammatory pseudomembrane in the course of pseudomembranous inflammation 1. Is composed of fibrin and necrotic tissue 2. It can occur in the large intestine after treatment with antibiotics 3. The inflammatory infiltrate is composed mainly of neutrophils 4. After the removal of the pseudomembrane we observe a bleeding mucosal defect in its place

1. Is composed of fibrin and necrotic tissue 2. It can occur in the large intestine after treatment with antibiotics 3. The inflammatory infiltrate is composed mainly of neutrophils 4. After the removal of the pseudomembrane we observe a bleeding mucosal defect in its place

Dystrophic calcification 1. Is described in degenerated heart valves 2. Is described in atherosclerosis 3. Is dependent on blood calcium levels 4. It typically affects veins

1. Is described in degenerated heart valves 2. Is described in atherosclerosis

Choose the statement valid for the sentinel lymph node 1. Is examined in malignant melanoma 2. The clinician usually identifies it though palpation 3. It is examined in brest carcinoma 4. It is examined in adenocarcinomas of the pancreas

1. Is examined in malignant melanoma 3. It is examined in brest carcinoma

Non-specific granulation tissue 1. Is present in a healing gastric ulcer 2. S present in cernification of the lung 3. Is made up of hypocellular hyalinized collagen tissue 4. Is made up of fibroblasts and capillaries

1. Is present in a healing gastric ulcer 2. S present in cernification of the lung 4. Is made up of fibroblasts and capillaries

Renal infarction 1. Is wedge-shaped 2. It heals with a scar 3. Liquefactive necrosis is described 4. Coagulative necrosis is described

1. Is wedge-shaped 2. It heals with a scar 4. Coagulative necrosis is described

Select the valid statement about cystic fibrosis? 1. It affects bronchial glands 2. Can lead to malabsorption 3. It typically manifests in adulthood 4. The inheritance is autosomally recessive

1. It affects bronchial glands 2. Can lead to malabsorption 4. The inheritance is autosomally recessive

Hyperplasia 1. It affects the prostate of older men 2. Is the opposite of aplsia 3. It is irreversible 4. In the hyperplastic cells significant nuclear atypia is present

1. It affects the prostate of older men

Hypovolemic shock 1. It can be caused by severe dehydration in cholera patients 2. Is present in sepsis 3. Is caused by anaphylaxis 4. Severe bleeding is present in every case

1. It can be caused by severe dehydration in cholera patients

Which statement is /are true Kruckenberg's tumor of the ovary 1. It has poor prognosis 2. Is a primary tumor of the ovary 3. Is usually bilateral 4. Frequently is a metastasis of gastric neuroendocrine tumors

1. It has poor prognosis 3. Is usually bilateral

Which of the following statement is/are true of choriocarcinoma 1. It is a highly malignant tumor 2. It is made up of small uniform tumor cells 3. It is described in the uterus 4. It typically produces carcinoembryonal antigene (CEA)

1. It is a highly malignant tumor 3. It is described in the uterus

Dysplasia 1. It is described in HPV- related lesions 2. In all causes it progresses to malignant tumors 3. It does not develop in cylindric epithelium 4. In dysplastic cells gradual accumulation of gene mutations is described

1. It is described in HPV- related lesions 4. In dysplastic cells gradual accumulation of gene mutations is described

Choose the correct statement for phlegmone 1. It is frequently abserved in the lower limbs of diabetic patients 2. Is usually caused by S aurus 3. The inflammation spreads diffusely 4. The affected tissues are hyperemic

1. It is frequently abserved in the lower limbs of diabetic patients 3. The inflammation spreads diffusely 4. The affected tissues are hyperemic

Which statement is/are true of thrombosis 1. It is itravital blood clot formation in the cardiovascular system 2. During organization a thrombus can undergo calcification 3. The predisposing factors are described ij the so-called Virchow's triad 4. It does not occur within the heart

1. It is itravital blood clot formation in the cardiovascular system 2. During organization a thrombus can undergo calcification 3. The predisposing factors are described ij the so-called Virchow's triad

Which of the following is/are true of Barrett's esophagus 1. It is the precancerouscondition of adenocarcinoma of the esophagus 2. Microscopically we observe squamous metaplasia of the esophageal epithelium 3. It is the precancerous lesion of squamous cell carcinoma of the esophagus 4. It is usually described in the area of the gastroesophageal junction

1. It is the precancerouscondition of adenocarcinoma of the esophagus 4. It is usually described in the area of the gastroesophageal junction

KRAS 1. Its mutation is frequently described in adenocarcinoma of the colon 2. Is a protooncogene 3. Is a tumor supressor gene 4. Its germinal mutation causes fimilar adenomatous polyposis

1. Its mutation is frequently described in adenocarcinoma of the colon 2. Is a protooncogene

Which of the following is caused by a meningococcal infection 1. Leptomenyngitis 2. AL amyloidosis 3. Waldestrom macroglobulinemia 4. Rheumatic fever

1. Leptomenyngitis

PAS positivity is described in 1. Lipofuscin 2. Cytoplasm of clear cell renal carcinoma 3. Hemosiderin 4. Goblet cells

1. Lipofuscin 2. Cytoplasm of clear cell renal carcinoma 4. Goblet cells

Which of the following pair is /are correct? 1. Lymphocytes - chronic inflammation 2. Past bleeding - haemosiderin 3. Neutrofils - purulent infammation 4. Osinophils - parasitic infection

1. Lymphocytes - chronic inflammation 2. Past bleeding - haemosiderin 3. Neutrofils - purulent infammation 4. Osinophils - parasitic infection

The complex of rheumatic fever includes 1. Myocarditis 2. Mitral valve stenosis 3. Aneurism of the circle of Willis 4. Huntington's chorea

1. Myocarditis 2. Mitral valve stenosis

Peritoneal metastatic implant formation is typical of 1. Ovarian carcinoma 2. Thyroid carcinoma 3. Germinal tumors 4. Tumors of the gastrointestinal tract perforating to the serosal surface

1. Ovarian carcinoma 4. Tumors of the gastrointestinal tract perforating to the serosal surface

Paradoxical embolism 1. Patent foramen ovale is its basis 2. Is directed from the right to the left side of the heart 3. Develops in bacterial endocarditis 4. Develops in patients with hypotension

1. Patent foramen ovale is its basis 2. Is directed from the right to the left side of the heart

Choose the disease caused by immunocomplexes 1. Post-streptococal glomerulonephritis 2. Autoimmune haemolytic anemia 3. Systemic lupus erythematodes 4. Ulcerous colitis

1. Post-streptococal glomerulonephritis 3. Systemic lupus erythematodes

Granulomas are a typical morphologic manifestation of 1. Sarcoidosis 2. Tuberculosis 3. Ulcerous colitis 4. Anthracosis

1. Sarcoidosis 2. Tuberculosis

Which of e following statements rapresents an enthiopathogenic connection? 1. Silicosis causes cor pulmonale chronicum

1. Silicosis causes cor pulmonale chronicum

Neuroendocrine tumors 1. Small cell carcinoma is an example 2. Can cause episodes of hypoglycemia 3. They are benign tumors 4. They are most frequently found in the skin

1. Small cell carcinoma is an example 2. Can cause episodes of hypoglycemia

Metastatic calcification typically develops in 1. The intestitium of the lung 2. Cardiomyyocites 3. Kidneys 4. Brain tissue

1. The intestitium of the lung 3. Kidneys

Choose the correct stetment about Balser necrosis 1. The necrotic areas undergo saponification due to precipitation of plasmatic calcium 2. The unleashed pancreatic enzymes digest the pancreatic tissue 3. It accompanies celiac spue 4. It can affect peripancreatic adipous tissues

1. The necrotic areas undergo saponification due to precipitation of plasmatic calcium 2. The unleashed pancreatic enzymes digest the pancreatic tissue 4. It can affect peripancreatic adipous tissues

Choose the statement valid for a patient with septic shock 1. The pulp of the spleen is activated 2. It may be the cause of acute kidney failure 3. The causative infectious agent is typically a virus 4. The patient has tachycardia an tachypnoe

1. The pulp of the spleen is activated 2. It may be the cause of acute kidney failure 4. The patient has tachycardia an tachypnoe

Ulcerous colitis 1. Toxic megacolon is a potentially lethal complication 2. The infection of the bowel wall is transmutral 3. It affects also the rectum 4. Is an inflammatory process with gastric polyp formation

1. Toxic megacolon is a potentially lethal complication 3. It affects also the rectum

Chron's disease 1. Typically leads to stenosis of the intestinal lumen 2. Can cause fistula formation 3. The most affected segment of te intestines in the terminal ileum 4. The large intestine is affected uniformly in its entire length

1. Typically leads to stenosis of the intestinal lumen 2. Can cause fistula formation 3. The most affected segment of te intestines in the terminal ileum

Which of the following is/are true of malignant mesothelioma 1. Usually affects the pleural surface 2. It is usually well treatable 3. It is non-aggressive tumor 4. Atises in connection with tobacco smoke exposure

1. Usually affects the pleural surface

Arachidonic acid: 1. belongs among the precursors of infiammator mediators 2. it is present in mast cells 3. it is activated by specific immunity 4. its metabilism is inhibited by corticosteroids

1. belongs among the precursors of infiammator mediators 2. it is present in mast cells 4. its metabilism is inhibited by corticosteroids

Select the valid statement(s) 1. cellular swelling precede necrosis 2. characteristic geature of necrosis is te fragmentatio of nuclei and cell shrinkage (bleeding) 3. necrosis is characterized by the residuction of cytoplasmic cosinophilia 4. cause of necrosis may be depletion of ATP stores

1. cellular swelling precede necrosis 4. cause of necrosis may be depletion of ATP stores

We describe microscopically in tuberculosis the following: 1) Langerhans cells 2) basophilic cellular debris in central caseous necrosis 3) acidoresistant (acid-fast) cocci 4) inclusions in Orth cells

2) (find source) Langerhans cells must not be confused with langhans cells:)....... Mycobacteria tuberculosis does stain with acid fast staining.... Does that mean they are acidoresistand or not.? What is a orth cell?

Select the valid statement(s): 1) melanocytic nevus is a malignant skin tumor 2) the poorest prognosis among the variants of malignant melanoma is ascribed to the nodular type 3) the basal cell carcinoma is a benign tumor 4) malignant melanoma belongs to the neuroectodermal tumors

2) 4)

Which of the following is/are true for repair and regeneration: 1) a typical example of regeneration is the formation of nodes in a cirrhotic liver 2) a primitive bony callus is an example of a reparative process 3) a postencephalomalatic pseudocyst wall is formed by connective tissue as a result of healing of encephalomalacy 4) organization and recanalization of a thrombus is one of the regenerative processes

2) A primitive bony callus is an example of a reparative process. -Encephalopathy: softening of brain tissue 1 - scarring ? reparation maybe 3 - no connective tissue in brain, only gliosis 4 - reparative or organization..

Which of the following is/are true for anaplastic large cell lymphoma (AICI): 1) Contains diagnostic "popcorn" cells. 2) A tumor formed of T cells. 3) A tumor formed of B cells. 4) Creates a follicular pattern.

2) A tumor formed of T cells. -ALCL: a type of non-Hodgkin lymphoma involving aberrant (unormal) T-cells -Popcorn cells: seen in Hodgkin´s lymphoma

Which of the following is/are NOT true of mantle cell lymphoma (MCL): 1) a tumor formed of small B-cells 2) a tumor formed of large B-cells 3) a tumor formed of large T-cells 4) a tumor formed of small T-cells

2) A tumor formed of large B-cells 3) A tumor formed of large T-cells 4) A tumor formed of small T-cells -MCL: Non-Hodgkin´s lymphomas, caused by B-cell malignancy. Cancer of lymph. system

EBV plays a role in pathogenesis of: 1) malignant mesothelioma 2) Burkitt`s lymphoma 3) Cancer of the urinary bladder 4) Chronic lymphocytic leukemia

2) Burkitt`s lymphoma The Epstein-Barr virus (EBV), also called human herpesvirus 4 (HHV-4), is a virus of the herpes family, and is one of the most common viruses in humans. It is best known as the cause of infectious mononucleosis (glandular fever). It is also associated with particular forms of cancer, such as Hodgkin's lymphoma, Burkitt's lymphoma, nasopharyngeal carcinoma, and conditions associated with human immunodeficiency virus (HIV), such as hairy leukoplakia and central nervous system lymphomas. There is evidence that infection with the virus is associated with a higher risk of certain autoimmune diseases,[3] especially dermatomyositis, systemic lupus erythematosus, rheumatoid arthritis, Sjögren's syndrome, and multiple sclerosis. Infection with EBV occurs by the oral transfer of saliva and genital secretions.

Which of the following is/are characteristic for types of inflammation seen in influenza in the upper airways: 1) Proliferative. 2) Catarrhal hemorrhagic 3) Granulomatous. 4) Septic.

2) Catarrhal hemorrhagic -Proliferative: new CT fibres, also called productive -Catarrhal hemorrhagic: normal in resp.tract and colon due to goblet cells -Granulomatous: often chronic inflammation -Septic: due to poison, e.g bacterial product

Acne rosacea may be associated with: 1) measles 2) demodex follicolorum 3) chickenpox 4) smallpox

2) Demodex follicolorum. -Acne rosacea: pink/red "rash" in face -Demodex follicolorum: a face mite (parasites) of hair follicles. Demodex brcuis in seb.glands

Which of the following is/are NOT true for classical Hodgkin´s lymphoma: 1) a common tumor 2) diagnostic koilocytes are present 3) its formation may be associated with EBV infection 4) a typical tumor of adolescents and young adults

2) Diagnostic koilocytes are present ( I would say 1. is not true, cause it represents less than 1% of cancers, 2.not true HPV associated. 3. true, there is some association with EBV. 4. not true, again because it is not typical.... but this can be discussed, cause it does occur in young adults) -Hodgkin´s lymphoma: originates from WBC lymphomas -Kiliocytes: changed sq.epith.cell, found when infection of cell by papilloma virus -Epstein-Barr virus infection -The diagnosis of Hodgkin's disease is confirmed by visualizing tissue samples using a microscope. When a biopsy from the cancer contains a certain type of cell termed a Reed-Sternberg cell, the lymphoma is classified as Hodgkin's disease (DOES ANYONE HAVE A SOURCE THAT SAYS ITS associated with EBV???)

Which of the following changes is/are characteristic for rheumatoid nodule: 1) numerous giant cells of foreign body-type with crystals 2) fibrinoid necrosis with histocytic rim 3) septic necrosis with granulocytes 4) caseous necrosis with histocytic rim

2) Fibrinoid necrosis with histocytic rim. -Rheumatoid nodule: local swelling or tissue lump often seen in rheumatoid arthritis -Histocytic rim: consists of a shell of fibrous tissue

An 18 year old healthy man with random identified slightly elevated unconjugated hyperbilirubinaemia, with virtually normal levels of ALT and AST. What is the most likely diagnosis? 1) Crigler-Najjar syndrome 2) Gilbert´s syndrome 3) Rotor syndrome 4) Dubin-Johnson syndrome

2) Gilbert´s syndrome -Crigler-Najjar syndrome: or CNS is a rare disorder affecting the metabolism of bilirubin, a chemical formed from the breakdown of the heme in red blood cells. The disorder results in an inherited form of non-hemolytic jaundice, which results in high levels of unconjugated bilirubin and often leads to brain damage in infants -Gilbert´s syndrome: a genetic liver disorder and the most common hereditary cause of increased bilirubin -Rotor syndrome and Dubin-Johnson: a rare, relatively benign autosomal recessive bilirubin disorder. It is a distinct, yet similar disorder to Dubin-Johnson syndrome — both diseases cause an increase in conjugated bilirubin

Which of the following is/are autoimmune diseases? 1) AIDS 2) Hashimoto thyroiditis. 3) asthma bronchiale 4) histoplasmosis

2) Hashimoto thyroiditis. Human immunodeficiency virus infection and acquired immune deficiency syndrome (HIV/AIDS) is a disease spectrum of the human immune system caused by infection with human immunodeficiency virus. Histoplasmosis (also known as "Cave disease," is a disease caused by the fungus Histoplasma capsulatum. Symptoms of this infection vary greatly, but the disease affects primarily the lungs. Occasionally, other organs are affected; this is called disseminated histoplasmosis, and it can be fatal if left untreated.

Which of the following are true for adenomiomatpus hyperplasia of prostate A. It is precancerous condition B. May be a cause of phylonephritis

B. May be a cause of phylonephritis

Chronic peptic ulcer of the duodenum is probably associated with: 1) Hiatus hernia 2) Helicobacter pylori infection 3) Intestinal metaplasia of squamous epithelium in the lower third of the esophagus 4) High risk of developing adenocarcinoma in the small intestines

2) Helicobacter pylori infection -Helicobacter pylori infection: a spiral-shaped bacterium commonly found in the stomach. The bacteria's shape and the way they move allow them to penetrate the stomach's protective mucous lining, where they produce substances that weaken the lining and make the stomach more susceptible to damage from gastric acids. -Hiatus hernia:is the protrusion (or herniation) of the upper part of the stomach into the thorax through a tear or weakness in the diaphragm

Which of the following is/are true of paraneoplastic syndrome? 1) Manifested only in veins. 2) Is often found in small cell lung cancer. 3) Is always caused by hormonal depletion. 4) Is only associated with squamous cell carcinoma.

2) Is often found in small cell lung cancer. -PS is a consequence of cancer, but, unlike mass effect it is not due to the local presence of cancer cells -Mediated by humoral factors (by hormones or cytokines) -Most commonly present with cancers of the lung, breast, ovaries or lymphatic system (a lymphoma

Ghons complex is a term that refers to: 1) The generalization in resistance to therapy. 2) Primary complex. 3) Late generalization. 4) Early generalization.

2) Primary complex. -Gohns complex: a lesion seen in the lung that is caused by tuberculosis. The lesions consist of a calcified focus of infection and an associated lymph node. -Primary complex: combination of a Ghon focus and a corresponding lymph node focus in primary tuberculosis.

Pulmonary alveoli filled with fibrin rich exudate and granulocytes preferably corresponds to which of the following stages of lobar pneumonia?: 1) lysis 2) Red hapatisation 3) carnification 4) inflammatory oedema

2) Red hapatisation Hepatization is conversion into a substance resembling the liver; a state of the lungs when gorged with effused matter, so that they are no longer pervious to the air. Red hepatization is when there are red blood cells, neutrophils, and fibrin in the alveoli; it precedes gray hepatization, where the red cells have been broken down leaving a fibrinosuppurative exudate. The main cause is lobar pneumonia

Which of the following is/are true of metaplasia: 1) it is intravital blood clotting in veins 2) a transformation of columnar epithelium of the uterine cervix into squamous epithelium 3) formation of secondary deposits at a greater or lesser distance from the primary site of the disease 4) it is the movement of an object through the blood stream to a place where anatomical narrowing of blood vessels prevents further movement

2) a transformation of columnar epithelium of the uterine cervix into squamous epithelium

Which statement describes the correct sequence of events: 1) active hyperemia, transudation, passive hyperemia 2) active hyperemia, passive hyperemia, exudation 3) passive hyperemia, exudation, active hyperemia 4) cell infiltration, passive hyperemia, inflammatory edema

2) active hyperemia, passive hyperemia, exudation Hyperaemia or hyperemia is the increase of blood flow to different tissues in the body.

Which of the following is/are true for autolysis? 1) never occurs intra vitam 2) can be associated with necrosis 3) other opinions are NOT true. 4) it is only a posthumous phenomen

2) can be associated with necrosis autolysis, more commonly known as self-digestion, refers to the destruction of a cell through the action of its own enzymes. Posthumous means arising, occurring, or continuing after one's death.

Which of the following statement/s is/are true for CLL: 1)has no relationship to small B-cell lymphoma 2) can form liver infiltrates 3) affects mainly adolescents 4) other options are NOT true

2) can form liver infiltrates B-cell chronic lymphocytic leukemia (B-CLL), is the most common type of leukemia in adults. CLL is a disease of adults, but, in rare cases, it can occur in teenagers=adolescents and occasionally in children

Wich of the following statement/s is/are true for CLL (chronic lymphocytic leukemia): 1)has no relationship to small B-cell lymphoma 2) can form liver infiltrates 3) affects mainly adolescents 4) other options are NOT true

2) can form liver infiltrates B-cell chronic lymphocytic leukemia (B-CLL), is the most common type of leukemia in adults. CLL is a disease of adults, but, in rare cases, it can occur in teenagers=adolescents and occasionally in children infiltration to bone marrow spleen lymph nodes and liver is almost always present

which of the following best characterizes inflammatory forms of bronchopneumonia: 1) exudative-serous 2) exudative purulent 3) serous 4) granulomatous

2) exudative purulent

Which of the following is/are true of Atrophy 1) Is always a pathological process 2) numerical atrophy occurs at a reduced cell production or destruction 3) Simple and numeric atrophy cannot be combined 4) involution is the only hormone caused atrophy

2) numerical atrophy occurs at a reduced cell production or destruction can be both physiological (thymus) and pathological.

Which of the following is/are characteristic for chronic right heart failure 1) peripheral edema 2) pulmonary edema 3) nutmeg liver 4) Splenomegaly.

2) pulmonary edema The question must either be regarding left sided heart failure, or the question must be what characteristics are NOT associated with right sided heart failure

A typical complication of an aneurysm rupture of blood vessels of the willis circle is: 1) epidural hemorrhage 2) subarachnoidal bleeding 3) subdural hemorrhage 4) bleeding into the periventricular white matter

2) subarachnoidal bleeding

Which of the following is/are true for adenocarcinomas: 1) adenocarcinomas arise mainly from surface epithelium 2) they include signet ring cell carcinomas 3) gelatinous carcinoma is NOT characterized by a significant production of mucous 4) they never arise through malignant transformation of adenomas

2) they include signet ring cell carcinomas Epithelial tissue sometimes includes, but is not limited to, the surface layer of skin, glands, and a variety of other tissue that lines the cavities and organs of the body.

Which of the following is/are true of inflammatory cell infiltrate: 1) other options are NOT true 2) viral infections show mainly lymphoplasmocytic infiltrate 3) lymphocytes are mainly involved in acute inflammation 4) red blood cells presence represents a manifestation of infectious inflammation

2) viral infections show mainly lymphoplasmocytic infiltrate lymphocytic infiltrate = relating to, or consisting of lymphocytes and plasma cells. Lymphocytes are often involved in chronic inflam.

Which of the following is/are true for hamartoma? 1) It is a bleeding into joints 2)tumor derived from erroneously involved tissue/s 3) there is a bleeding into abdominal cavity 4) there are a subperiosteal hematomas of the skull bones after birth

2)tumor derived from erroneously(feilaktig) involved tissue/s A hamartoma is a benign, focal malformation that resembles a neoplasm in the tissue of its origin. This is not a malignant tumor, and it grows at the same rate as the surrounding tissues.

haemorrhagic (red) infarction is typically described in: 1. kidneys 2. lungs 3. the spleen 4. the myocardium

2,

a ganglion cyst is a 1. benign tumor 2. pseudotumor 3. pseudocyst 4. cyst

2, 3

Select the correctly paired statement(s) 1.subdural hemmorage= hemocephalus 2. hemoptysis= blood in sputum 3. Metrorrhagia= uterine bleeding outside the menstrual cycle 4. hemoptoe= blleding into the lumen of the stomach

2,3

basalioma is a tumor: 1. with a high metastatic potential 2. that is malignant 3. that is locally agressive 4. that is benign

2,3

choose the cell(s) that are not diagnostic for classic Hogkin lymphoma: 1. Reed-stemberg cells 2. L+H cells 3. lymphoblasts 4. Hodgkin cells

2,3

choose the correct statement(s) 1. apoptosis is a form of programmed cell death, described only in tumors 2. apoptosis does not incite an inflammatory reaction 3. necrosis is not a physiological reaction 4. the apoptitic bodies are not enveloped by a membrane

2,3

choose the tumor(s) NOT belonging to testicular germ cell neoplasias: 1. choriocarcinoma 2. sertoli cell tumor 3. leydig cell tumor 4. seminoma

2,3

choose the valid statement(s) about cholera: 1. the saliva of the sick is the source of infection 2. the small intestine is mostly affected 3. the patient dies of mineral imbalance 4. the microbe produces endotoxin

2,3

enterobius: 1. is found only perianally 2. may lead to appendicitis 3. is highly infectious 4. it can cause encephalitis

2,3

select the valid statement(s) about choriocarcinoma 1. it is an example for a mixed tumor 2. it is an example for a germ cell tumor 3. it is an aggressive tumor with a early lymph node metastasizing 4. its microscopic features are rosettes and pseudorosettes

2,3

select the valid statement(s): 1. cholersterolosis always affects hepatocytes 2. steatosis typically affects hepatocytes 3. cholesterolosis is an example for microvesicular steatosis 4. the basis for hepatic steatosis is the accumulation of foamy macrophages

2,3

what kind of neoplastic cells characterize DLBCL 1. cells resembeling lipid-laden macrophages 2. centroblasts 3. immunoblasts 4. plasmablasts

2,3

which of the following histologic changes are characteristic for marfan syndrome: 1. prominent mediocalcinosis 2. cystic medionecrosis 3. collection of mucinous material positive with Alcian blue 4. accelerated artherosclerosis

2,3

acute myeloid leukemia is characterized by: 1. lymphoblasts 2. bone marrow infiltration 3. myeloblasts 4. the presence of philadelphia chromosome

2,3 (Lymphoblasts + philadelfia chromosome is in acute lymphoblastic leukemia)

choose the valid statement(s) about the diphteric inflammation of the tonsils: 1. we describe koplik spots 2. the ulcerated surface of the tonsil is covered by a thick pseudomembrane 3. the patient is endangered by cardiotoxin produced by C. diptheriae 4. the detachment of the pseudomembrane is not followed by bleeding

2,3,4

colliquative necrosis: 1. is described only in organs with a high protein content 2. can be caused through the action of proteolytic enzymes 3. can be a consequense of purulent inflammation 4. it is usually not described in sarcoidosis

2,3,4

in clostridial infection of the muscles we usually describe: 1. an inflammatory infiltrate consisting mostly of lipophages 2. numerous gram-positive rod-shaped microorganisms 3. extensive necrosis of the muscle fibers 4. gas bubbles

2,3,4

in which organ(s) is lipomatous atrophy usually described: 1. liver 2. pancreas 3. thymus 4. bone marrow

2,3,4

choose the statement(s) NOT true for multiple myeloma: 1. it affects the skeleton 2. is typically followed hypocalcemia 3. it causes pathological fractures 4. is may not be followed by metastatic calcification in the lungs

2,4

choose the statements characterizing yolk sac tumors: 1. it is not a component of mixed germ cell tumors 2. schiller-Duval bodies are present 3. it occurs mostly in women 4. it can secrete alpha-fetoprotein

2,4

hyperthrophy of the right cardiac ventricle: 1. it exists for a relatively long time as concentric hypertrophy 2. it develops due to pulmonary hypertension 3. it is the cause of siderotic induration of the lungs 4. it can accompany left side heart failure

2,4

hypovolemic shock: 1. is caused by the release of endotoxin into the bloodstream 2. may accompany haemorrhages 3. is present in the terminal stage of chronic heart failure 4. may be cause by dehydration

2,4

in the rheumatic nodule we observe: 1. the centrum contain caseous necrosis 2. the centrum contain fibrinoid necrosis 3. several foreign body-type giant cells 4. anickov aschoff cells

2,4

select the correctly paired statements 1. myasthenia gravis = type 1 immunopathological reaction 2. immune-complex vasculitits = type 3 immunopathological reaction 3. graves disease = delayed type hypersensistivity 4. asthma = type 1 immunopathological reaction

2,4

small cell lung carcinoma: 1. has a relatively favorable prognosis 2. it belongs among neuroendocrine tumours 3. it belongs among adenocarcinomas 4. consists of small basophilic cells

2,4

what is characteristic for measles infection: 1. koplik spots on the conjuctiva 2. warthin-finkeldey cells are microscopically present 3. infection during pregnancy never causes fetal malformation 4. the rash is maculopapular

2,4

Which of the following statement represents an ethiopathogenetic connection? 1. Inhalation of asbestosis fibers and coal workers pneumoconiosis 2. Asbestosis and fhe development of mesothelioma 3. Cigarette smoking and urothelial carcinoma 4. Silicosis and cor pulmonale chronicum

2. Asbestosis and fhe development of mesothelioma 3. Cigarette smoking and urothelial carcinoma 4. Silicosis and cor pulmonale chronicum

Which of the following is/are true of TP53 1. It is a protooncogene 2. Belongs to the tumor suppressor genes 3. Its mutation leads to the development of the so-called Philadelphia chromosome 4. Its mutation in tumors is rerely present

2. Belongs to the tumor suppressor genes

Chronic inflammation can cause 1. AE amyloidosis 2. Cachexia 3. Elevation in Serum AA protein levels 4. Anaemia

2. Cachexia 3. Elevation in Serum AA protein levels 4. Anaemia

Liquefactive necrosis 1. Is described in organs with a high protein content 2. Can be caused through the action of proteolytic enzymes 3. Can be present in stroke 4. Is usually not described in encephalomalacia

2. Can be caused through the action of proteolytic enzymes 3. Can be present in stroke

Which statement are true of lyme boreliosis 1. Is spread by Anopheles spp 2. Causes neurologic complaints 3. A typicl dermal symptom is erythema migrans 4. Is caused by a Corynabacterium

2. Causes neurologic complaints 3. A typicl dermal symptom is erythema migrans

Select the most frequent brain tumor in adults 1. Medulloblastoma 2. Glioblastoma 3. Pilocytic astrocytoma 4. Oligodendroglioma

2. Glioblastoma

Which stetemnt is/are true of pigments 1. They are usually invisible in unstined histologic sides 2. Hepatocytes can produce endogenous pigments 3. Can be of endogenous or exogenous origin 4. Anthracotic pigment is of exclusively endogenous origin

2. Hepatocytes can produce endogenous pigments 3. Can be of endogenous or exogenous origin

Select the valid statement about syphilis 1. Miliary and gummatous forms are distinguished 2. In tertiary syphilis there are no living traponemas in the blood 3. The manifestations of secondary syphilis include gummas and aortitis luetica 4. The manifestation of tertiary syphilis include tabes dorsalis and progressive paralysis

2. In tertiary syphilis there are no living traponemas in the blood 4. The manifestation of tertiary syphilis include tabes dorsalis and progressive paralysis

Which of the following s/are true of sarcoma botryoides 1. Is a benign intraepitheliak tumor of adulthood 2. Is embryonal rhabdomyosarcoma 3. Is a malignant mesenchimal neoplasm occuring primarly in children under age 5 4. Is a tumor seen most commonly in the brain

2. Is embryonal rhabdomyosarcoma 3. Is a malignant mesenchimal neoplasm occuring primarly in children under age 5

Type II immunopathologic reaction 1. Is dependent on the presence of immune complexes 2. Is mediated by IgG antibodies 3. The thyroid gland can be affected 4. A typical example is systemic lupus erythomatodes

2. Is mediated by IgG antibodies 3. The thyroid gland can be affected

Which of the following is/are true of paraneoplastic syndrome 1. Is typically caused by the locally aggressive growth of the tumor 2. Is often found in small cell lung cancer 3. An example is myasthenia gravis 4. Is not described in neuroendocrine tumors

2. Is often found in small cell lung cancer 3. An example is myasthenia gravis

Which of the following statement is true of gangreen 2. It can be caused by trauma

2. It can be caused by trauma

Which of the following about amyloid is/are true 1. It is polysaccharide 2. It can be deposited in the kidney during chronic inflammation in the organism 3. It is deposited mostly intracellularly 4. Its ultrastructure consists mostly of alpha-helices

2. It can be deposited in the kidney during chronic inflammation in the organism

Choose the statement valid for mulluscum contagiosum 2. It is a viral infection 4. It is an high contagious bacterial infection

2. It is a viral infection

Which statement is/are true of DLBCL 1. It has a slow clinical course 2. It is made up of centroblasts and immunoblasts 3. It is present in Richter's syndrome 4. The affected lymph nodes are usually shrunken and soft to the touch

2. It is made up of centroblasts and immunoblasts 3. It is present in Richter's syndrome

Which statement is/are true of neuroblastoma 1. It never metastasizes 2. It's a tumor consisting of small blue cells 3. It's rarely found in children 4. It is most frequently found in the cerebellum

2. It's a tumor consisting of small blue cells

Select the correctly paired statement 1. BRAF= tumor suppressor gene 2. KRAS= protoncogene 3. APC= a tumor suppressor gene 4. BRCA1= a tumor suppressor gene

2. KRAS= protoncogene 3. APC= a tumor suppressor gene 4. BRCA1= a tumor suppressor gene

HPV infection can be typically present in the following diseases 1. Condylomata lata 2. Koilocytosis of the squamous epithelium of the cervix 3. Serous ovarian carcinoma 4. Squamous carcinoma of the lung

2. Koilocytosis of the squamous epithelium of the cervix

Which statement is /are true of B and T lymphocytes 1. Plasma cells develop from fransformed T-lymphocytes 2. Mantle cell lymphoma is a B-lymphoma 3. Thir clonal prolififeration is typical for lymphomas 4. The so-called natural killer are transformed macrophages

2. Mantle cell lymphoma is a B-lymphoma 3. Thir clonal prolififeration is typical for lymphomas

In the stroma of which thyroid cancer can we find amyloid 1. Anaplastic carcinoma 2. Medullary carcinoma 3. Follicular carcinoma 4. Papillary carcinoma

2. Medullary carcinoma

Which staining method can be used to highlight hemosiderin? 1. Alcian blue 2. Perls Prussian blue 3.PAS 4. Methods staining divalent iron ions

2. Perls Prussian blue 4. Methods staining divalent iron ions

Pulmunary alveoli filled with fibrin-rich exudate, numerous erythrocytes and granulocytes preferably correspond to which of the following stages of lobar pneumonia? 1. In none of these stages 2. Red hepatissation 3. Resolution 4. Carnification

2. Red hepatissation

cerebriform nuclei are typical for: 1. glioblastoma multiforme 2. medullary thyroid carcinoma 3. mycosis fungoides 4. nodular paraganglioma

3

choose the correct statement(s) about gangrene 1. mummification is an example of wet gangrene 2. gas gangrene is usually caused by ischemic lower limb disease 3. dry gangrene usually affects the extremities 4. an example of dry gangrene is noma

3

choose the correct statement(s) about syphilis: 1. when treated it usually manifests in three stages 2. the first stage is characterized by ulcus molle 3. gummas are described in tertiary syphilis 4. a patient with neurosyphilis is highy infectious

3

choose the option(s) NOT representing a risk factor for urinary bladder cancer (regardless of histo. type): 1. HPV infection 2. cigarette smoking 3. repeated catheterization 4. schistosoma haematobium infection

3

choose the tumors not typically described in childhood: 1. Wilms tumor 2. DLBCL 3. oligodendroglioma 4. medulloblastoma

3

cribriform arrangement is typical for: 1. some schwannomas 2. dermatofibrisarcoma protuberans 3. prostatic carcinoma 4. thyroid carcinoma

3

hyperplasia: 1. is the opposite of aplasia 2. it may affect neurons in the CNS 3. it may combined with hypertrophy 4. in most cases is irreversible

3

paradoxical embolisation: 1. is always present in hypertension 2. the thrombus moves from the left side to the right side of the heart 3. its basis is a patent foramen ovale 4. it occurs in atrial fibrosis

3

posthepatic (mechanical)jaundice usually occurs in: 1.viral hepatitis 2. occurs in fetal erythroblastosis 3. carcinoma of the pancreatic head 4. hepatocellular carcinoma

3

the immunopathological reaction of the 2nd type: 1. Is always mediated by IgE antibodies 2. only endogenous antigens can cause this reaction to arise 3. a typical example is autoimmune hemolytic anemia 4. this kind of immunopathological reaction is always complement dependant

3

the tuberculous granuloma contain: 1. eosinophils 2. langerhans cells 3. epitheloid histiocytes 4. pratt-type multinucleated giant cells

3

what kind of cell(s) compose the uterine leiomyoma? 1. rhabdomyoblasts 2. giant cells of the osteoclastic type 3. spindle cells with a cigar-shaped nucleus 4. spindle cells with a comma-shaped nucleus

3

which of the following does not always have to be a germinal tumor? 1. yolk sac tumor 2. dysgerminoma 3. choriocarcinoma 4. polyembryoma

3

Choose the statement that describes the correct sequence of events: 1) active hyperemia, transudation, passive hyperemia 2) passive hyperemia, exudation, active hyperemia 3) active hyperemia, passive hyperemia, exudation 4) cell infiltration, passive hyperemia, inflammation and edema

3)

Manifestations of tuberculosis do not include: 1) caseous pneumonia 2) basilar meningitis 3) ulcus durum 4) Pott abscess

3)

Select the valid statement(s) about sepsis: 1) it is a generalised response of the organism to viral infection 2) it has the same clinical picture as premier 3) an increased number of leukocytes in peripheral is characteristically described 4) the characteristic finding at autopsy is induration of spleen

3)

Thromboembolism to pulmonary arteries: 1) occurs only when the foramen ovale is open 2) is a common complication of thrombosis of the left cardiac auricle extremities 3) is a common complication of deep vein thrombosis of the lower extremities 4) leads to dilation of the right ventricle

3) 4)

Struma ovarii: 1) is basically primary ovarian follicular carcinoma 2) it is of metastatic origin 3) may occur in mature ovarian follicles 4) it is the same as Kruckenberg tumor

3) is a rare form of monodermal teratoma that contains mostly thyroid tissue, which may cause hyperthyroidism. Despite its name, struma ovarii is not restricted to the ovary. The vast majority of struma ovarii are benign tumors; however, malignant tumors of this type is found in a small percentage of cases

Inflammatory edema is caused by: 1) increased hydrostatic pressure 2) reduced oncotic pressure 3) increased permeability of capillaries 4) the release of histamine and heparin from mast cell granules

3) 4)

Schloffer tumor: 1) is a semimalignant tumor 2) histologically is a mixture of epithelial and mesenchymal components 3) is associated with proliferative inflammation 4) belongs among pseudotumors

3) 4)

Select the valid statement(s): 1) the granular cell tumor belongs among germinal tumors 2) mature teratoma in the ovary is usually highly malignant 3) mature teratoma belongs among germ cell tumors 4) differentiated teratoma with malignant transformation belongs among germ cell tumors

3) 4) (GRANULAR cell tumor is a lesion of neural derivation)

Select the valid statement(s) about glioblastoma multiforme: 1) it is a CNS tumor typically affecting children 2) microscopically we distinguish between Antoni A and Antoni B regions 3) typically we describe extensive tumor necrosis deem in the surrounding tissue 4) it is a grade IV tumor according to the WHO grading system

3) 4) Antoni A and Antoni B is for Schwannoma. Typically affects patients older than 50.

Select the valid(s) about squamous cell carcinoma: 1) in the lungs it occurs only rarely 2) it is the same as adenosquamous carcinoma 3) is is the same as spinalioma 4) it can include cancroid pearls

3) (Squamous-cell carcinoma, spinalioma, epidermoid carcinoma are synonyms ) 4)

Which of the following is/are NOT an autoimmune disorder/s: 1) Graves disease. 2) Myasthenia gravis. 3) Di george syndrome. 4) Diabetes mellitus.

3) Di george syndrome. -Di george: a syndrome caused by the deletion of a small piece of chromosome 22 - CATCH 22 -Graves: hyperthyroidism, frequently causing it to enlarge to twice its size or more -Myasthenia: neuromuscular disease that leads to fluctuating muscle weakness and fatigue

Hepatic metabolism impairment is typically associated with: 1) Gilbert syndrome 2) Dubbin Jonson syndrome 3) Down syndrome. 4) Rotor Syndrome.

3) Down syndrome Again, the question must be: hepatic metabolism impairment is typically NOT associated with...? Gilbert is due to a defective liver enzyme and leads to accumulation of unconjugated bilirubin. Dubin Johnson and Rotor are also due to defective liver enzymes and lead to accumulation of conjugated bilirubin

The cause of hepatic steatosis in adults include: 1) gallstones 2) TB 3) Ethylism 4) hepatocellular carcinoma

3) Ethylism = chronic alcoholism

Uterine cervix specimen microscopically shows severe dysplasia of the surface squamous epithelium (CIN III). Which of the following factors is likely to have contributed most to the development of this lesion: 1) Estrogen therapy in postmenopausal women 2) Exposure to dietylstilbestrol 3) Frequent sexual partners 4) Infection of Candida albicans

3) Frequent sexual partners. -Dietylstilbestrola: synthetic nonsteroidal estrogen -Candida albicans: a diploid fungus that grows both as yeast and filamentous cells and a causal agent of opportunistic oral and genital infections in humans

Which of the following disease is not a congenital disorder of pigmentation: 1) ochronosis 2) albinism 3) gout 4) hemochromatosis

3) Gout -Ochronosis: accumul.of homogentistic acid, bluish/black discoloration -Albinism: absence of pigment, defect tyrosinase -Gout: crystallization of uric acid -Hemochromatosis: ion deposition, hyperpigmentation

Which of the following is/are true for the role of immune cells? 1) Cells remain in the node where they were created. 2) NK cells are part of the aquired immune response. 3) Interdigiting dendritic cells present antigens to T lymphocytes. 4) Macrophages serve mainly to the destruction of antigens.

3) Interdigiting dendritic cells present antigens to T lymphocytes. -They move around between lymph nodes and blood, they don't return to thymus or BM -NK cells are part of innate. -Macrophages engulf and digest cellular debris, foreign substances, microbes, and cancer cells in a process called phagocytosis. Main function is probably Ag-presentation, and activation of acquired immune system.

Which of the following is/are NOT true for multiple myeloma? 1) is associated with paraproteinemia 2) is associated with the formation of defective immunoglobulins 3) is a reactive leasion 4) belongs among lymphomas

3) Is a reactive leasion 4) what do you guys think? -Multiple myeloma: cancer of plasma cells, second most prevalent after non-hodgkin´s -Paraproteinemia: abnormal antibody causing kidney disease -Reactive lesion: painless production of hyperplastic tissue resulting from repair respond BUT I DONT THINK IT IS A LYMPHOMA.... cause its not from lymphatic cells is it, it is from proliferation of plasma cells in bone marrow _

Which of the following is/are true for thrombosis: 1) It is intravital intra and extravascular blood clotting. 2) The multiplication of platelets is always necessary for thrombi formation. 3) It is intravital blood clotting in the cardiovascular system. 4) No recanalization ios seen in the process of organization of thrombi.

3) It is intravital blood clotting in the cardiovascular system.

Which of the following is/are true of GIST: 1) it is always a highly malignant tumor 2) it is not related to mutations in the gene c-kit 3) it usually involves the retroperitoneum 4) is typically associated with carcinoid syndrome

3) It usually involves the retroperitoneum. -Also involves liver and omentum -GIST: gastrointestinal stromal tumor is a mesenchymal tumor. Non-epithelial -It IS related to mutations in the gene c-kit -Carcinoid syndrome: occurs secondary to carcinoid tumors. Includes diarrhea, flushing and also heart failure + bronchoconstriction. Flushing (red in the face) is also a cardinal symptom of carcinoid syndrome—the syndrome that results from hormones (often serotonin or histamine) being secreted into systemic circulation.

Which of the following is/are true for embryonal carcinoma? 1) Other opinions are not true. 2) It is the other name for an immature teratoma. 3) Its a typical germinal tumor. 4) Tends to have light PAS positive cytoplasm.

3) Its a typical germinal tumor. -Embryonal c: uncommon type of germ cell tumour that occurs in the ovaries and testes -Immature teratoma: a rare type of malignant (cancerous) germ cell tumor (type of tumor that begins in the cells that give rise to sperm or eggs).

Which of the following is/are inherited *metabolic* disorders with manifestations in the CNS: 1) Alzheimer´s disease 2) Parkinsons`disease 3) Niemann-Pick disease 4) Paget`s disease

3) Niemann-Pick disease Niemann-Pick disease= sphingomyelin to accumulate in lysosomes Alzheimer's and Parkinson´s disease is classified as a neurodegenerative disorder. Paget's is caused by the excessive breakdown and formation of bone, followed by disorganized bone remodelling.

Giardiasis is a: 1) bacterial infection 2) autoimmune mechanism 3) protozoa infection 4) viral infection

3) Protozoa infection. -It inhabits digestive tract, most common pathogenic parasitic infection

Which of the following is/are characteristic of inflammation seen in erysipelas? 1)Abscess 2)Empyema 3)Phlegmone. 4)Fibrinous.

3)Phlegmone. -Phlegmone: a spreading diffuse inflammatory process with formation of exudate or pus -Erysipelas: an acute infection of the upper dermis and superficial lymphatics, usually caused by streptococcus bacteria. -Empyema: a collection of pus within a naturally existing anatomical cavity, e.g pleural empyema -Abscess: a collection of pus in a newly formed cavity within tissues

Granulomas are typical morphological manifestation of: 1) Mucoviscidosis 2) Asbestosis. 3) Sarcoidosis 4) Tuberculosis.

3) Sarcoidosis 4) Tuberculosis -Sarcoidosis: a disease involving abnormal collections of inflammatory cells (granulomas) that can form as nodules in multiple organs. Most often located in the lungs or its associated lymph nodes, but any organ can be affected. -Tuberculosis: infectious disease caused by various strains of mycobacteria, usually Mycobacterium tuberculosis. -Granulomas: an inflammation, a collection of immune cells known as macrophages. -Mucoviscidosis/cystic fibrosis: An autosomal recess. genet. disorder that affects mostly the lungs but also pancreas, liver, and intestine. Accumul. of excessively thick and tenacious mucus and abnormal secretion of sweat and saliva. -Asbestosis: chronic inflamm. and fibrotic medical condition affecting parenchymal tissue of lungs caused by the inhalation and retention of asbestos fibers.

Which of the following vessels is/are affected in polyarthritis nodosa? 1) Arterioles. 2) Venules. 3) Small and medium sized muscular type arteries. 4) Capillaries.

3) Small and medium sized muscular type arteries. -Also called Kussmaul disease: is a vasculitis of medium and small-sized arteries, which become swollen and damaged from attack by rogue immune cells.

Which of the following is/are true for tuberculosis in adults: 1) other options are NOT true 2) TB leptomeningis is a common event 3) the most common location are the apical part of the lungs 4) the process tends mainly to hematogenous dissemination

3) The most common location are the apical part of the lungs Hematogenous =produced by or derived from the blood. dissemination = formidling Hematogenous dissemination of bacteria is part of the pathophysiology of meningitis and endocarditis, and of Pott's disease and many other forms of osteomyelitis. chronic meningitis can be caused by Tuberculosis

Which of the following is/are NOT true for neuroendocrine tumors of the gastrointestinal tract? 1) May produce hormonally active mediators. 2) May cause carcinoid syndrome. 3) They are unconditionally benign. 4) May cause impariment of the heart valvular apperatus.

3) They are unconditionally benign. -Many are benign, while some are malignant.

ANCA antibodies are found in: 1) nephrocalcinosis 2) gouty kidney 3) wegener`s granulomatosis 4) acute streptococcual glomerulonephritis

3) Wegener`s granulomatosis -ANCA antibodies: anti-neutrophil cytoplasmic antibody = autoantibodies (IgG mostly) -Nephrocalcinosis: deposits of Ca2+ in renal parenchyma due to hyperparathyroidism W.G = It is a form of vasculitis (inflammation of blood vessels) that affects small- and medium-size vessels in many organs. Damage to the lungs and kidneys can be fatal.

Cyanotic indruration develops in the lungs in settings of: 1) acute left heart insufficiency 2) shock 3) chronic insufficiency of the left heart 4) in chronic right heart insufficiency

3) chronic insufficiency of the left heart cyanotic induration: Induration related to chronic venous congestion in an organ or tissue, frequently resulting in fibrous thickening of the walls of the veins and eventual fibrosis of adjacent tissues. why not in chronic right heart deficiency though?

Gout is characterized by: 1) a black discoloration of cartilage 2) caseous granuloma formation 3) creation of tophus 4) destruction of articular cartilage

3) creation of tophus It is caused by elevated levels of uric acid in the blood. The uric acid crystallizes, and the crystals deposit in joints, tendons, and surrounding tissues.

Which of the following disease is not a congenital disorder of pigmentation: 1) ochronosis 2) albinism 3) gout 4) hemochromatosis

3) gout

Which of the following is/are true for apoptosis and necrosis: 1) necrosis is a process regulated by CASPases 2) the difference between necrosis and apoptosis is only quantitative 3) in apoptosis individual cells disappear resulting in tissue shrinkage 4) apoptosis is caused primarly by a damaged cell membrane

3) in apoptosis individual cells disappear resulting in tissue shrinkage (caspases activate apoptosis)

Which of the following is/are true for nephroblastoma: 1) it is a primary CNS tumor 2) it typically occurs in adulthood 3) it typically occurs in childhood 4) the occurrence in childhood is rare

3) it typically occurs in childhood = Wilms tumor cancer of the kidneys rarely in adults.

Which of the following is true for germinal tumors. 1) other options are NOT true 2) struma ovarii does not belong among germinal tumors 3) may be benign or malignant 4) occur only in gonads

3) may be benign or malignant germ cell tumors occur both within and outside the ovary and testis. inside the cranium — pineal and suprasellar locations are most commonly reported inside the mouth — a fairly common location for teratoma neck mediastinum — account for 1% to 5% of all germ cell neoplasms pelvis, particularly sacrococcygeal teratoma, ovary, testis

Which of the following is/are true of chronic inflammation?: 1) arises in the mucous membranes 2) other options are NOT true 3) may be superficial or deep 4) its typical manifestation is a blister

3) may be superficial or deep maby nr1 is false because it doesn't always arise there... but it can occur in mucous membranes, so this is def up for dicussion tydeligvis kan ikke chronic inflam. arise in mucous membrane? dette må vi undersøke.

Which of the following tumors of the thyroid gland can show a presence of amyloid 1) papillart carcinoma 2) anaplastic carcinoma 3) medullary carcinoma 4) follicular adenoma

3) medullary carcinoma

Typical morphological signs of myocardial infarction do not cover: 1) cardiomyocyte nuclei decay 2) infiltration of neutrophilic granulocytes 3) perivascular amyloid deposition 4) the disintergration of cell membrane of cardiomyocytes

3) perivascular amyloid deposition

Which of the following changes is/are NOT characteristic for chronic left-heart failure? 1) brown induration (forhardning) of lungs 2) the presemce of siderophages in pulmonary alveoli 3) pulmonary artery arteriosclerosis 4) impaired gas diffusion

3) pulmonary artery arteriosclerosis A siderophage is a macrophage that has absorbed iron-containing particles

Which of the following is/are true for follicular lymphoma (FL): 1) diagnostic RS cells are present in this tumor 2) a tumor formed of large B-cells 3) the most common type of adult lymphoma 4) a tumor formed of large T-cells

3) the most common type of adult lymphoma It is defined as a lymphoma of follicle center B-cells (centrocytes and centroblasts), which has at least a partially follicular pattern.

mantle cell lymphoma is characterized: 1. by bcl-2 expression in the neoplastic cells 2. has excellent prognosis with an average survival of 10-15 years 3. it affects the skin in the form of mycosis fungoides 4. it affects the waldeyer circle

4 1. is wrong because its a t(11;14) translocation, i.e translocation of cyclin d1 (cell cycle regulator) to Ig heavy chain.... (BCL2 is amplified in follicular lymphomas) 2. is wrong because The overall 5-year survival rate for MCL is generally 50%. 3. is wrong because mycosis fungoideus is related to cutaneous T-cell lymphoma. 4. is correct because....... can't find any info bout it... put here if you see it

which of the following conditions is typically observed in chronic right sided heart failure 1. pneumonia, hematuria 2. brown induration lungs, myochardial ishemia 3. pulmonary edema, vena cava thrombosis 4. pleural effusion, nutmeg liver

4 . (Pleural effusion is excess fluid that accumulates in the pleural cavity,) (pulmonar edema- left sided heart failure)

Edema of the lower extremities, hepatomegaly and jugular vein distension are typical symptoms of: 1) brown induration 2) shock 3) liver cirrhosis 4) right sided cardiac insufficiency

4)

Select the valid statement(s): 1) nephroblastoma typicaly affects adults 2) angiomyolipoma is a very common malignant renal neoplasm 3) malignant tumors of the kidney typically spread via peritoneal implantation metastases 4) urothelial carcinoma may occur in the pelvis of the kidney

4)

Which disease(s) are not caused by prions? 1) Kuru 2) Creutzfeldt-Jakob disease 3) Bovine spongiform encephalopathy 4) Krabbe leukodystrophy

4)

Acanthosis is defined as: 1) a vacuolar dystrophy of keratinocytes in the basal epidermis 2) intracellular edema of keratinocytes of squamous epithelium, sometimes with the formation of vesicles 3) intercellular edema of squamous epithelium, sometimes with the formation of vesicles 4) a thickening of the epidermis

4) A thickening of the epidermis. -Hyperplasia of Malphigian layer = str.basale + str.spinosum

Basilar meningitis is: 1) of unknown etiology 2) caused by pneumococcal infections 3) caused by herperic infections 4) caused by mycobacterial infection

4) Caused by mycobacterial infection. -Usually caused by TB mycobacteria

Which of the following applies to trophoblastic disease: 1) changes induced by vaccination 2) changes induced by viral infection 3) other options are NOT true 4) includes moa hydatidosa

4) Includes moa hydatidosa. Moa hydatidosa: clump of growing tissue in e.g gestitional trophoblastic disease, where a non-volatile fertilized egg will implant in uterus and fail to come to term.

Which of the following is/are true of tuberculous epinephritis? 1) Always leads to sterility. 2) Is formed by non specific granulation tissue. 3) Advanced double sided form is the cause of chronic adrenal hyperplasia. 4) Is made up of specific granulation tissue.

4) Is made up of specific granulation tissue.

.

4) Is made up of specific granulation tissue. -

Condyloma accuminatum: 1) is caused by HSV type 1 infection. 2) is caused by the chlamydial infection 3) is caused by HSV type 2 infection. 4) Its a warty condyloma. 5) In most cases progression into malignancy is described

4) Its a warty condyloma. 5) In most cases progression into malignancy is described -CA: an epidermal manifestation attributed to the epidermotropic human papillomavirus (HPV)

Which of the following disorders do not belong among congenital liver disorders associated with impaired metabolism of bilirubin? 1) Rotor syndrome. 2) Crigler-Najjar syndrome. 3) Dubin-johnson syndrome. 4) Recklinghausen disease.

4) Recklinghausen disease. -Recklinghusen: also called Neurofibromatosis type I (NF-1). A tumor disorder caused by mutation of a gene on chromosome 17 responsible for control of cell division. Causes tumors along the nervous system!

Which of the following is/are true for dystrophic calcification: 1) results from hypercalcemia 2) an example is stone formation 3) is typical of fibrinoid necrosis 4) storing of calcium precedes tissue damage

4) Storing of calcium precedes tissue damage -Occurs in degenerated or necrotic tissue

The presence of in situ germinoma in testicular tissue is associated with: 1) Contralateral testicular torsion. 2) Testicular leukemic infiltration. 3) Gonococcal infections. 4) The failure of testicular descent.

4) The failure of testicular descent.

Mucous content in carcinomas can be specifically illustrated using: 1) blue tricrome 2) congo red 3) berlin blue 4) alcian blue

4) alcian blue

Pulmonary embolism: 1) its source is surface thrombophlebitis 2) hyaline thrombi are evident macroscopically 3) does not arise in the postoperative period 4) can be causes by a thrombus

4) can be causes by a thrombus blockage of the main artery of the lung or one of its branches by a substance that has travelled from elsewhere in the body through the bloodstream (embolism).

Which of the following is/are true for varicella: 1) is a bullous disease, which usually heals with scars 2) polyradiculoneuritis is frequrntly a serious complication 3) is often accompanied by giant cell pneumonia 4) herpes zoster represents an adult reactivation of the viral infection

4) herpes zoster represents an adult reactivation of the viral infection Chickenpox is a highly contagious disease caused by primary infection with varicella zoster virus. mostly heal without scarring. can get: complications, such as varicella pneumonia.

which of the following is/are true for dysgerminoma: 1) it is another name for a mature teratoma 2) this is another name for an immature teratoma 3) this is another name for a mature teratoma with malignant progression 4) it is an ovarian seminoma

4) it is an ovarian seminoma A dysgerminoma is a type of germ cell tumor;it usually is malignant and usually occurs in the ovary. Seminoma (also known as pure seminoma or classical seminoma) is a germ cell tumor of the testis or, more rarely, the mediastinum or other extra-gonadal locations. A teratoma is a tumor with tissue or organ components resembling normal derivatives of more than one germ layer.

Which of the following is/are true of cor pulmonale: 1) it is a major cause of pulmonary sclerosis 2) right heart hypertrophy and cor pulmonale are synonyms 3) other options are NOT true 4) it is right heart hypertrophy- induced by changes in lungs

4) it is right heart hypertrophy- induced by changes in lungs Pulmonary heart disease, also known as Cor pulmonale is the enlargement and failure of the right ventricle of the heart as a response to increased vascular resistance or high blood pressure in the lungs (pulmonary hypertension).

Which of the following is/are true of Barret`s esophagus: 1) a pre-cancerosis of squamous cell carcinoma of the esophagus 2) microscopic image of squamous metaplasia is found in columnar epithelium 3) a pre-cancer of gastric adenocarcinoma 4) it is the pre-cancerosis of adenocarcinoma of the esophagus

4) it is the pre-cancerosis of adenocarcinoma of the esophagus abnormal change (metaplasia) in the cells of the lower portion of the esophagus. characterized by the replacement of the normal stratified squamous epithelium lining of the esophagus by simple columnar epithelium with goblet cells. Adenocarcinoma = It is defined as neoplasia of epithelial tissue that has glandular origin, glandular characteristics, or both.

Which of the following brain tumor/s is/are most common in children: 1) neurilemmoma 2) Glioblastoma 3) meningoma 4) medulloblastoma

4) medulloblastoma A schwannoma = "neurilemoma,"->is a benign nerve sheath tumor composed of Schwann cells

Which of the following is/are included among organ autoimmune diseases: 1) lupus erythematosus 2) other options are NOT true 3) acute rheumatic fever 4) myasthenia gravis

4) myasthenia gravis Lupus erythematosus may manifest as a systemic disease or in a purely cutaneous form. Myasthenia gravis er en alvorlig autoimmun muskelsykdom. Sykdommen karakteriseres av antistoffdannelse mot muskelendeplatenes nikotinerge acetylcholinreseptorer i den postsynaptiske delen av den nevromuskulære synapse. Denne antistoffdannelsen fører til en gradvis destruksjon av acetylcholinreseptorene, noe som resulterer i defekt nevromuskulær transmisjon. Symptomene er økt trettbarhet i muskulaturen Systemic autoimmune diseases include SLE, Sjögren's syndrome, sarcoidosis, scleroderma, rheumatoid arthritis, cryoglobulinemic vasculitis, and dermatomyositis. These conditions tend to be associated with autoantibodies to antigens which are not tissue specific

Which of the following is/are true for necrosis? 1) coagulative necrosis occurs only in mesenchymal tissues 2) no further modification of necrotic tissue is known 3) apoptosis and necrosis differ only in the extent of the affected tissue 4) necrosis is an irreversible tissue damage

4) necrosis is an irreversible tissue damage

Kongophillic angiopathy: 1) develops in purulent inflammations 2) occurs in viral encephalitis 3) occurs in Parkinson`s disease 4) regulary occurs in Alzheimer`s disease

4) regulary occurs in Alzheimer`s disease = Cerebral amyloid angiopathy (CAA) is a form of angiopathy in which amyloid deposits form in the walls of the blood vessels of the central nervous system. Named kongophilic because amyloid is stained by congo red.

Which statement is/are true of cardiac hypertrophy 1. None of the statement are valid 2. Axcentric hypertrophy procedes concentric hypertrophy 3. Proliferation of cardiomyocytes is typical present 4. Can be reversible

4. Can be reversible

Primary tuberculosis complex 1. Is typically described in the kidney 2. Milliary dissemination is present in most cases 3. Undergoes typically metastatic calcification 4. Consists of a primary focus, lymphagiotis of its draining lymphatics and lymphadenitis

4. Consists of a primary focus, lymphagiotis of its draining lymphatics and lymphadenitis

Choose the statement valid for the sentinel lymph node 4. It is always only one lymph node

4. It is always only one lymph node WRONG

Which statement is/are true of GISTs 1. Its made up of cells analogous smooth muscle cells 2. It is excusively a benign lesion 3. Its cells are positive in silver impregnation 4. It is frequently found in the stomach

4. It is frequently found in the stomach

Which stetement is/are true of dysgerminoma 1. It is different name for mature teratoma with malignant change 2. It is a different name for dysplastic embryonal carcinoma 3. It is differet name for mature teratoma 4. It is seminoma of the ovary

4. It is seminoma of the ovary

Brown atrophy is 1. Hemosiderin accumulation in the liver, leading to its atrophy 2. Lipofuscin accumulation in the interstitium of atrophic organs 3. Numeric atrophy with fibrosis of the organ 4. Lipofuscin accuymulation intracellularly

4. Lipofuscin accuymulation intracellularly

Choose the valid statement for brown atrophy 1. Is more prominent in children 2. Hemosiderin pigment is present in the cytoplasm leading to atrophy of the cell 3. Lipofuscin is deposited extracellularly in the affected organs 4. Lipofuscin is accumulated intracellularly

4. Lipofuscin is accumulated intracellularly

The most frequent malignant brain tumor in children is 1. Neuroblastoma 2. Glioblastoma 3. Meningeoma 4. Medulloblastoma

4. Medulloblastoma

Ich of the following is/are true autolysis? 1. Hyperemia is typically present in the affected region 2. Is the same process as apoptosis 3. It is a reversible process 4. None of the other options are true

4. None of the other options are true

Choose the disease caused by immunocomplexes 4. Systemic lupus erythematosis

4. Systemic lupus erythematosis

Which statement is/are true of trophoblastic disease 1. None of the statements are correct 2. It's a disease caused by vaccination 3. It's a disease caused by viral infection 4. The pathologic changes are related to pregnancy

4. The pathologic changes are related to pregnancy

Selecet the valid statement(s) about hepatic steatosis: 1. it macroscopically leads to a gree coloration of the liver 2. it is the deposition of fat in adipocytes located in the portal fields 3.it is the result of impaired beta-oxidation of amino acids 4. common causes include hypoxia or toxic effects

4. common causes include hypoxia or toxic effects

Mycrobacterium avilum: A)causes granuloma tous infiammation B) belong among acis-fast (acidoresistant bacteria) C) ziehl Neelsen staining can be used for its detection D) only people with concurrent M. Tubercolossis infection are at risk for infection

A)causes granuloma tous infiammation B) belong among acis-fast (acidoresistant bacteria) C) ziehl Neelsen staining can be used for its detection

Which of the following statements about atrophy are correct a. It concerns reduction in size of normally developed organs or their parts b. It`s determined only by decrease in cell size c. It`s determined only by loss of cells d. It`s a consequence of decreased size or loss of cells e. Reduction of cell size occurs by autophagocytosis or via proteasome complexes

A, D, E

Leukodystrophy is a term that indicates a. Lysosomal dysfunction of the glial cells b. Lysosomal dysfunction of leukocytes c. Lysosomal dysfunction of endothelia d. Regressive changes of leukocytes e. This term don`t exist

A- lysosmal dysfunction of glial cells

Which of the following are true of paraneoplastic sindrome A. An example is thrombophlebitis migrans(trouseau sign)

A. An example is thrombophlebitis migrans(trouseau sign)

HPV infection can be tipically present in the following diseases D. Condylomata

D. Condylomata WRONG

Hyperplasia is A. Elgargment of the uterus during pregnancy

A. Elgargment of the uterus during pregnancy

Systemic hypertension can lead A. Excentric hypertrophy of the left ventricle

A. Excentric hypertrophy of the left ventricle

Asbestosis A. Fibers asbestos are stored in tissue in so colled ferroginous nodes B. Give rise to lung carcinoma

A. Fibers asbestos are stored in tissue in so colled ferroginous nodes B. Give rise to lung carcinoma

Select the possible complication of myocardial infarction A. Fibrinous pericarditis

A. Fibrinous pericarditis

Acute inflamation of the appendix A. Has often an ulcerousphlagmanous character B. Is accompanied by pronouced fibroproduction around the appendix C. May be complicated by a periappendiceal abscess D. After spontaneus healing a higher risk of developming cancer in the inflamed mucosa of the appendix is described

A. Has often an ulcerousphlagmanous character C. May be complicated by a periappendiceal abscess

Choose the disease with an increse risk of developing of malignant tumor A. Hepatitis b infection

A. Hepatitis b infection

Choose the place where urate crystals are typically deposited A. Interstitium of the kidney C. Dermis

A. Interstitium of the kidney C. Dermis

Amyloid a. Is always localized extracellularly b. Is bound to epithelial structures c. Is commonly a component of cells of parenchymal organs d. Usually don`t injure organs e. Induce inflammatory changes in its surroundings

A. Is always localized extracellularly

The inflammatory pseudomembrane in the course of pseudomembranous inflammation A. Is composed of fibrin and necrotic tissue B. It can occur in the small intestine after treatment with antibiotic C. The inflammatory infiltrate is composed mainly by macrophages D. Is coulored red in blue trichrome

A. Is composed of fibrin and necrotic tissue D. Is coulored red in blue trichrome

Which of the following is not true of systemic AA amiloidosis A. It is caused by pathological immunoglobulin chain production B. It can accompany chronic tuberculosis C. It can accompany chronic suourative processes D. Amyloid is produced by macrophages

A. It is caused by pathological immunoglobulin chain production D. Amyloid is produced by macrophages

Select the valid statement about cystic fibrosis A. It leads to metabsorbtion

A. It leads to metabsorbtion

Select the valid statements about atrophy A. Its histological hallmark is intracellular lipid deposition

A. Its histological hallmark is intracellular lipid deposition WRONG

Diseases with local amyloid formation include A. Medullarythiroidadenoma B. Parkinson's disase C. Type 2 diabetes miellitus

A. Medullarythiroidadenoma B. Parkinson's disase C. Type 2 diabetes miellitus

Clostridia are the causative agents of A. Myonecrosis and toxemia B. Tetanus

A. Myonecrosis and toxemia B. Tetanus

Mallory hialine A. Occurs in hepatocytes B. Can mpbemanifested in chronic alcoholism C. Is described in hialin degradationof the kidney

A. Occurs in hepatocytes B. Can mpbemanifested in chronic alcoholism

Choose the statement valid for a patient with septic shock A. The pulp of the spleen is activated B. It may be tge cause of acute fatty liver C. Can lead to the development of anasarca D. The patient has a positive nitrogen balance e. The patient has tachicardia and tachipnoe

A. The pulp of the spleen is activated B. It may be tge cause of acute fatty liver C. Can lead to the development of anasarca e. The patient has tachicardia and tachipnoe

Choose the disease frequently caused by candida infection A. Thrush

A. Thrush

Primary tuberculosis complex A. Undergo distrophic calcification B Can arise in other organsbiside lungs

A. Undergo distrophic calcification B Can arise in other organsbiside lungs

Which of the following disease is not a congenital disorder of pigmentation A. Virtigo

A. Virtigo

Which of these statements about liquefactive necrosis are true? 1. It originates in tissue with low protein content 2. During its healing pseudocysts are often formed 3. It`s a `substrate`/base for encephalomalacia

All 3 of them

In which of the following organs are aging associated with atrophic changes a. Bone b. Kidneys c. Bone marrow d. Brain

All are correct

chronic venous stasis can lead to 1. cyanotic induration of the spleen 2. nutmeg liver 3. brown induration of lung 4. congestive catarrh of the intestines

All correct

Which of the following characteristics applies to amyloid a. Without structure ultramicroscopically b. Fibrillary structure ultramicroscopically c. It`s stored intracellularly d. It`s stored extracellularly e. It`s made up of beta-pleated sheets

B - fibrillary structure ultramicroscopically D - store extracellularly E - made of Beta-pleated sheets

Deficiency of the enzyme alpha-1,4-glucosidase cause a. Cori disease b. Pompe disease c. Hurler syndrome d. McArdle syndrome e. Gaucher disease

B - pompe disease

Select the valid statements B. Amyloid can be demostrated microscopically with the vircow reaction

B. Amyloid can be demostrated microscopically with the vircow reaction WRONG

What is typical following cellular ischemia a. Increased oxidative phosphorylation b. Increased levels of intracellular calcium c. Decreased amount of intracellular water d. Increased amount of intracellular potassium e. Increased proteosynthesis

B, increased levels of intracellular calcium increased amount of water, decrease in proteosynthesis, less oxidative phosphorylation (ATP production)

Zahn infarct a. Acute myocardial infarct b. Infarction of the liver due to occlusion of the intrahepatic branches of the portal vein c. Infarction of the liver due to occlusion of the hepatic artery d. Splenic infarct e. Combined infarct of the liver and spleen due to portal hypertension

B, infarction of the liver due to occlusion of the intrahepatic branches of the portal vein

Choose which of the following statements about apoptosis are incorrect a. It affect single cells or groups b. Its evoked by activating the gene BCL-2 c. We can encounter it during viral hepatitis d. It occurs by fragmentation of nuclear chromatin e. It can be physiologic or pathologic

B, it is not eveoked by activating the BCL-2 gene, this is more likely to lead to tumor than apoptosis

Ulcerative colitis A. Is an autoimmune inflamation of the colon wall B. Accumulation of neutrophils forming crypt abscesses is described C. Is associated with primary biliarry chirrosis D. Therisk of adenocarcinoma is incresed

B. Accumulation of neutrophils forming crypt abscesses is described D. Therisk of adenocarcinoma is incresed

Hepatocellular carcinoma can be asociated with A. Oral contraceptive B. Alcohol abuse C. Hepatitis B

B. Alcohol abuse C. Hepatitis B

Chronic inflamation can cause a? AL amyloidosis B. Cachexia

B. Cachexia

Dystrophic carcification is typically described A. In the kidney in nephrocalcinosis B. In the leaflets of a systemic aortic valve

B. In the leaflets of a systemic aortic valve

Which of the following are true of GIST A. It belongs to neuroendocrine tumor B. It most frequentky involves the stomach C. It is typically associated with carcinoid syndrome

B. It most frequentky involves the stomach

Which of the following statements about Wilson's disease are correct a. It's induced by aggregation of lead b. It damages neurons of the frontal lobe c. Kayser-Fleischer rings can be found in the cornea d. In the liver there is increased content of copper e. In the liver there is increased conent of iron

C - (learned it from Dr. House) D - remember the copper colored Kayser-Fleischer ring around the eye

Pseudoabscess A.is filled with serofibrinous exudate B. Is confined by a piogenic membrane C. An example is the oviduct filled with pus D. Can occur in the brain

C. An example is the oviduct filled with pus D. Can occur in the brain

Small uncomplicated surgery (eg pigmented nevus excision) A. Never causes inflammation B. Causes defensive type of inflammation C. Causes reparative type of inflammation D. The following inflammation can also have a granulomatous component

C. Causes reparative type of inflammation D. The following inflammation can also have a granulomatous component

Which of the following is characteristic for chronic lymphocytic leukemia C. Frequent infections complication

C. Frequent infections complication

Ulcerous colitis C. It is an inflammotory process with adenomatous polyp formation

C. It is an inflammotory process with adenomatous polyp formation

Free radicals injures cells by chemical alterations to all of the following structures except a. Cell membranes b. Nucleotides c. Glycogen droplets/supply d. Phospholipids e. Disulfide-bridged proteins

C: Glycogen droplets/supply

Hypoplasia originates during a. Cells lost secondary to injury in puberty b. Atrophy as a consequence of hormonal failure c. Inadequate in utero development d. Disuse of biological structures e. Ischemia

C; Inadequate in utero development

Gangrene is a. The designation for the decay of dead tissue due to putrefactive organisms b. Modified necrosis by changing temperatures c. Synonym for wet gangrene, i.e. necrosis modified by wet conditions d. Modified necrosis, for example due to drying e. Necrosis whose appearance is modified by autoimmune mechanisms

D : Modified necrosis, for example by drying (WHAT DO they mean by this??)

Choose the statement valid for the lobar pneumonia D. It is frequentky associated with secondary pleural inflamation

D. It is frequentky associated with secondary pleural inflamation

Select the correctly paired statement D. KRAS = protoncogene

D. KRAS = protoncogene

Atrophy is most often associated with a. Autolysis b. Enlarged cells c. Abnormal cellular differentiation d. Chronic decrease of blood supply e. Increased cellular proliferation

D: Chronic decrease of blood supply Obs! Not acute stop of blood supply --> ischemia

Which of the following statements about atrophy are correct a. Simple atrophy is the opposite of hyperplasia b. Numerical atrophy can`t originate from permanent cell populations c. Simple atrophy and numerical atrophy can`t be combined d. Brown atrophy occurs in parenchymatous organs e. One of the mechanisms of simple atrophy is autophagocytosis

D: brown atrophy occurs in parenchymatous organs

All of the following situations are examples of atrophy except a. Skeletal muscle after severing of its motorneuron b. Skeletal muscle after long-term immobilization for extremity fracture healed in a cast c. Ovary after hypophysectomy d. Endometrium after long-term administration of estrogen e. Brain in a very old person

D; Endometrium after long term administration of estrogen (would lead to hypertrophy I believe)

Which of the following statements about necrosis are true a. The cell membranes stay intact b. The organelles remain intact c. The nuclear membrane stays intact d. Necrosis don`t induce any inflammatory reaction e. Between two basic types belongs coagulative and liquefactive necroses

E : two basic types are coagulative and liquefactive necrosis. The last type vernerova thinks is a basic type is: fibrinoid

Which of the following characteristics are not typical for amyloid a. It has an amorphous eosinophilic appearance in histologic preparations b. It shows yellow-green birefringence after being stained with Congo red c. In electron microscopy it has a fibrillary structure d. Its caused by inflammatory reactions e. It stains similarly to starch with Lugol solution

It is NOT caused by inflammatory reactions

Which of the following is/are true for follicular lymphoma (FL): 1) diagnostic RS cells are present in this tumor 2) a tumor formed of large B-cells 3) the most common type of adult lymphoma 4) a tumor formed of large T-cells

Non are correct I think! It is defined as a lymphoma of follicle center B-cells (centrocytes and centroblasts), which has at least a partially follicular pattern. 2 is wrong because it is formed of small B cells. 4 is wrong cause B cells not T cells. 1. is wrong cause RS(reed sternberg cell) are giant cells found in Hodgkins Lymphoma.... (Follicular lymphoma is a non-hodgkins l.) 3. The most common adult lymphoma is DLBCL, not follicular lymphoma....?

Which of the following stains stain amyloid a. Congo red b. Thioflavin-T c. Methyl violet d. Alcian blue e. Prussian blue

a Congo red b. Thioflavin-T

Which of the following phenomena don`t belong along the uncertain markers of death a. Death spots (Livores) b. Cessation of breathing c. Cessation of cardiac activity d. Areflexia e. Fall in body temperature

a Death spots - livores

In which of the following conditions plays cyclin D1(PRAD-1) an important role? a) Lymphoma cells from the plastic zone b) CLL/SLL c) Mycosis fungoides d) Hodgkins disease e) Uterine leiomyoma

a) Lymphoma cells from the plastic zone (mantle cell lymphoma)

Which factor(s) cause the tissue injury occurring in reperfusion: a) Toxic oxygen radicals b) TNF c) Endotoxin d) Apoptosis

a

for colorectal malignancies the following statement(s) apply: a) histologically they are mostly adenocarcinomas b) they arise without exceptions from polyps c) they are almost always discovered indentically d) cause pernicious anemia

a

metastasis calcification can develop in: a) the mucosa of the stomach b) cardiomyocytes c) keloid d) uterine leiomyoma

a

. The risk of tumour occurrence increases with age as a consequence of: a) Accumulation of mutations b) Rarer occurrence of mutations in older cell populations c) Faster cell proliferation in older organism d) Faster metabolism of older cells

a

Choose the statement(s) valid for asbestosis: a) There is fibroproduction in the affected tissues b) One of its short term complications is pleural mesothelioma c) Represents a higher risk for developing lymphoma of the mediastinum d) Asbestos bodies are negative in Pens Prussian blue stain

a

Choose the statement(s) valid for melanoma: a) The prognostically least favourable variant is nodular melanoma b) Residual pigmentation of tumour cells is always retained, or at least it is immunohistochemically demonstrable c) The uveal melanomas metastasize only rarely d) In ovarian melanoma the depth of invasion is determined according to Clark's score

a

Choose the statement(s) valid for pseudomelanosis: a) It is a post modem change b) It is a regressive change c) It is hyperpigmentation of the basal layer of epidermal cells following the hyperplasia of melanocytes d) It develops from regressively changed melanin

a

Congenital aplasia of the thymus is associated with: a) DiGeorge syndrome b) Bruton agamaglobulinemia c) Severe combined immunodeficiency (SCID) d) Chronic granulomatosis

a

Excentric hypertrophy of the left ventricle is caused by: a) Post infarctum aneurysm of the ventricle wall b) Tuberculosis of the lung c) Compensated arterial hypertension d) Bullous emphysema of the lung

a

For oxalate stones the following apply: a) They are found in the urological tract b) They are part of some cancers (eg. papillary thyroid cancer) c) They are very rare d) They are rarely found in atherosclerotic plaques

a

Select the valid statement(s): a. Trypanosoma cruzi belongs among the protozoa b. The causative agent of malaria belongs among the bacteria c. Trichinella spiralis belongs among the treponemes d. The causing agent of borreliosis belongs among viruses

a

Senile amyloid is typically described: A. In the myocardium and in the tongue b. In the kidneys c. Perivascularly in the brain d. In the submucosa of the intestine

a

Tuberculous granuloma: a) May contain acid-fast rods b) Lymphoma frequently develops in its vicinity c) Is surrounded by macrophages (so-called osteoclast-like cells) d) It has the same histological structure as granulomas in sarcoidosis

a

mitochondrial DNA a) codes some subunits of oxidative phosphorylation chain, mitochondrial tRNAs and rRnas b) is controlled by highly effective reparative mechanisms in contrast to nuclear DNA characterized by paucity reparation c) contain relatively higher amount of introns in proportion to exons than nuclear DNA d) codes all subunits of oxidative phosphorylation chain located in inner mitochondrial membrane

a

steatosis is a) pathological accumulation of triacylglycerol or cholesteryl esters in the cytosol of cell types which normally do not store lipids b) induction of adipose tissue in the stroma of various organs c) genetically determined accumulation of lipids in the lysosomal system d) always an acquired disorder

a

the risk of tumor occurence increases the age as a consequence og a) accumulation of mutations b) rarer occurence of mutations in older cell popularions c) faster cell proliferation in older organisms d) faster metabolism of older cells

a

Choose the statements(s) valid for pseudomelanosis: a) it is a post mortem change b) it is a regressive change c) it is hyperpigmentation of the basal layer of epidermal cells following the hyperplasia of melanocytes d) it develops from regressivljy changed melanin

a (A dark greenish or blackish postmortem discoloration of the surface of the abdominal viscera, resulting from the action of sulfureted hydrogen on the iron of disintegrated hemoglobin)

Choose the statements(s) valid for asbestosis a)there is fibroproduction in the affected tissue b) One of its short term complications is pleural mesothelioma c) represents a higher risk for developing lymphoma of the mediastinum d)Asbestos bodies are negative in Pens Prussian blue stain

a (mesothelioma is a rare tumor, can develop in workers exposed to asbestos, increased risk by X1000, it is a very dangerous tumor, and death is usually outcome, no short term complication:) (asbestos bodies stains prussian blue!)

Pus contains: a) lipids b) fibrin c) collagen d) polymorphonuclear leukocytes

a +b+d

What is the origin of the most common amyloid deposit in patients with Hodgkin`s disease a. AA amyloid b. Beta-amyloid c. Beta-2-microglobulin d. AL amyloid e. AE amyloid

a AA - amyloid

Which type of amyloid occurs in Familial Mediterranean fever a. AA amyloid b. Beta-2-microglobulin c. Amyloid derived from transthyretin d. AL amyloid e. Amyloid derived from gelsolin

a AA amyloid

Which type of amyloidosis can arise in the setting of chronic suppurative osteomyelitis a. AA amyloid b. Beta-amyloid c. Beta-2-microglobulin d. AL amyloid e. AE amyloid

a AA amyloid

Which type of amyloid are found in patients with bronchiectasis and chronic inflammation a. AA amyloid b. Beta-amyloid c. Beta-2-microglobulin d. AL amyloid e. AE amyloid

a AA amyloid (remember, it is connected with inflammation)

What is the origin of amyloid deposits in patients with paraneoplastic complications of renal carcinoma a. AA amyloid b. Beta-amyloid c. Beta-2-microglobulin d. AL amyloid e. AE amyloid

a AA-amyloid

Interleukin-1 stimulates the liver to produce precursors of which of the following amyloids a. AA amyloid b. Beta-amyloid c. Beta-2-microglobulin d. AL amyloid e. AE amyloid

a AA-amyloid (aa-amyloid is an acute phase protein, produced during inflammation)

Which of the following diseases don`t belong among lysosomal storage disorders a. Alcaptonuria b. Tay-Sachs disease c. Gaucher disease d. Niemann-Pick disease e. Mucopolysaccharidoses

a Alcaptonuria A rare inherited genetic disorder in which the body cannot process the amino acids phenylalanine and tyrosine.

Which enzyme is defective in Fabry disease a. Alpha-galactosidase b. Hexosaminidase A and B c. Fructokinase d. Beta-galactosidase e. Acid lipase

a Alpha-galactosidase

Which of the following substances accumulates exclusively extracellularly a. Amyloid b. Glycogen c. Lipofuscin d. Fat e. Melanin

a Amyloid

In which of the following processes does the bax gene become expressed a. Apoptosis b. Proliferation c. Cell division d. Inhibition of Ras/Ras inhibition e. Angiogenesis

a Apoptosis

Which of the following changes can be observed in kidneys during longstanding ureteral obstruction a. Atrophy b. Dysplasia c. Hyperplasia d. Hypertrophy e. Metaplasia

a Atrophy

Which of the following changes characterizes the most severe lethal cellular damage a. Autolysis b. Autophagy c. Drop of calcium levels d. Intracellular glycogen accumulation e. Intracellular protein accumulation

a Autolysis

Examples of dystrophic calcification are a. Calcification damaging the aortic valve b. Bone ossification in the process of skeletal growth c. Deposits of calcium in the kidneys during hyperparathyroidism d. Renal concrements e. Saponification in necrosis

a Calcification damaging the aortic valve e. Saponification in necrosis

Which of the following manifestations are typical for apoptosis a. Caspase activation b. Localized inflammatory response c. Phagocytosis of cell fragments by local macrophages d. Chromatin condensation e. Hydrolysis of chromatin by endonucleases

a Caspase activation c Phagocytosis of cell fragments by local macrophages d Chromatin condensation e Hydrolysis of chromatin by endonucleases

Patients with Pompe disease will most likely have a. Cells with cytoplasmic brightening or vacuolization b. Accumulation of hyaline c. Cells with abnormal staining with Prussian blue d. Accumulation of pigment e. Metaplastic cells in some organ

a Cells with cytoplasmic brightening or vacuolization (accumulation of glycogen, which does not stain in HEstain)

Which of the following statements applies to congenital mitochondrial defects a. Clinical triad - Neuropathy, Myopathy, Cardiomyopathy b. Hypoglycemia c. AD heredity d. AR heredity e. Decreased levels of lactate

a Clinical triad - Neuropathy, Myopathy, Cardiomyopathy X-linked heredity Increased lactate levels hyperglycemia

Which of the following types of necroses are characteristic for myocardial infarcts a. Coagulation necrosis b. Liquefactive necrosis c. Caseous necrosis d. Fibrinoid necrosis e. Zenker degeneration/Wax degeneration/Zenker wax necrosis

a Coagulation necrosis

Nothing to do with osteoporosis has: a) Addisons disease b) bone fractures c) low levels of estrogen d) immobilization

a) Addisons disease - low cortisol (and aldosterone) Osteoporosis is a progressive bone disease that is characterized by a decrease in bone mass and density which can lead to an increased risk of fracture

The genetic material of the EBV virus can be found in cells of: a) African Burkitts lymphoma type b) urothelial bladder cancer c) squamous cell carcinoma of uterine cervix d) squamous cell carcinoma of the esophagus

a) African Burkitts lymphoma type

Which of the following is true about bleeding? a) Blood output outside the blood vessel b) External bleeding is not bleeding from a gastric ulcer c) Petechiae are extensive bleeding into the skin surface d) Hematoma indicates only bleeding into internal organs e) Hematemesis means blood in the stool

a) Blood output outside the blood vessel b - to bleed into GI tract is kind of "outside" the body c - petechiae are small bleedings, not extensive d - hematomas can form anywhere e - vomiting blood

A 65-year-old patient lost 15 pounds in the past two months, and feels tired. There is high suspicion of a malignant neoplasm. Which of the following sequences, according to the most common incidence of malignant neoplasms in women, is correct? a) Breast - Lung - Large intestine and rectum b) Brest - Uterus - Lung c) Large intestine and rectum - Lung - Ovarian d) Lung - Breast - Ovarian e) Ovarian - Uterus - Lung

a) Breast - Lung - Large intestine and rectum

Which tumor formation(s) is/are related to EBV? a) Burkitt lymphoma. b) Some B cell non-Hodgkin lymphoma. c) Kaposi`s sarcoma. d) T-lymphoblastic leukemia. e) Nasopharyngeal carcinoma.

a) Burkitt lymphoma. b) Some B cell non-Hodgkin lymphoma. e) Nasopharyngeal carcinoma.

Which malignant lymphomas are NOT accompanied by amyloidosis: a) Burkitts lymphoma b) DLBCL c) multiple myeloma d) plasmocytoma

a) Burkitts lymphoma b) DLBCL - Burkitts lymphoma: a cancer of the lymphatic system, particularly B lymphocytes found in the germinal center -DLBCL: diffuse large B-cell lymphoma is a cancer of B cells, a type of white blood cell responsible for producing antibodies. It is the most common type of non-Hodgkin lymphoma among adults -Multiple myeloma: also known as plasma cell myeloma or Kahler's disease, is a cancer of plasma cells, a type of white blood cell normally responsible for producing antibodies. Amyloidosis is a distant third in the causation -Plasmocytoma: a malignant plasma cell tumor growing within soft tissue or within the axial skeleton

Which is true of inflammation of the mucous membranes: a) Catarrhal inflammation is a common type of inflammation b) Catarrhal inflammation never becomes chronic c) Often forms abscesses d) Chronic inflammation can lead to hypertrophy or atrophy of the mucosa

a) Catarrhal inflammation is a common type of inflammation d) Chronic inflammation can lead to hypertrophy or atrophy of the mucosa -Catarrhal inflammation can become chronic (tydligvis:p) -Does not often form abscesses, but it may probably happen..

Which of the following claims about the nuclear characteristics of the tumor obtained during the flow cytometry examination is valid? a) Diploid lesions may be malignant or benign b) Aneuploid tumors are always malignant c) Malignant tumors can be diploid d) Reactive lesions are usually aneuploid e) Malignant tumors are never diploid

a) Diploid lesions may be malignant or benign c) Malignant tumors can be diploid

Which of the following pairs are correct? a) EBV - Nasopharyngeal carcinoma b) AIDS - Lymphoma, especially T-cell lymphoma c) HPV type 18 - Stomach cancer d) Hepatitis B - Breast cancer e) Herpes simplex type II - Burkitt lymphoma

a) EBV - Nasopharyngeal carcinoma ( Herpes simplex virus type 1 infection activates the Epstein-Barr virus)

Risk factors of atherosclerosis is not: a) HDL b) diabetes mellitus c) smoking d) hypertension

a) HDL

Which is true of autoimmune diseases: a) Hashimoto thyroiditis is an example of pathological type IV immune response b) May be systemic and organ specific c) Systemic forms include Sjögren's syndrome d) Goodpasutre syndrome is an autoimmune disease with antibodies against pneumocytes and podocytes

a) Hashimoto thyroiditis is an example of pathological type IV immune response b) May be systemic and organ specific c) Systemic forms include Sjögren's syndrome -Sjögren syndrome: a chronic autoimmune disease in which the body's white blood cells destroy the exocrine glands, specifically the salivary and lacrimal glands, that produce saliva and tears. -Goodpasutre syndrome: causes the abnormal production of anti-GBM antibodies, by the plasma cells of the blood. The anti-GBM antibodies attack the alveoli and glomeruli basement membranes --> type II

What is characteristic of a measles infection: a) Koplik spots are present b) Microscopic granulomas are present c) Infection in pregnant women frequently induces fetal malformations d) The exanthema has vesiculopostulous pattern

a) Koplik spots are present -Measles: rubeola is an infection of the respiratory system, immune system and skin caused by a virus, specifically a paramyxovirus. -Koplik spots: characterized as clustered, white lesions on the buccal mucosa (on the buccal mucosa opposite the lower 1st & 2nd molars) and are pathognomonic for measles -No blisters, but a spot-like rash - Pregnanty women : is this wrong because it says "frequently?", because it is very dangerous for pregnant womens and their baby

Feature of carcinoma in situ is: a) Limited basement membrane lesions b) potential of metastasis c) invasion into blood vessels d) lack of genetic mutations e) low nuclear-cytoplasmic ratio

a) Limited basement membrane lesions

Which is true for precancerosis: a) We distinguish precancerosis conditions and lesions b) Precancerosis does not include bening tumors c) The example is prostate hyperplasia d) Solar keratosis shall be considered precancerosis

a) We distinguish precancerosis conditions and lesions d) Solar keratosis shall be considered precancerosis -Precancerosis condition: a generalized state associated with an increased risk of cancer -Precancerosis: a condition marked by the presence of one or more precancerous lesions -Solar keratosis: also called actinic keratosis; a pre-cancerous patch of thick, scaly, or crusty skin -Prostate hyperplasia: mostly benign? (BPH)

Malignant tumors generally have: a) a reduced contact inhibition b) elevated intercellular cohesion c) low ratio of nucleus / cytoplasm d) low mitotic activity e) low degree of invasive growth

a) a reduced contact inhibition

Which of the featured states occurs as the complication of cholangitis a) abscesses in the liver b) perisplenitis cartilaginea c) thrombophlebitis of appendicular veins d) ulverophlegmonous appendicitis

a) abscesses in the liver

Which of the following states represents purulent inflammation: a) Abscesses in the liver b) Portal pyemic c) Thrombophlebitis of appendicular veins d) Ulcerphlegmonous appendicitis

a) abscesses in the liver b) portal pyemic c) thrombophlebitis of appendicular veins d) ulcerphlegmonous appendicitis -Hard to find.... -Pyemia: a type of septicaemia that leads to widespread abscesses of a metastatic nature

Which of the following is not an organ specific autoimmune disease? a) addison's disease b) juvenile diabetes mellitus c) hashimoto thyroiditis d) pernicious anemia

a) addison's disease DM --> pancreas Hash --> thyroid gland Anemia --> stomach (parietal cells)

Which of the following type of cancer in children occurs LEAST frequently: a) adenocarcinoma b) leukemia c) lymphoma d) sarcoma e) meduloblastoma

a) adenocarcinoma

Epithelial tumor can take which of the following forms: a) adenocarcinoma b) fascicular c) trabecular d) solid alveolar

a) adenocarcinoma c) trabecular d) solid alveolar -Epithelial tumor: a cluster of irregularly-shaped cells growing specifically on the outer membrane of an organ, gland or body part. It may or may not be cancerous.

Steatosis examples: a) alcoholic liver b) myocardial ischemia c) the proliferation of fat d) lipidosis of the gallbladder

a) alcoholic liver b) myocardial ischemia

Which type of steatosis is caused by mitochondrial disorder? a) alcoholic liver steatosis b) steatosis in myocardial ischemia c) granular cells in CNS d) lipidosis of gallbladder mucosa

a) alcoholic liver steatosis b) steatosis in myocardial ischemia Steatosis (also called fatty change, fatty degeneration or adipose degeneration) is the process describing the abnormal retention of lipids within a cell. It reflects an impairment of the normal processes of synthesis and elimination of triglyceride fat. Excess lipid accumulates in vesicles that displace the cytoplasm. When the vesicles are large enough to distort the nucleus, the condition is known as macrovesicular steatosis; otherwise, the condition is known as microvesicular steatosis.

Which type of amyloid can develop in a patient with long-term rheumatoid arthritis? a) amyloid AA b) AL amyloid c) amyloid AE d) amyloid ARA

a) amyloid AA

Which of the following statements/s is/are true of oncogenes? a) are present in human cells b) are formed by viral DNA c) they code growth factor receptors d) they are present in all patients with malignancies

a) are present in human cells b) are formed by viral DNA c) they code growth factor receptors

Air and gas embolism: a) are the same conditions, there are present air bubbles in the blood b) air embolism occurs when air is sucked into the veins with negative pressure c) in gas embolism the blood oxygen is released mainly in the form of bubbles d) gas embolism occurs as fast release of nitrogen during decompression

a) are the same conditions, there are present air bubbles in the blood b) air embolism occurs when air is sucked into the veins with negative pressure d) gas embolism occurs as fast release of nitrogen during decompression

The calcification applies: a) arises in caseous necrosis b) results from hypercalcemia c) bone metaplasia arises on its bases d) does not affect the vascular wall

a) arises in caseous necrosis b) results from hypercalcemia

Which cell product or cell function pairing is most accurate? a) basophils - histamine b) extravascular erythrocytes - Hemosiderin c) neutrophils - suppurative inflammation d) eosinophils - fungal infection

a) basophils - histamine b) extravascular erythrocytes - Hemosiderin c) neutrophils - suppurative inflammation

Which of the following genes are frequently altered in the process of gene amplification? a) c-erbb-2 b) bcl-2 c) abl d) p53 e) ras

a) c-erbb-2

Chronic heart failure left side: a) causes brown induration of the lung b) causes cardiac hydrops c) the cause of systemic hypertension d) may lead to right heart failure

a) causes brown induration of the lung d) may lead to right heart failure

Testicular germ cell tumors include: a) choriocarcinoma b) sertoli cell tumor c) yolk sac tumor d) seminoma

a) choriocarcinoma c) yolk sac tumor d) seminoma

For which of the following pathologic states is characteristic for reciprocal chromosomal translocation leading to amplification of onogene abl? a) chronic myeloid leukemia b) Barrett oesophagus c) Lymphocytic leukemia d) osteosarcoma e) Burkitt lymphoma

a) chronic myeloid leukemia (burkitt lymphoma=c-myc)

HPV infection does not cause: a) condyloma lata b) c) condyloma acuminatum d) cervical intraepithelial neoplasia second degree (CIN2)

a) condyloma lata b) Cond.lata: a cutaneous condition characterized by wart like lesions on the genitals. They are generally symptoms of the secondary phase of syphilis, caused by the spirochete, Treponema pallidum

Ulcer means: a) deeper tissue surface defect b) the cavity created by the distintegration of tissue c) the inflammatory infiltrate in the tissue d) a synonym of the erosion

a) deeper tissue surface defect

What is true of gangrene: a) it is modified secondary necrosis b) never occur physiologically c) does not endanger young individuals d) is always of an ischemic origin

a) it is modified secondary necrosis b) never occur physiologically

Which of the following is typical of alpha-1 antitrypsin deficiency? a) emphysema panlobular b) centrolobular emphysema c) paraseptal emphysema d) is absent from emphysema

a) emphysema panlobular A genetic disorder that causes defective production of alpha 1-antitrypsin (A1AT), leading to decreased A1AT activity in the blood and lungs, and deposition of excessive abnormal A1AT protein in liver cells. In absence of A1AT, neutrophil elastase is free to break down elastin, which contributes to the elasticity of the lungs, resulting in respiratory complications such as emphysema, or COPD (chronic obstructive pulmonary disease) in adults and cirrhosis in adults or children

An example of hypertrophy is: a) enlargement of the heart in hypertension b) changes of the endomterium in increased levels of estrogen c) nodular enlargement of the thyroid gland d) an inflammatory tumor

a) enlargement of the heart in hypertension Why not b? It's both hypertrophy and hyperplasia

Which of the following tumors show the presence of hematogenous metastasis? a) follicular thyroid carcinoma b) follicular adenoma of the thyroid gland c) medullary thyroid carcinoma d) papillary thyroid carcinoma

a) follicular thyroid carcinoma together with clear cell renal carcinoma, hepatocellular carcinoma and choriocarcinoma. (and sarcomas ofcourse)

Which is true of prostate cancer: a) forms osteplastic metastasis in bones b) forms predominantly ostelytic metastasis c) arises on the basis of adenomyomatous hyperplasia d) starts from the inner part of the prostate

a) forms osteplastic metastasis in bones

For the diagnosis of papillary thyroid carcinoma is required finding: a) ground glass nuclei b) follicles c) metastasis d) papillae

a) ground glass nuclei (órphan annie body)

Which of following symptoms of hyaline thrombi is correct a) has microscopic dimensions b) emboli in pulmonary artery often leads to death c) arises from stagnation of blood in varicose veins d) is a synonym for so-called hyaline droplets e) arises on atherosclerotic plaques in the aorta

a) has microscopic dimensions Color atlas: homogeneous eosinophilic red thrombus in minor vessels such as capillaries, arterioles and venules. Composed of disintegrated thrombocytes and fibrin. These thrombi are hyaline-micro thrombi.

Blood in the sputum is called: a) hemoptysis b) hematemesis c) hematomyelie d) melena

a) hemoptysis What is hematomyelie ? hematomyelie - acute bleeding in the spinal cord due to trauma blow compressed air in the blast, falls from heights, car accident. Predisposing factor is pathologically altered blood vessel from any cause. H. often ends lethally, in other cases, the clinical picture corresponds spinal shock; obv. is part of polytrauma

Which of the following pigments stains with Prussian blue? a) hemosiderin b) hematin c) bilirubin d) hematoidin

a) hemosiderin Hematin - oxidized heme, ferric state does not bind the stain Bilirubin + hematoidin is the same, and they are both non-ferrous

Burkitt's lymphoma is associated with: a) infection of EBV b) infections of Mycobacterium avium c) infections of CMV d) JC virus infection

a) infection of EBV

Which of the following diseases are associated with exposure to asbestos? a) interstitial pulmonary fibrosis b) mesothelioma c) hepatocellular carcinoma d) asthma

a) interstitial pulmonary fibrosis b) mesothelioma Mesothelioma: a rare form of cancer that develops from cells of the mesothelium, the protective lining that covers many of the internal organs of the body. Mesothelioma is most commonly caused by exposure to asbestos. The most common anatomical site for mesothelioma is the pleura (the outer lining of the lungs and internal chest wall), but it can also arise in the peritoneum (the lining of the abdominal cavity), the pericardium (the sac that surrounds the heart), or the tunica vaginalis (a sac that surrounds the testis).

Cellular embolism a) Is a prerequisite for tumor metastasis b) may arise as a result of comminuted bone fractures c) can occur after traumatic fragmentation of the liver d) occurs after blunt trauma of the heart

a) is a prerequisite for tumor metastasis b) may arise as a result of comminuted bone fractures c) can occur after traumatic fragmentation of the liver -Cellular embolism: embolism due to a mass of cells transported from disintegrating tissue

What is true of metastasis: a) is a subsidiary focus of malignant tumor b) there is blood and lymph spreading c) the tumor infiltration into surrounding tissues d) attachment of tumor cells occurs on the surface of serous membranes (ovary metastasize by surface linings)

a) is a subsidiary focus of malignant tumor b) there is blood and lymph spreading d) attachment of tumor cells occurs on the surface of serous membranes (ovary metastasize by surface linings)

Which of the following does not apply to Lyme borelliosis: a) is caused by rickettsia b) affects joints c) a typical symptom is skin erythema migrans d) is transmitted by ticks

a) is caused by rickettsia - Lyme borreliosis: Borrelia burgdorferi, Borrelia afzelii and Borrelia garinii. Transmitted via the bite of an infected tick. Symptoms may include fever, headache, and fatigue. A rash occurs in 70-80%, may or may not appear as the well-publicized bull's-eye (erythema migrans). Left untreated, later symptoms may involve the joints, heart, and central nervous system -Rickettsia: causes e.g typhus

Which of the following is/are not true of gout? a) is not caused by increased levels of uric acid b) affected patients suffer from recurrent attacks of acute arthritis c) the characteristic lesions are gouty tophi d) precipitated urate crystals damage the lysosomal membrane

a) is not caused by increased levels of uric acid Meaning Not not --> It is caused by increased levels of uric acid! ;)

For Krukenberg tumor of the ovary does not apply: (primary tumor somewhere else!!) Malignant tumor in ovary from another place! a) it is a primary ovarian mucinous adenocarcinoma b) it is usually bilateral c) tumor cells are mucin-producing d) in most cases it is a metastasis of gastric caner

a) it is a primary ovarian mucinous adenocarcinoma A Krukenberg tumor refers to a malignancy in the ovary that metastasized from a primary site, classically the gastrointestinal tract, although it can arise in other tissues such as the breast. Gastric adenocarcinoma, especially at the pylorus, is the most common source. Krukenberg tumors are often (over 80%) found in both ovaries, consistent with its metastatic nature

For choriocarcinoma apply: a) it is highly malignant b) it origins in connection with pregnancy c) it affects only females d) it produced hCG

a) it is highly malignant b) it origins in connection with pregnancy d) it produced hCG

which of the following assertion is valid for hamartoma: a) it is usually present from birth b) contains cells from all three germ layers c) predisposes its bearer, malignant (predisposes to malignancy) d) contains metaplastic cell types e) is an example of sebaceous adenoma in tuberous sclerosis

a) it is usually present from birth c) predisposes its bearer, malignant (predisposes to malignancy) e) is an example of sebaceous adenoma in tuberous sclerosis

For paget disease of the breast nipple is typical: a) it occurs simultaneously with invasive ductal carcinoma or with ductal carcinoma in situ b) large light cells with hyperchromatic nuclei in the epidermis c) it early metastasizes to bones and lungs d) it is histologically similar to malignant melanoma

a) it occurs simultaneously with invasive ductal carcinoma or with ductal carcinoma in situ b) large light cells with hyperchromatic nuclei in the epidermis d) it is histologically similar to malignant melanoma Paget disease of the breast (also known as Paget disease of the nipple and mammary Paget disease) is a rare type of cancer involving the skin of the nipple and, usually, the darker circle of skin around it, which is called the areola. Most people with Paget disease of the breast also have one or more tumors inside the same breast. These breast tumors are either ductal carcinoma in situ or invasive breast cancer (1-3).

Meningococal infection cause: a) leptomeningitis b) encephalitiss c) Waldestrom macroglobulinemia d) chronic sepsis

a) leptomeningitis

Which of the following is/are true of types of Hodgkin's malignant lymphoma: a) lymphocyte rich and mixed b) small cell and large cell type c) nodular sclerosis and lymphocyte depleted d) Hodgkin's lymphoma with RS cells and Hodgkin's lymphoma with Hodgkin's cells

a) lymphocyte rich and mixed c) nodular sclerosis and lymphocyte depleted 1. Lymphocyte rich 2. Lymphocyte depleted 3. Mixed cellularity 4. Nodular sclerosis

Which pair is correct: a) lymphocytes - chronic inflammation b) bleeding - hemosiderin c) neutrophils - suppurative inflammation d) eosinophils - fungal infection

a) lymphocytes - chronic inflammation b) bleeding - hemosiderin c) neutrophils - suppurative inflammation

Immunohistochemical markers HMB-45 and Melan A are used in diagnosis of: a) malignant melanoma b) squamous cell carcinoma of the lip c) hodgkins lymphoma d) fibrosarcoma e) breast cancer

a) malignant melanoma

Which statement about the leiomyoma of the uterus is not true? a) malignant transformation is very often b) there is regression in postmenopausal women c) there is abnormal bleeding d) they are often multiple

a) malignant transformation is very often

Which of the following is/are true of choriocarcinoma? a) malignant tumor of trophoblastic structures b) does not affect males c) typical signet ring cells are present d) it is a tumor of the remnants of the dorsal root

a) malignant tumor of trophoblastic structures

Dilatation of the heart: a) may be a consequence of myocarditis b) occurs decompensed arterial hypertension c) occurs at amyloidosis d) occurs at Bowen disease

a) may be a consequence of myocarditis b) occurs decompensed arterial hypertension

autoimmune diseases: a) may be caused by different types of pathological immune responses b) circulating antibodies against its own tissue antigens indicate disease always c) atrophic corporal gastritis is a typical example d) myasthenia gravis is caused by antibodies against actin muscle fiber

a) may be caused by different types of pathological immune responses c) atrophic corporal gastritis is a typical example Myasthenia gravis: due to destruction of postsynaptic ACh receptors --> muscle weakness

Which is true of dysplasia: a) may progress to malignant tumor b) is seen only premalignant lesions c) is irreversible d) is stationary

a) may progress to malignant tumor

The main features of Cushing's syndrome include: a) moon face b) buffalo hump c) hypotension d) striae

a) moon face b) buffalo hump d) striae (from the weight gain, stretching of skin) Cushing's syndrome describes the signs and symptoms associated with prolonged exposure to inappropriately high levels of the hormone cortisol. This can be caused by taking glucocorticoid drugs, or diseases that result in excess cortisol, adrenocorticotropic hormone (ACTH), or CRH levels.

For which of the following tumors is characteristics of "arrayed" cores: a) neurilemom (schwannoma) b) neuroblastoma c) ganglioneuroma d) neurofibroma e) neurofibrosarcoma

a) neurilemom (schwannoma)

Which pair of answers are correct: a) neutrophils - chemotaxis b) erythrocytes - active penetration of the vascular wall c) eosinophils - drug reactions d) macrophages - mast cells arise from macrophages

a) neutrophils - chemotaxis c) eosinophils - drug reactions

Which of the following is the most common type of Hodgkin lymphoma? a) nodular sclerosis b) mixed cellularity c) the form with the depletion of lymphocytes d) the lymphocyte-rich form

a) nodular sclerosis

Schwannoma and *: a) nuclear palisading b) location in the cerebrellopontine angle c) infiltrative growth d) tumor of the peripheral nerve

a) nuclear palisading b) location in the cerebrellopontine angle d) tumor of the peripheral nerve

Which of the following factor does NOT affect the selection process for the most appropriate location for the founding of tumor cell metastasis: a) ploidy of tumor cells b) compatibility of the tumor and tissue where tumor metastasizes c) lymphatic drainage from the places of primary tumor d) venous drainage of the places of primary tumor e) vascular anatomy of the organ where metastases are secondary post

a) ploidy of tumor cells

Which of the following predicated on viral oncogenesis invoke oncoviruses NOT: a) responsible for most human cancers b) some virus proteins being structurally similar growth factors c) Viruses represent some proteins structurally similar to receptors for growth factors d) all homologous oncogenes onkornaviru correspond (structural relatives) gene in normal cells e) some oncogenes are mutated versions onkornaviru structurally related gene normal cells

a) responsible for most human cancers

Leiomyosarcoma and rhabdomyosarcoma is different in that: a) rhabdomyosarcoma is only found in the cytoplasm of striated muscle b) leiomyosarcoma is only found in the uterus and rhabdomyosarcoma is only found in the heart c) in rhabdomyosarcoma we find huge multinucleated cells d) leiomyosarcoma is found predominately in young people e) rhabdomyosarcoma does not have distant metastasis

a) rhabdomyosarcoma is only found in the cytoplasm of striated muscle

Gonococcal infections causes a) septic inflammation b) non-infected inflammation c) only affects the urogenital tract d) may affect joints, meninges and heart valves

a) septic inflammation d) may affect joints, meninges and heart valves

Which of the following assertions are true about choristomas? (really sorry for the translation, but see wikipedia for definition of choristomas): a) terms of normal tissue to isolate territorial place b) example of the pancreas is woven in the stomach wall c) This is a local of the tissue but incorrect wiring d) is an example of adenocarcinoma of the colon e) choristom is a synonym for chodrohamartoma of lung

a) terms of normal tissue to isolate territorial place c) This is a local of the tissue but incorrect wiring

Which of the following statements/s is/are true of radiation? a) the bone marrow and lymphoid system are the most radiosensitive tissues in the body b) a dose of 1000 rad of total body radiation would kill all members of the exposed population c) post-radiation sarcomas may develop after a 20 years interval d) cartilaginous and muscle tissue are relatively radiosensitive

a) the bone marrow and lymphoid system are the most radiosensitive tissues in the body b) a dose of 1000 rad of total body radiation would kill all members of the exposed population c) post-radiation sarcomas may develop after a 20 years interval

Which of the following is/are true of tumor differentiation? a) the degree tumor to which tissue resembles normal tissue b) is high in anaplastic tumors c) differentiated tumors are immature d) more differentiated tumors generally have a worse prognosis than less differentiated tumors

a) the degree tumor to which tissue resembles normal tissue b - low c - mature d -better prognosis

Mantle cell lymphoma is characterized by: a) the expression of cyclin D1 in tumor elements b) infiltration of the digestive tract in the form of Lyme disease c) impairment of the skin in the form of mycosis fungoides d) is a T-cell lymphoma

a) the expression of cyclin D1 in tumor elements Mantle cell lymphoma (MCL) is one of the rarest of the non-Hodgkin's lymphomas (NHLs). MCL is difficult to treat and seldom considered cured. MCL is a subtype of B-cell lymphoma, due to CD5 positive antigen-naive pregerminal center B-cell within the mantle zone that surrounds normal germinal center follicles. MCL cells generally over-express cyclin D1 due to a chromosomal translocation in the DNA.

Which of the following is/are true of primary TB? a) the first reaction to TB mycobacteria is the accumulation of macrophages b) the first reaction is the formation of TB nodules c) the formation of a primary Ghon complex d) the site of primary infection is most commonly seen in the intestines

a) the first reaction to TB mycobacteria is the accumulation of macrophages c) the formation of a primary Ghon complex

Which of the following statements about fat embolism is/are true? a) the most common cause is bruising of subcut. tissue b) tends to accompany multiple fractures of long bones c) apply mainly in the pulmonary bloodstream and CNS d) syndromes appear within one week of trauma

a) the most common cause is bruising of subcut. tissue b) tends to accompany multiple fractures of long bones

Which pair contains a true statement: a) the papilloma virus and cervical carcinomas b) EBV and Burkitt lymphoma malignant c) asbestos and small cell lung carcinoma d) ionizing radiation and colon

a) the papilloma virus and cervical carcinomas b) EBV and Burkitt lymphoma malignant

Sezary syndrome is characterized by: a) the presence of neoplastic T lymphocytes b) the presence of neoplastic B lymphocytes c) lymphadenopathy d) no previous

a) the presence of neoplastic T lymphocytes c) lymphadenopathy A type of cutaneous lymphoma that was first described by Albert Sézary. The affected cells are T-cells that have pathological quantities of mucopolysaccharides. Sézary's disease is sometimes considered a late stage of mycosis fungoides with lymphadenopathy.

Which of the following statements is/are correct: a) the stagnation of blood can cause the formation of a red thrombus b) pancreatic cancer cannot cause a mesenteric veins thrombosis c) DIC can case organ ischemic changes d) other options are not true

a) the stagnation of blood can cause the formation of a red thrombus(aka venous/stasis thrombi) c) DIC can case organ ischemic changes

.

a) the system used for grading of prostate adenocarcinoma

The Gleason system is: a) the system used for grading of prostate adenocarcinoma b) the system used in the grading of renal cell carcinoma c) the depth of invasion of squamous cell carcinoma of the uterine cervix d) the grading system used in CNS tumors

a) the system used for grading of prostate adenocarcinoma

Crohn's disease is: a) the transmural inflammation of the intestine b) granulomatous inflammation associated with suppurative granuloma formation c) associated with the formation of fistula d) granulomatous inflammation associated with non-caseous granuloma formation

a) the transmural inflammation of the intestine c) associated with the formation of fistula d) granulomatous inflammation associated with non-caseous granuloma formation A type of inflammatory bowel disease (IBD) that may affect any part of the gastrointestinal tract from mouth to anus. Crohn's disease is caused by a combination of environmental, immune and bacterial factors in genetically susceptible individuals. It results in a chronic inflammatory disorder, in which the body's immune system attacks the gastrointestinal tract possibly directed at microbial antigens. While Crohn's is an immune related disease, it does not appear to be an autoimmune disease (in that the immune system is not being triggered by the body itself).

For malignant lymphoma the following statements do not apply: a) they are never disseminated b) it is often a diffuse large cell lymphoma c) patients are often children d) follicular lymphoma is an example of T-cell lymphoma

a) they are never disseminated d) follicular lymphoma is an example of T-cell lymphoma Cancerous cells that have the ability to spread are called malignant cells. Our bodies have what is called a lymph system, which runs throughout our bodies. It is composed of lymphoid tissue, vessels, and fluid. Lymphoid tissue contains lymph nodes, which are part of the immune system. The immune system's job is to produce blood cells and protect against harm from invading germs. Cancers that start anywhere in the lymphatic system are lymphomas. Follicular lymphoma is the most common of the indolent non-Hodgkin's lymphomas, and the second-most-common form of non-Hodgkin's lymphomas overall. It is defined as a lymphoma of follicle center B-cells (centrocytes and centroblasts), which has at least a partially follicular pattern

Which of the following is/are true of trombangitis obliterans? a) thrombosis with obliteration of the arteries and veins b) it affects mainly younger women c) pulseless disease d) multiple small aneurysms in small arteries are present

a) thrombosis with obliteration of the arteries and veins

Example of paradoxal emboli a) Thrombus travelled from femoral artery to cerebral arteries b) fat from bone marrow to cerebral arteries c) thrombus released from left atrium and going to renal artery d) thrombus from IVC into hepatic vein e) thrombus from femoral veins into pulmonary artery

a) thrombus travelled from femoral artery to cerebral arteries Paradoxal emboli explanation (very good)! Passage of a clot (thrombus) from a vein to an artery. When clots in veins break off (embolize), they travel first to the right side of the heart and, normally, then to the lungs where they lodge, causing pulmonary embolism. On the other hand, when there is a hole at the septum, either upper chambers of the heart (an atrial septal defect) or lower chamber of the heart (ventricular septal defects), a clot can cross from the right to the left side of the heart, then pass into the arteries as a paradoxical embolism. Once in the arterial circulation, a clot can travel to the brain, block a vessel there, and cause a stroke (cerebrovascular accident).

To mesenchymal tumors does NOT belong: a) urothelial papilloma b) liposarcoma c) leiomyoma of uterus d) rhabdomyosarcoma of heart e) hemangioma of liver

a) urothelial papilloma

Undifferentiated sarcomas usually only express: a) vimentin b) actin c) S 100 protein d) they do not express any marker e) synaptopodin

a) vimentin

An association with EBV infection has been described in the following disease(s): a) Infectious mononucleosis b) burkitt lymphoma c) carcinoma of the tonsil d) carcinoma of the tongue

a, b

Chronic right ventricular failure: a) Can lead to the development of nutmeg liver b) Can lead to so-called cardiac cachexia c) Causes the development of rusty induration in the lungs d) Manifests as asymmetric oedema of lower extremities

a, b

Helicobacter infection is a risk factor for: a. Peptic ulcers b. Stomach cancer c. Chronic pancreatitis d. Choledocholithiasis

a, b

Oxyuriasis: a) Is caused by the pinworm b) Eosinophils are predominant in the inflammatory infiltrate c) Lymphocytes are predominant in the inflammatory infiltrate d) Neutrophils are predominant in the inflammatory infiltrate

a, b

Renal infarction: a) Is wedge-shaped b) Is mostly caused by embolization c) Colliquative necrosis is described d) It is a haemorrhagic infarction

a, b

Select the valid statement(s): a. AA amyloid may occur secondary to chronic osteomyelitis b. AL amyloid may arise on the basis of B-NHL c. Tumor-amyloid can be found in papillary thyroid carcinoma d. Perisplenitis cartilaginea is an example of localized amyloidosis

a, b

Systemic amyloidosis can occur in: a. Rheumatoid arthritis b. Clonal production of lambda or kappa light chains c. Alzheimer's disease d. Medullary thyroid carcinoma

a, b

Which of the following is/are true of pathological immune response type II reactions? a) Occurs in myasthenia gravis b) It can lead to urticaria c) This type of reaction can occur in mismatched transfusions d) Typical examples are immuno-complex diseases (e.g. glomerulonephritis)

a, b

Diseases with local amyloid formation include: a) Insulinoma b) Alzheimer's disease c) Multiple myeloma d) Medullary thyroid adenoma

a, b Medullary thyroid CARCINadenoma is local

Amyloid stains in: a) Congo red b) Trichrome c) Thioflavin T d) Sudan red

a, b, c

Choose the chemical mediator(s) that play a role in acute inflammation: a) Complement b) Histamine c) Bradykinines d) Lymphokines

a, b, c

Choose the disease(s) frequently caused by candida infection: a) Thrush b) lntertrigo c) Vaginitis d) Epiglottitis

a, b, c

Choose the statement(s) valid for a patient with septic shock: a) The pulp of the spleen is activated b) It may be the cause of acute fatty liver c) Can lead to the development of anasarca d) The patient has a positive nitrogen balance

a, b, c

Choose the statement(s) valid for herpes zoster virus: a) Is the causative agent of Varicella b) is the causative agent of shingles c) Can lead to conjuctival inflammation d) Causes the development of vesicles on the genitals and in the oral cavity

a, b, c

Choose the statement(s) valid for molluscum contagiosum: a) It is a viral infection b) The skin is affected c) It is more frequent in children d) It is a precancerosis

a, b, c

During the degradation of haemoglobin the iron: a) Is transported into the liver b) Is transported into the spleen c) Is transported into the bone marrow d) The transport is mediated by macrophages

a, b, c

In chronic inflammatory infiltrate we describe: a) Macrophages b) Lymphocytes c) Dendritic cells d) Polymorphonuclear leukocytes

a, b, c

Inflammation can consists of the following processes: a) Alteration b) Exudation c) Proliferation d) Defence

a, b, c

Which of the following infectious agents cause liver abscesses a. Entamoeba histolytica b. Giardia lamblia c. Trichinella spiralis d. Schistosoma hematobium e. Trichophyton

a. Entamoeba histolytica

Which of the following infectious agents cause ulcerations of the large intestine a. Entamoeba histolytica b. Giardia lamblia c. Trichinella spiralis d. Schistosoma hematobium e. Mycobacterium leprae

a. Entamoeba histolytica

K onemocněním způsobeným druhem Haemophilus patří a. Epiglottitis in children b. Otitis media c. Flu d. Pertussis e. Moniliasis

a. Epiglottitis in children b. Otitis media

Adenocarcinoma's origin is: a. Epithelial b. Of lymphoid tissue c. Mesenchymal d. Of nerve bundles e. Endothelial

a. Epithelial

Chronic low-grade chemical damage is important in the genesis of: a. Esophageal cancer b. Leiomyosarcoma c. Retinoblastoma d. Malignant melanoma e. Prostate cancer

a. Esophageal cancer

Which of the following are true about grading? a. Expresses the degree of maturity of malignant tumor b. It takes into account only the cytological criteria c. It takes into account the cytological and structural criteria d. Relies on tumor size e. Determines the degree of expansion of metastases in the body

a. Expresses the degree of maturity of malignant tumor c. It takes into account the cytological and structural criteria

Deficiency of folic acid is most often seen in patients with which of the following infections: a. Giardia lamblia b. Escherichia coli c. Treponema pallidum d. Neisseria gonorrhea e. Staphylococcus aureus

a. Giardia lamblia

For which of the following diseases are Donovan bodies pathognomonic: a. Granuloma inguinale (klebsiella granulomatis) b. Syphilis c. Chancroid d. Gonorrhea e. Lymphogranuloma venereum

a. Granuloma inguinale (klebsiella granulomatis) Granuloma inguinale is a bacterial disease caused by Klebsiella granulomatis characterized by ulcerative genital lesions. It is endemic in many less developed regions. It is also known as donovanosis. They appear deep purple when stained with Wright's stain. These intracellular inclusions are the encapsulated Gram-negative rods of the causative organisms

Which of the following infection is associated with Kaposi's sarcoma? a. Human herpesvirus 8 (HHV-8) b. Herpes simplex virus 2 (HSV-2) c. Aspergillus d. Treponema pallidum e. Staphylococcus aureus

a. Human herpesvirus 8 (HHV-8) A systemic disease that can present with cutaneous lesions with or without internal involvement

Which of the following statements about parotitis are correct: a. Humans are the only natural host b. It often occurs in the infant period c. Very often it develops into orchitis resulting in sterility d. One can find intracellular inclusions in infected acinar cells of the parotid glands e. Prevention is achieved by dead viral mumps vaccine

a. Humans are the only natural host -Parotitis is an inflammation of one or both parotid glands -Acute bacterial parotitis: is most often caused by a bacterial infection of Staphylococcus aureus but may be caused by any commensal bacteria. - Parotitis as Extrapulmonary Tuberculosis: The mycobacterium that cause tuberculosis can also cause parotid infection. - Acute viral parotitis (mumps): the most common viral cause of parotitis is mumps. Routine vaccinations have dropped the incidence of mumps to a very low level

Contago disease (polyserositis) are based on a. Hyaline alteration of connective tissue b. Fibrinoid alteration of connective tissue c. Myxoid alteration of connective tissue d. Transparent alteration of connective tissue e. Storage disease

a. Hyaline alteration of connective tissue

Curshmann disease (polyserositis) are based on a. Hyaline alteration of connective tissue b. Fibrinoid alteration of connective tissue c. Myxoid alteration of connective tissue d. Transparent alteration of connective tissue e. Storage disease

a. Hyaline alteration of connective tissue

Which of these are irreverisble changes of cells? Apoptosis Karyorrhexis Karyolysis Lysosomal rupture

all

Hematoidin a) this pigment is exogenous b) it has a similar structure as bilirubin c) perls prussian blue is positive d) is almost always stored extracellularly

b (A pigment derived from hemoglobin that contains no iron but is closely related to or identical to bilirubin. Hematoidin is formed intracellularly, presumably within reticuloendothelial cells, but is often found extracellularly after 5-7 days in foci of previous hemorrhage. It occurs as refractile, yellow-brown and orange-red granules, but more characteristically as rhomboid plates arranged in a radial pattern, so-called hematoidin burrs.)

Select the valid statement(s): a. The causative agent of Ascariasis belongs among yeasts b. The causative agent of condulomata acuminata belong to viruses c. The causative agent of leprosy is a virus d. Naegleria fowleri belongs among yeasts

b (causative agent for leprosy is mycobacteria leprae)

Choose the possible source(s) of pulmonary embolism a) Superficial veins of the lower limbs b) paraprostatic venal plexus c) left auricle og the heart d) mural thrombus of a chromic aneurysm of the left ventricle

b (emphasis on possible, cause the main source is thrombi in deep leg)

Which of the following associations are correct a. Chronic inflammation - AL amyloidosis b. Aggregations of intermediate filaments in hepatocytes - Alcoholic hyaline c. Hyaline - Russel bodies in plasma cells d. Lipofuscin - Berlin blue dye e. Calcium deposits in necrotic tissue - Metastatic calcification

b Aggregations of intermediate filaments in hepatocytes - Alcoholic hyaline

What is the cause of amyloid deposits in the brain of patients with trisomy 21 a. AA amyloid b. Beta-amyloid c. Amyloid derived from transthyretin d. AL amyloid e. AE amyloid

b Beta amyloid (same amyloid as in Alzheimers disease, therefor the downs syndrome kids have a greater chance of getting alzheimers)

Which of the following cause amyloid deposits in the brain of patients with Alzheimer`s disease a. AA amyloid b. Beta-amyloid protein c. Amyloid derived from transthyretin d. AL amyloid e. AE amyloid

b Beta amyloid protein

Patients with Fabry disease usually die from a. Anemia b. Cardiovasculars injury c. Liver failure d. Respiratory failure e. Non-Hodgkins lymphoma of the B line

b Cardiovascular injury

What is the usual finding in the liver in a patient with chronic right-sided heart failure? - a) Acute inflammation - b) Congestion - c) Edema - d) Hemorrhage - e) Infarct

b Congestion

Hyaline transformation of connective tissue are found physiologically in a. Ganglions b. Corpus albicans c. Fibrinoid necrosis d. Cornea e. Corpora amylacea

b Corpus albicans

We use hybrid techniques (Southern blot or in-situ hybridization) to diagnose all of the following conditions, except a. Identification of viral DNA in specimens b. Diagnosis of different types of anemia c. Classification of B lymphomas d. Differentiation of ovarian carcinomas from breast carcinoma e. Classification of T lymphomas

b Diagnosis of different types of anemia

Early reversible phenomena, which occurs during potential lethal hypoxic cellular injury is/are a. Flocculant/precipitated densities in the mitochondria (Flocculant - a reagent added to a dispersion of solids in a liquid to bring together the fine particles to form flocs) b. Evaginations of the plasma membrane (so-called blebs) c. Release of lysosomal enzymes d. Loss of nuclear basophilia e. Autolysis

b Evagation of the plasma membrane - blebs

All of the following substances are examples of exogenous and endogenous antioxidants except a. Catalase b. Hydrogen peroxide c. Glutathione d. Vitamin E e. Vitamin C

b Hydrogen peroxide Is a part of the whole formation of ROS, so it can't be an antoxidant!

Deposits of amyloid with beta-2-microglobulins we typically find a. In the heart b. In the synovia and tendons c. In skeletal muscle d. In parenchymal organs e. In the leptomeninges

b In the synovia and tendons (complication of patients on long term dialysis, can not be filtered by the dialysis machine)

Which of the following causes does not cause edema a)increased vascular permeability b)increased plasma oncotic pressure c)heart failure d)lymphatic obstruction e) vein obstruction

b Increased plasma oncotic pressure

What is true of bleeding? - a) External bleeding must always be seen externally - b) Internal bleeding may be accompanied by jaundice - c) Only external bleeding leads to jaundice after hemorrhagia - d) Always presume there is a disorder of coagulation factors - E) Manifested with blood loss greater than 5 liters

b Internal bleeding may be accompanied by jaundice

Which of the following statements about fusca atrophy (atrophy with lipofuscin deposition) are correct a. It is a type of numerical atrophy b. It is a type of simple atrophy c. It`s characteristic for CNS d. It occurs during hemosiderin accumulation in tissue e. None of the answers are correct

b It is a type of simple atrophy

What applies to secondary amyloidosis a. AL is a precursor b. It occurs mainly in the liver, spleen, kidneys, thyroid gland and intestine c. It can lead to kidney failure and death of the patient d. It's commonly connected with Alzheimer`s dementia e. One of the protein precursors is transthyretin

b It occurs mainly in the liver, spleen, kidneys, thyroid gland and intestine c It can lead to kidney failure and death of the patient

Calcinosis - Examples a. Dystrophic calcification b. Metastatic calcification c. Calciphylaxis d. Disease arising always during hypercalcemia e. It`s synononymous with spurs (calcification) of the Achilles tendon

b Metastatic calcification calcinosis: formation of calcium deposits in any soft tissue Wikipedia says dystrophic calcification as well..

Primary hemostatic plug is formed by: a) Endothelium b) Platelets c) Emboli d) Fibrin e) Plasma coagulation factors

b Platelets

Dystrophic calcification is most commonly associated with a. Hypercalcemia due to pathology of the parathyroid glands b. Regional healing after previous cell necrosis c. Increasing accumulations of pigment d. Injury of the renal tubules and stomach e. Increase of the working load of injured tissue as a result of successful adaptation

b Regional healing after previous cell necrosis

Which of the following changes causes loss of nuclear chromatin staining: a) Atrophy b) Hyaline change c) Apoptosis d) lschemia

d

Which of the following statements is/are correct for p53? a) It is a protooncogene b) Belongs to the tumor suppressor genes c) Its mutation leads to increased production of growth factors receptors d) Its mutation inhibits apoptosis of cells with damaged DNA

b) Belongs to the tumor suppressor genes d) Its mutation inhibits apoptosis of cells with damaged DNA -p53: a protein that in humans is encoded by the TP53 gene. The p53 protein is crucial in multicellular organisms, where it regulates the cell cycle and, thus, functions as a tumor suppressor, preventing cancer. It can initiate apoptosis - programmed cell death - if DNA damage proves to be irreparable

Marker for malignant B lymphoma is: a) CD 30 b) CD 20 c) CD 3 d) CD 99 e) CD 1

b) CD 20

General marker of T lymphocyte is: a) CD 20 b) CD 3 c) CD 4 d) CD 10 e) CD 19

b) CD 3

Thrombosis: a) Is the only source of embolism b) Can be transformed into a phlebolith c) Always closes the vessel lumen d) Does not occur in the bloodstream

b) Can be transformed into a phlebolith -Thrombosis: the formation of a blood clot inside a blood vessel, obstructing the flow of blood. When a blood vessel is injured, the body uses platelets (thrombocytes) and fibrin to form a blood clot to prevent blood loss. Even when a blood vessel is not injured, blood clots may form in the body under certain conditions. A clot that breaks free and begins to travel around the body is known as an embolus. -Phlebolith: a small local calcification within a vein. These are very common in the veins of the lower part of the pelvis, and they are generally of no clinical importance. - Thrombosis can be a sign of phleboliths. Blood flow can slow when this happens, but you may not notice it on your own.

Which of the following condition(s) is/are predisposing to hepatocellular carcinoma? a) Acute viral hepatitis b) Chronic hepatitis B c) Chronic congestion of liver d) Pre-existing hepatic adenoma e) Cholestasis

b) Chronic hepatitis B

Platelet aggregation at the site of damage to blood vessels is induced by what? a) Activation of Hageman factor b) Contact of platelets to collagen subendothelium c) lysosomal enzymes from neutrophil d) release of tissue thromboplastin e) thrombi

b) Contact of platelets to collagen subendothelium

Hyperplasia: a) Is the opposite of simple atrophy b) Does not affect neurons in the CNS c) Cannot be combined with hypertrophy c) Regularly precedes dysplasia

b) Does not affect neurons in the CNS simple atrophy is by reduction in cell size, hyperplasia is often combined with hypertrophy, and I would say metaplasia, not dysplasia, often precedes dysplasia

The most frequent etiologic agents of meningitis in neonates is? a) Toxoplasmosis b) E. coli c) meningococcus d) amoebae

b) E. coli And group B strep

Each of the following is related to the expression of cancer except: a) Molecular genetic changes b) Ectopic secretion of hormones c) Clonal evolution d) Tumor heterogeneity e) Karyotype changes

b) Ectopic secretion of hormones

Belongs among mixed tumors: a) Chondroid hamartoma b) Fibroadenoma c) Carcinosarcoma d) Mixed pigmented nevus e) Basalioma

b) Fibroadenoma

The prostate cancer is graded according to: a) Breslow b) Gleason c) Paget d) Lauren

b) Gleason The Gleason Grading system is used to help evaluate the prognosis of men with prostate cancer. Together with other parameters, it is incorporated into a strategy of prostate cancer staging which predicts prognosis and helps guide therapy. A Gleason score is given to prostate cancer based upon its microscopic appearance. Cancers with a higher Gleason score are more aggressive and have a worse prognosis

Wet gangrene: a) Is always the result of injuries b) Is caused by putrefactive bacteria c) Never affects internal organs d) Always precedes dry gangrene

b) Is caused by putrefactive bacteria -Gangrene: arises when a considerable mass of body tissue dies (necrosis). May occur after an injury or infection, or in people with chronic health problem affecting blood circulation. Primary cause is reduced blood supply to the affected tissues, which results in cell death -Dry: begins at the distal part of limb due to ischemia, and often occurs in the toes and feet due to arteriosclerosis (known as senile gangrene). Mainly due to arterial occlusion and there is limited putrefaction. -Wet: occurs in naturally moist tissue and organs (mouth, bowel, lungs, cervix, and vulva). Bedsores are also wet gangrene infections. Wet gangrene is coagulative necrosis progressing to liquefactive necrosis. -Putrefaction: 1/7 stages in the decomposition of a dead body. It can be viewed as the decomposition of proteins in a process that results in the eventual breakdown of cohesion between tissues and the liquefaction of most organs. Caused due to bacterial or fungal decomposition of organic matter and results in production of noxious odours.

To which organ usually metastasize osteosarcomas? a) Liver b) Lung c) Other bones d) Lymphatic nodes e) Spleen

b) Lung

Which of the following apply to cancer designations (betegnelse)? a) Malignant epithelial tumors are called ... b) Malignant mesenchymal tumors are designated as sarcomas c) Malignant neurogenic sarcomas are of neuroectodermal origin d) Benign tumors of melanocytes are called nevi

b) Malignant mesenchymal tumors are designated as sarcomas d) Benign tumors of melanocytes are called nevi -Nevi: medical term for sharply circumscribed and chronic lesions of the skin or mucosa, commonly named birthmarks or beauty marks, are benign by definition but 25% of malignant melanomas arise from pre-existing nevi -Carcinoma is the term used for a malignant epithelial tumor

What is true about fat emboli? a) Occurs most frequently after blunt abdominal trauma b) May occur as a brain embolism c) Occurs most frequently in long bone fractures d) Do not lead to severe symptoms e) Can occur following the ingestion of fatty food

b) May occur as a brain embolism c) Occurs most frequently in long bone fractures

From which tissue does rhabdomyoma origin? a) Epithelium b) Mesenchymal tissue c) Lymphoid tissue d) Mixed tissue e) Endothelium

b) Mesenchymal tissue

Waterhouse-Friderichsen syndrome is characterized by: a) hypertension b) hypotension c) diabetes mellitus d) massive bleeding into the adrenals

b) hypotension d) massive bleeding into the adrenals Waterhouse-Friderichsen syndrome (WFS) or hemorrhagic adrenalitis is a disease of the adrenal glands most commonly caused by the bacterium Neisseria meningitidis. The infection leads to massive hemorrhage into one or (usually) both adrenal glands

Which statement is correct about tumors? a) Tumors arising from neuroendocrine cells are carcinoids b) Neurilemmoma arises from Schwann cells c) Identification of adenomas is based on their structure d) Names of adenomas are derived from the name of their source body organs

b) Neurilemmoma arises from Schwann cells c) Identification of adenomas is based on their structure -Carcinoids:a slow-growing type of neuroendocrine tumor originating in the cells of the neuroendocrine system OLD TERM, no we use NET1,NET2,NEC -Neurilemmoma: benign, encapsulated tumors of the nerve sheath (from schwann cells of nerual crest) A: They're only called carcinoids if they're in the small intestine or lung I think. So all carcinoids are neuroendocrine, but not all neuroendocrine are carcinoid

Autocrine growth factor homologous with c-sis oncogene is: a) TGF-beta b) PDGF c) TGF-alpha d) IGF e) EGF

b) PDGF

Mycosis fungoides is characterized by the presence of: a) Munro microabscesses b) Pautrier microabscesses c) Burkitt abscesses

b) Pautrier microabscesses Mycosis fungoides, also known as Alibert-Bazin syndrome[1] or granuloma fungoides, is the most common form of cutaneous T-cell lymphoma. It generally affects the skin, but may progress internally over time. Symptoms include rash, tumors, skin lesions, and itchy skin

Which of the following mediators between oncogenes belongs: a) p53 b) Ras c) RB-1 d) BRCA-1 e) WT-1

b) Ras

The complex of rheumatic fever does not include: a) pericarditis b) Raynaud phenomenon c) migratory polyarteritis d) chorea minor

b) Raynaud phenomenon Raynaud's phenomenon is excessively reduced blood flow in response to cold or emotional stress, causing discoloration of the fingers, toes, and occasionally other areas Sydenham's chorea or chorea minor (historically referred to as Saint Vitus Dance) is a disorder characterized by rapid, uncoordinated jerking movements primarily affecting the face, hands and feet. Sydenham's chorea (SC) results from childhood infection with Group A beta-hemolytic Streptococcus and is reported to occur in 20-30% of patients with acute rheumatic fever (ARF)

Tumors of the peripheral nervous system usually express: a) desmin b) S 100 protein c) melan A d) CD 117 e) CD 3

b) S 100 protein

Hypostasis is a) postmorten diffusion of liquids b) Overfilling of the lower part of an organ by blood c) post mortem autodigestion of the lower part of an organ d) slowdown of blood flow in capillaries in the inflammatory focus e) post mortem clots in the lower part of an organ

b) The settling of blood in the lower part of an organ or the body as a result of decreased blood flow) (directly translated: "overflow part of the organ below the stored blood")

Which of the following is true about thrombus? a) Thrombus formation is an unchangeable structure b) Thrombus composition is variable, but it always contains fibrin and platelets c) So-called white thrombus contains only white blood cells d) Organization of thrombus leads to its fibrous change/remodulation e) Never affecting limb vessels

b) Thrombus composition is variable, but it always contains fibrin and platelets d) Organization of thrombus leads to its fibrous change/remodulation

Repair and regeneration: a) tumor necrosis is one of the regenerative processes b) a primitive bony callus is an example of the reparative process c) postencephalomalatic pseudocyst wall is formed of the connective tissue d) a typical manifestation of regeneration is the formation of nodes in cirrhotic liver

b) a primitive bony callus is an example of the reparative process

Which of the following tumors is most likely the primary source of metastasis to the liver? a) carcinoma in situ of the cervix b) adenocarcinoma of the cecum c) testicular germinal tumor d) pleomorphic adenoma of the parotid gland

b) adenocarcinoma of the cecum

Nasal polyps are; a) viral origin b) allergic origin c) result of long-term smoking d) the effects of chronic rhinitis

b) allergic origin d) the effects of chronic rhinitis Nasal polyps are soft, painless, noncancerous growths on the lining of your nasal passages or sinuses. They hang down like teardrops or grapes. They result from chronic inflammation due to asthma, recurring infection, allergies, drug sensitivity or certain immune disorders.

Gliomas of the brain include: a) neuroblastoma b) anaplastic astrocytoma c) oligodendroglioma d) ependyoma

b) anaplastic astrocytoma c) oligodendroglioma d) ependyoma A glioma is a type of tumor that starts in the brain or spine. It is called a glioma because it arises from glial cells. The most common site of gliomas is the brain. Gliomas are named according to the specific type of cell they share histological features with, but not necessarily originate from. The main types of gliomas are: - Ependymomas—ependymal cells. - Astrocytomas—astrocytes (glioblastoma multiforme is a malignant astrocytoma and the most common primary brain tumor among adults). - Oligodendrogliomas—oligodendrocytes. - Brainstem glioma — develop in the brain stem - Optic nerve glioma — develop in or around the optic nerve Neuroblastoma (NB) is the most common extracranial solid cancer in childhood and the most common cancer in infancy. It is a neuroendocrine tumor, arising from any neural crest element of the sympathetic nervous system (SNS)

What is true of pigments: a) they are soluble substances which lead to the diffuse cell coloring b) are exogenous and endogenous c) the occurrence of endogenous pigments may participate in macrophages d) iron oxide pigments are only exogenous origin

b) are exogenous and endogenous c) the occurrence of endogenous pigments may

Which of the following is/are true of pigments: a) they are soluble substances which lead to diffuse cell coloring b) are exogenous and endogenous c) the occurrence of endogenous pigments may be present in macrophages d) non-oxide pigments are of exogenous origin only

b) are exogenous and endogenous c) the occurrence of endogenous pigments may be present in macrophages

What is true about adenomas? a) are tumors from superficial epithelium b) are tumors of glandular epithelium c) are mixed mesenchymal-epithelial tumors d) designate only to neoplasms originating from endocrine glands e) arise only in the digestive tract

b) are tumors of glandular epithelium

What is true about papillomas? a) the basic structure of the papilloma consist of epithelial tissue in the center surrounded by tissue b) arise from surface epithelium c) has a papillary structure in the center of the lumen d) a typical example is verruca vulgaris e) occur in the urinary bladder

b) arise from surface epithelium d) a typical example is verruca vulgaris e) occur in the urinary bladder

Granulomas are not a typical morphological manifestation of: a) tuberculosis b) asbestosis c) sarcoidosis d) histoplasmosis

b) asbestosis -Histoplasmosis caused by the fungus Histoplasma capsulatum. Symptoms of this infection vary greatly, but the disease affects primarily the lungs. Granulomas are seen in most forms of histoplasmosis (acute histoplasmosis, histoplasmoma, chronic histoplasmosis). -Sarcoidosis: a disease involving abnormal collections of inflammatory cells (granulomas) that can form as nodules in multiple organs. The granulomas are most often located in the lungs or its associated lymph nodes, but any organ can be affected. Sarcoidosis seems to be caused by an immune reaction to an infection or some other trigger

Benign tumors: a) the name is always the name of the tissue with the suffix -oma b) benign epithelial tumors are usually adenomas and papillomas c) benign tumors usually exhibit expansive growth profile d) may sometimes threaten the patients life

b) benign epithelial tumors are usually adenomas and papillomas c) benign tumors usually exhibit expansive growth profile d) may sometimes threaten the patients life

Which of the following events is the fourth in chronological order? a) bronchial carcinoma b) brain metastasis c) smoking 40 cigarettes per day d) airway epithelium metaplasia

b) brain metastasis

Follicular thyroid carcinoma is characterized by the presence of: a) a) b) b) capsular invasion c) c) Orphan-Annia eye nuclei d) d) psammoma bodies

b) capsular invasion - Psammoma bodies: meningiomas and papillary thyroid cancer - Orphan Annie eye nuclei: papillary thyroid cancer

Bowen disease is: a) the proliferation of squamous epithelium in the sites of chronic irritation b) carcinoma in situ c) benign tumor of the sweat glands d) pseudolymphoma of the skin

b) carcinoma in situ Bowen's disease (BD) (also known as "squamous cell carcinoma in situ") is a neoplastic skin disease; it can be considered as an early stage or intraepidermal form of squamous cell carcinoma

In neuroectodermal tumors and carcinomas, we can find: a) actin b) chromogranin c) desmin d) LCA(Leukocyte Common Antigen) e) vimentin

b) chromogranin

Which of the following patterns is/are characteristic of necrosis caused by denaturation? a) caseous b) coagulative c) gangrenous d) liquefactive

b) coagulative No destruction of cells, because the enzymes are not working. Keeps the architecture of the tissue

Which of the following is/are not false tumor? a) cysts b) cystadenomas c) inflammatory tumor d) precancerosis

b) cystadenomas d) precancerosis

Subdural hematoma is usually caused by: a) rupture of the medial meningeal artery b) damage to bridging veins c) rupture of the ... artery d) damage of cavernous sinus

b) damage to bridging veins

Cor pulmonale is characterized by all following symptoms EXCEPT a) distended jugular vein b) decreased pulmonary vascular resistance c) nutmeg liver d) edema, depending on body position e) chronic lung disease

b) decreased pulmonary vascular resistance

Grading is a term indicating: a) degree of spread of tumor in the body b) degree of tumor differentiation c) the location of the tumor in the body d) the severity of clinical symptoms

b) degree of tumor differentiation

In germinal tumors belongs: a) dermoid cyst of skin b) dermoid cyst of ovary c) embryonic carcinoma of testis d) seminoma e) dysgerminoma of ovary

b) dermoid cyst of ovary c) embryonic carcinoma of testis d) seminoma e) dysgerminoma of ovary

Cytological examination of cervical smear is not used for: a) detection of sexually transmitted diseases b) diagnosis menstrual cycles disorders c) detection of precancerous changes and malignancy of cervix uteri d) detection of endometrial cancer

b) diagnosis menstrual cycles disorders

Which of the following substances is considered a tumor promoter? a) plasminogen activator b) estrogen c) p53 d) radiation e) ras oncogene

b) estrogen d) radiation

In advanced liver cirrhosis there is not present: a) spider nevi b) glaucoma c) rupture of esophageal varices d) ascites

b) glaucoma - Cirrhosis is a slowly progressing disease in which healthy liver tissue is replaced with scar tissue, eventually preventing the liver from functioning properly. The scar tissue blocks the flow of blood through the liver and slows the processing of nutrients, hormones, drugs, and naturally produced toxins. - A spider angioma (spider nevus) is a type of telangiectasis (swollen blood vessels) found slightly beneath the skin surface, often containing a central red spot and reddish extensions which radiate outwards like a spider's web. They are common, however, having more than 3 spider angiomas is likely to be abnormal and may be a sign of liver disease

Which of the following is/are the most common CNS tumors in adults? a) ganglioglioma b) glioblastoma c) neuroblastoma d)

b) glioblastoma

Which of the following associations are not correct? a) meingioma - arachnoidea b) glioblastoma - Schwann cells c) medulloblastoma - rosette arrangement d) m. Recklinghausen - acustic neurinoma

b) glioblastoma - Schwann cells Glioglastoma = Astrocytes

Biopsies form the base of a skin ulceration showed proliferation of small blood vessels and the formation of fine collagen fibers. It is typical for: a) basalioma b) granulation tissue c) pemphigus vulgaris d) the radial growth phase of malignant melanoma

b) granulation tissue

Select the INCORRECT answer. In the etiology of carcinogenesis: a) It can play an important role in food ingredients b) have various tissues / cells are roughly comparable sensitivity c) Are certain jobs associated with exposure to risk factors d) consumption is unambiguously aflatoxin risk fakor e) hormones may play a role in tumor promoter

b) have various tissues / cells are roughly comparable sensitivity

Which statement is true for hernia? a) are always obtained b) hemorrhagic infarct may occur c) may originate in the scar d) always contains intestinal loops

b) hemorrhagic infarct may occur c) may originate in the scar

lymphogenic metastasis occur: a) only in carcinomas b) in most carcinomas c) in all tumors d) in mesenchymal tumors as often as in carcinomas

b) in most carcinomas

Herpes zoster: a) is caused by a measle virus b) is caused by the same virus strain as varicella c) is caused by herpes simplex virus d) latent virus survives in the spinal ganglia

b) is caused by the same virus strain as varicella d) latent virus survives in the spinal ganglia -Herpes zoster: commonly known as shingles is a viral disease characterized by a painful skin rash with blisters in a limited area on one side of the body (left or right), often in a stripe. The initial infection with varicella zoster virus (VZV) causes the acute, short-lived illness chickenpox which generally occurs in children and young adults. -Herpes zoster is not the same disease as herpes simplex -After the initial episode of chickenpox resolves, the varicella zoster virus remains latent in the nerve cell bodies and, less frequently, the non-neuronal satellite cells of the dorsal root, cranial nerve or autonomic ganglia.

Which of the following is/are true of shock? a) is a metabolic disruption caused by tissue necrosis b) it is a consequence of tissue hypoperfusion c) shock causes changes which can be reversible d) is always associated with DIC

b) it is a consequence of tissue hypoperfusion c) shock causes changes which can be reversible (in the beginning atleast)

Which statement is true of necrosis: a) tissue ischemia is a synonym for tissue necrosis b) it is a morphological manifestation of intravital tissue death c) occurs only posthumously d) necrosis is always caused by endogenous reasons

b) it is a morphological manifestation of intravital tissue death

Gynecomastia is associated with: a) seminoma b) it is commonly bilateral c) turner syndrome d) liver cirrhosis

b) it is commonly bilateral d) liver cirrhosis What is Gynectomastia: Gynecomastia is a common endocrine disorder in which there is a benign enlargement of breast tissue in males. Gynecomastia is thought to be caused by an altered ratio of estrogens to androgens mediated by an increase in estrogen production, a decrease in androgen production, or a combination of these two factors In individuals with liver failure or cirrhosis, the liver's ability to properly metabolize hormones such as estrogen may be impaired.

What is true of thrombosis: a) it is intravital blood clotting intra and extravascular b) it is intravital blood clotting in the cardiovascular system c) for thrombi formation is always necessary multiplication of platelets d) the organization of thrombi may lead to their recanalization

b) it is intravital blood clotting in the cardiovascular system d) the organization of thrombi may lead to their recanalization

Which of the following is/are not characteristic of prostate hyperplasia? a) the periurethral (central) localization b) it represents a premalignant status c) urinary tract infections are common d) hypertrophy, ev. dilatation of the bladder wall is present

b) it represents a premalignant status

Which of the following is/are NOT true of benign prostatic hyperplasia? a) begins in the periurethral zone b) it leads to a narrowing of the ureter c) incidence increases with age d) is induced by hormonal factors

b) its leads to a narrowing of the ureter NOT URETHRA!! ;)

Melanin a) arises in ... b) its production is controlled indirectly by adrenal hormones (POMC) c) there are two basic types oculocutaneous melanin and neuromelanin d) albinism occurs only in animals, not in human

b) its production is controlled indirectly by adrenal hormones (POMC) There are three basic types of melanin: eumelanin, pheomelanin, and neuromelanin. The most common type of melanin is eumelanin.

The secondary systemic AA amyloidosis patient usually dies due to: a) malabsorption syndrome b) kidney failure c) in hepatic coma d) for the symptoms of advanced dementia

b) kidney failure

Which of the following organs are usually affected by metastasis of osteosarcoma? a) liver b) lungs c) other bones d) lymph nodes

b) lungs

Basalioma is in general: a) highly malignant b) malignant c) potentially malignant d) benign

b) malignant -Basalioma: a skin cancer, is the most common cancer. It rarely metastasizes or kills. However, because it can cause significant destruction and disfigurement by invading surrounding tissues, it is still considered malignant

Complication of peptic duodenal ulcers does not include: a) penetration b) malignant reversal c) perforation d) bleeding

b) malignant reversal

Metastatic calcification: a) is seen in generalized malignant tumors b) may be triggered by massive metastatic spreading of cancer to bone c) does not depend on the level of calcium in plasma d) affects mainly the liver

b) may be triggered by massive metastatic spreading of cancer to bone Deposition of calcium salts in otherwise normal tissue, because of elevated serum levels of calcium, which can occur because of deranged metabolism as well as increased absorption or decreased excretion of calcium and related minerals, as seen in hyperparathyroidism. In contrast, dystrophic calcification is caused by abnormalities or degeneration of tissues resulting in mineral deposition, though blood levels of calcium remain normal.

Which of these tumors is most typical in children? a) meningioma b) medulloblastoma c) glioblastoma d) oligodendroglioma

b) medulloblastoma

The meaning of steatosis: a) the presence of lipids in fat cells b) multiplication of lipids in parenchymal cells c) the presence of lipids in the subcutaneous adipose tissue on obese d) the presence of lipid droplets in hyaline connective tissue

b) multiplication of lipids in parenchymal cells

Which statement is correct about tumors: a) the only tumor type arising from neuroendocrine cells are carcinoids b) neurilemom arises from Schwann cells c) names of adenomas are based on their structure d) names of adenomas are derived from the name of the default organ

b) neurilemom arises from Schwann cells c) names of adenomas are based on their structure

To mesenchymal tumors do NOT belong: a) dermatofibroma b) neurofibroma c) nevus flammeus d) nevus pigmentosus e) leiomyoma

b) neurofibroma d) nevus pigmentosus

Mucocele is: (mucinous distention) a) congential b) obtained c) dysontogenic cyst d) retention of cyst of bronchial gland

b) obtained A mucocele is any dilatation (typically pathologic) with accumulation of mucus Retention cyst Examples: Oral mucocele Mucocele of the petrous apex Mucocele of the paranasal sinuses Mucocele of appendix Gallbladder mucocele

Breslow is defined as: a) the depth of invasion of squamous cell carcinoma measured from the basement membrane of the site with the deepsest invasion b) the depth of invasion measured in malignant melanoma from stratum granulosum of epidermis to the place with the deepest invasion c) histological subtype of malignant melanoma d) cytological subtype of malignant melanoma

b) the depth of invasion measured in malignant melanoma from stratum granulosum of epidermis to the place with the deepest invasion

Which of the following is/are associated with the term reparation? a) tissue replacement by another tissue that is morphologically and functionally the same as the original tissue b) the healing process resulting in a newly formed tissue c) exemplified by the regeneration of surface epithelium in healing of erosion d) reparative processes do not arise in bone tissue

b) the healing process resulting in a newly formed tissue

The source of pulmonary embolism is not: a) the deep vein thrombosis b) thrombosis in the aorta c) thrombosis in the right heart d) thrombosis in the left heart during a closed foramen ovale

b) thrombosis in the aorta d) thrombosis in the left heart during a closed foramen ovale

Hamartoma is: a) designation of bleeding into a joint b) tumor erroneously connected in tissues c) tumor of a given organ structures goblets d) bleeding into the abdominal cavity e) Subperiosteal hematoma skull bones after birth

b) tumor erroneously connected in tissues

Which of the following is/are true of epulis? a) aberrant goiter at the root of tongue b) tumor of the gum c) acquired dystrophy of mesenchymal tissue d) a special form of dental caries in milk teeth

b) tumor of the gum

Which of the following is/are true of pathological calcification? a) occurs only in pathologically changed tissues b) two forms are known - dystrophic and metastatic c) the assumption is always hypercalcemia d) always begins as an intracellular process

b) two forms are known - dystrophic and metastatic

Gangrene: a) never occur in children b) wet gangrene is caused by bacteria c) never affects internal organs d) it is usually preceded by necrosis, subsequently modified

b) wet gangrene is caused by bacteria d) it is usually preceded by necrosis, subsequently modified

What medullary tumor is commonly referred to tumors: a) growing up primarily in the bone marrow b) with small amounts of stroma and large amounts of cells c) with large amounts of stroma and few cells d) marrow nadlevin e) exclusively metastatic to the bone marrow

b) with small amounts of stroma and large amounts of cells

Schiller-Duval bodies are found in: a) Wilms tumors b) yolk sac tumor c) they do not exist d) in steroid-forming tumor cells

b) yolk sac tumor -Schiller-Duval body is a cellular structure in endodermal sinus tumors (yolk sac tumors) which are the most common testicular cancer in children. Schiller-Duval bodies are said to resemble a glomerulus. They have a mesodermal core with a central capillary, all lined by flattened layers of both visceral and parietal cells. Immunofluorescent stain may show eosinophilic hyalin-like globules both inside and outside the cytoplasm that contain AFP and alpha 1-antitrypsin. -Wilms' tumor or nephroblastoma is cancer of the kidneys that typically occurs in children, rarely in adults.Wilms' tumor is a malignant tumor containing metanephric blastema, stromal and epithelial derivatives. Characteristic is the presence of abortive tubules and glomeruli surrounded by a spindled cell stroma. The stroma may include striated muscle, cartilage, bone, fat tissue, fibrous tissue. The tumor is compressing the normal kidney parenchyma.

Choose the disease(s) that can be caused by Haemophilus infection: a) Ulcus durum b) Ulcus molle c) Leptomeningitis d) Encephalitis

b, c

Choose the statement(s) that apply for malignant tumours: a) The higher the malignancy, the higher the prevalence of the tumour in the population b) They manifest themselves with infiltrative growth into their surroundings c) They have a tendency for dedifferentiation d) All malignant tumours metastasize early

b, c

Choose the statement(s) valid for polyarteritis nodosa: a) Is a granulomatous infection b) Is a necrotizing infection c) The kidneys are typically affected d) Monocytes are predominant in the inflammatory infiltrate

b, c

Choose the statement(s) valid for porphyrias: a) The affected organs show fluorescence under infrared light b) They are mostly inherited in an autosomal dominant way c) May manifest as abdominal pain d) They don't affect the liver

b, c

Dystrophic calcification: a. Occurs in organs where changes in pH are present b. Accompanies degenerative changes of the heart valves c. Is the substrate of Psammoma bodies d. Occurs in liquefactive necrosis

b, c

For diffuse gastric adenocarcinoma does NOT apply: a) It creates the characteristic shape of the stomach (skirrhus) b) Unlike other histological types it is not associated with Helicobacter pylori infection c) May lead to the formation of Krukenberg tumours d) The tumour cells have the shape of a signet ring

b, c

For prostate cancer the following statement(s) applies: a) For the staging of prostate cancer the Gleason-score is used b) Histologically it is almost exclusively an adenocarcinoma c) Bone metastases are osteoplastic d) Most tumours in the Czech Republic are found in a clinically advanced stage

b, c

Necrosis: a) Is to a certain degree reversible b) Induces an inflammatory response in the surrounding tissue c) Ischemia is one of its causes d) Occurs in the tissues after death, before the onset of autolytic changes

b, c

Select the valid statement(s) about shock: a. Leads to the activation of the parasympathetic system and centralization of circulation b. During the autopsy cerebral edema is usually described c. Part of the autopsy finding is vasoparalytic nephrosis in the kidney d. Leads to hepatization in the lungs

b, c

Foamy macrophages: a) Contain mucus b) Contain lipids c) They are PAS positive d) They are the cause of ulceration on the surface of atherosclerotic plaques

b, c, d

Phlegmone: a) Is surrounded by a pyogenic membrane b) The inflammatory infiltrate consists of polymorphonuclear leukocytes c) It typically develops in the soft tissues of the lower limb in the so-called diabetic foot d) Is an uncircumscribed inflammation

b, c, d

Select the valid statement(s) about syphilis? a. Miliary and gummatous forms are distinguished b. The infectious agent can be demonstrated microscopically in the native specimen using phase contrast microscopy c. The manifestations of secondary syphilis include gummata and aortitis luetica d. The manifestations of tertiary syphilis include tabes dorsalis and progressive paralysis

b, d (aortitis and gumma is manifested in tertiary syphilis) Tabes dorsalis is also known as syphilitic myelopathy, is a slow degeneration (specifically, demyelination) of the nerves primarily in the dorsal columns (posterior columns) of the spinal cord (the portion closest to the back of the body). These nerves normally help maintain a person's sense of position (proprioception), vibration, and discriminative touch.abes dorsalis is caused by demyelination caused by advanced syphilis infection (tertiary syphilis), when the primary infection by the causative spirochete bacterium Treponema pallidum is left untreated for an extended period of time (past the point of blood infection by the organism)

Choose the statements(s) valid for porphyrinas: a) the affectedd organs show fluorence under infracted light b) they are mostly inherited in an autosomal dominant way c) may manifest as abdominal pain d) they dont affect the liver

b,c

Clostridia are the causative agents of: a. Gas gangrene and dysentery b. Myonecrosis and toxemia c. Pseudomembranous colitis and botulism d. Atypical pneumonia and tetanus

b,c

Mucocele a. Is a congenital dysontogenic cyst b. Is an acquired change c. Occurs in the appendix d. Is retention cyst in bronchial glands

b,c

The consequence(s) of chronic alcoholism include: a. Thrombangitis obliterans b. Cardiomyopathy c. Wernicke encephalopathy d. Pellagra

b,c

Which of the following diseases belong among the non-infectious sequelae of streptococcal pharyngitis a. Rheumatoid arthritis b. Acute glomerulonephritis c. Meningitis d. Hepatitis e. Orchitis

b. Acute glomerulonephritis

Which of the following does NOT apply to carcinoma in situ? a. It can be detected by a screening examination methods b. At the time of diagnosis may have already established metastases c. Tumor growth does not go through the basement membrane d. Connected with monoclonal disease e. Mutations occurs

b. At the time of diagnosis may have already established metastases CIS = a group of abnormal cells that grow in their normal place. For example, carcinoma in situ of the skin, also called Bowen's disease, is the accumulation of dysplastic epidermal cells within the epidermis only, that has failed to penetrate into the deeper dermis. For this reason, CIS will usually not form a tumor

With which deficiency of the following vitamins can there be an association with megaloblastic anemia and Chrohn`s disease: a. B1/Thiamine b. B12 c. B2/Riboflavin d. Pyridoxine e. Folic acid

b. B12 Crohn's disease is a type of inflammatory bowel disease (IBD) that may affect any part of the gastrointestinal tract from mouth to anus. Symptoms often include: abdominal pain, diarrhea (which may be bloody if inflammation is severe), fever and weight loss

How do tumor promoters work? a. Interaction with the receptor for PDGF b. Cellular growth stimulation c. Causes oxidative damage to the DNA d. Causes viral infection e. Activate initiators

b. Cellular growth stimulation

Which of the following infectious agents cause ornithosis a. Coxiella burnetii b. Chlamydia psittaci c. Actinomyces israelii d. Listeria monocytogenes e. Salmonella typhi

b. Chlamydia psittaci psittacosis — also known as parrot fever, and ornithosis — is a zoonotic infectious disease caused by a bacterium called Chlamydia psittaci and contracted from parrots, such as macaws, cockatiels and budgerigars, and pigeons, sparrows, ducks, hens, gulls and many other species of bird.

The foci of aggregation of macrophages filled with cholesterol esters in the mucosa of the gallbladder are known as a. Lipomatous atrophy b. Cholesterolosis c. Atheroma d. Granuloma e. Xanthoma

b. Cholesterolosis

Which of the following infectious agents cause gas gangrene: a. Streptococcus pyogenes b. Clostridium perfringens c. Clostridium botulinum d. Clostridium tetani e. Clostridium difficile

b. Clostridium perfringens

The stain Mucicarmine is useful to diagnose which of the following pathogens a. Mycobacteria b. Cryptococcus c. Leishmania d. Protoza e. Nocardia

b. Cryptococcus Generally this is limited to microorganisms with a cell wall that is composed, at least in part, of a polysaccharide component. One of the organisms that is identified using this staining technique is Cryptococcus neoformans. Another use is in surgical pathology where it can identify mucin. This is helpful, for example, in determining if the cancer is a type that produces mucin.

The cause of Reye syndrome is usually a. Disorders of lipoprotein synthesis b. Defects of beta-oxidation of fatty acids c. Excessive endocytosis of lipids d. Glycogen aggregations e. None of the answers are correct

b. Defects of beta-oxidation of fatty acids

Which of the following characteristics is most typical for benign tumors? a. Anaplasia b. Differentiated tumor tissue appearance c. Absence of genetic changes d. Rapid growth e. Minimal or no clinical manifestations

b. Differentiated tumor tissue appearance

Acute gonococcal infection in females typically manifests like a. Vulvitis b. Endocervicitis c. Vaginitis d. Oophoritis e. Parametritis

b. Endocervicitis

In which of the following organs of chronic abuse of nicotine increases the risk of malignancy? a. Liver b. Esophagus c. Colon d. Urinary bladder e. Gallbladder

b. Esophagus d. Urinary bladder

A patient (49 years old) has delirium tremens. He has macrocytic anemia and increased content of hemoglobin in the erythrocytes. The most likely cause of his anemia is a lack of: a. Vitamin C deficiency b. Folic acid deficiency c. Vitamin D deficiency d. Vitamin K deficiency e. Niacin deficiency

b. Folic acid deficiency

Which substance accumulate in Krabbe`s disease a. Glucocerebrosides b. Galactocerebrosides c. GM2 gangliosides d. Ceramides e. Copper

b. Galactocerebrosides Mr. Krabbe likes galactose, og han krabber langs peripheral nerves..

Which of the following infectious agents can cause lung lesions similar as TB: a. Aspergillus fumigatus b. Histoplasma capsulatum c. Tularemia d. Toxoplasma gondii e. Neisseria gonorrhea

b. Histoplasma capsulatum Histoplasmosis (also known as "cave disease") is a disease caused by the fungus Histoplasma capsulatum. Symptoms of this infection vary greatly, but the disease affects primarily the lungs.

What are the causes of steatosis of the liver a. Ingrowth of adipose tissue to the stromal connective tissue b. Inhibition of oxidation of fatty acids from the effects of free radicals on enzymes c. Action of pancreatic enzymes on triglycerides d. Changes of the staining properties of cells due to accumulation of ER e. Accumulation of apoprotein as a consequence of increased proteosynthesis

b. Inhibition of oxidation of fatty acids from the effects of free radicals on enzymes

Rb gene: a. Is a protooncogene b. Is a tumorsuppressor gene c. Found in neurofibromatosis d. The mutated form is found in retinoblastoma e. Its protein product is involved in HPV oncogenesis

b. Is a tumorsuppressor gene d. The mutated form is found in retinoblastoma e. Its protein product is involved in HPV oncogenesis

What applies to acute orchitis: a. Its always bilateral b. It can accompany infectious parotitis c. The inflammation has a granulomatous character d. Surgical therapy is indicated e. hCG production is increased

b. It can accompany infectious parotitis - Inflammation of the testes. It can also involve swelling, heavy pains and frequent infection, and is more rarely known as didymitis (as in epididymis). - Can be related to epididymitis infection that has spread to the testicles (then called "epididymo-orchitis"), sometimes caused by the sexually transmitted diseases chlamydia and gonorrhea

Which of the following statements applies to dystrophic calcification a. It can originate during hypercalcemia b. It can be caused by caseous necrosis c. It can be caused by hyperthyroidism d. It can be caused by hypoparathyroidism e. It can develop from osteolytic malignant metastases

b. It can be caused by caseous necrosis

Which of the following applies to Trachoma: a. It's a serious generalized skin disease b. It's chlamydial keratoconjunctivitis leading to blindness c. The same etiologic agent cause lymphogranuloma venereum d. Its a tropic parasitic disease e. Its a inflammatory disease of the trachea

b. It's chlamydial keratoconjunctivitis leading to blindness c. The same etiologic agent cause lymphogranuloma venereum Also called granular conjunctivitis and blinding trachoma is an infectious disease caused by the bacterium Chlamydia trachomatis. The infection causes a roughening of the inner surface of the eyelids. This roughening can lead to pain in the eyes, breakdown of the outer surface or cornea of the eyes, and possibly to blindness

Metastatic calcification typically affects a. Lungs, kidneys, and liver b. Kidneys, lungs, and cornea c. Liver, pancreas, and basal ganglia d. Choroid plexus and old vascular thromboses e. Lungs, cornea, and stomach

b. Kidneys, lungs, and cornea

With which of the following infectious agents do we find hepatosplenomegaly and macrophages in all of the organs containing the live form of the parasite a. Cryptosporidium b. Leishmania donovani c. Toxoplasma gondii d. Histoplasma capsulatum e. Bacillus anthracis

b. Leishmania donovani It is a human blood parasite responsible for visceral leishmaniasis or kala-azar, the most severe form of leishmaniasis. It infects the mononuclear phagocyte system including spleen, liver and bone marrow

Which findings can one expect to find first in the stool of patients with infectious typhoid fever (typhus abdominalis) a. Mulitple basophils b. Multiple monocytes c. None or only isolated leukocytes d. Multiple polymorphonuclear cells e. Multiple giant multinuclear cells

b. Multiple monocytes

Which of the following organisms don't cause acute inflammation a. Streptococcus pneumonia b. Mycobacterium tuberculosis c. Neisseria meningitides d. Mycobacterium leprae e. Borrelia vincenti

b. Mycobacterium tuberculosis d. Mycobacterium leprae

Which of the following metals can cause nasal sinus tumors after chronic exposure: a. Copper b. Nickel c. Lead d. Zinc e. Mercury

b. Nickel

What type of inflammation is produced by the influenza involving the upper respiratory tract a. Suppurative b. Nonsuppurative hemorrhagic c. Proliferative d. Granulomatous e. Serous

b. Nonsuppurative hemorrhagic

Which of the following statements about serum tumor markers is INCORRECT? a. CA-125 belong among them b. Normal levels of tumor marker reliably excludes the corresponding tumor c. Used as screening methods d. Can be used to monitor the effects of therapy e. Due to the low sensitivity in early stages of tumor it's not recommended for screening

b. Normal levels of tumor marker reliably excludes the corresponding tumor

Which substance cause thorny, hard, dark-brown concrements a. Urate b. Oxalate c. Cholesterol d. Mixed e. Phosphate

b. Oxalate

Which of the following are not examples of viral diseases a. Poliomyelitis anterior acuta b. Pertussis c. Trachoma d. Jakob-Creutzfeldt disease e. Osteomyelitis

b. Pertussis c. Trachoma d. Jakob-Creutzfeldt disease e. Osteomyelitis

What is the typical common histological feature of infections with Herpes simplex and Varicella a. Production of eosinophilic granules/corpuscles b. Production of intranuclear inclusions c. Production of granulomas with giant multinuclear cells d. Eosinophilia e. Plasmocellular response of the immune system

b. Production of intranuclear inclusions

Which of the following is not found in Horner's syndrome? a) miosis b) ptosis c) fever d) enopthalmus

c) fever

In which of the following diseases does so-called Negri bodies occur in the cytoplasm of neurons: a. Tetanus b. Rabies c. Toxoplasmosis d. Cat scratch disease e. Dengue fever

b. Rabies Negri bodies are eosinophilic, sharply outlined, pathognomonic inclusion bodies (2-10 µm in diameter) found in the cytoplasm of certain nerve cells containing the virus of rabies, especially in Ammon's horn of the hippocampus

Which of the following infectious agents cause spotted fever: a. Coxiella burnetti b. Rickettsia rickettsii c. Chlamydia psittaci d. Chlamydia trachomatis e. Rickettsia prowazekii

b. Rickettsia rickettsii - A spotted fever is a type of tick-borne disease which presents on the skin.They are all caused by bacteria of the genus Rickettsia. Typhus is a group of similar diseases also caused by Rickettsia bacteria, but spotted fevers and typhus are different clinical entities - Types of spotted fevers include: Mediterranean spotted fever Rocky Mountain spotted fever Queensland tick typhus Helvetica Spotted fever

For which disease is there a typical rash starting in the form of pink papules from the region of the ear which then rapidly becomes maculopapular and spreads to the face, neck, trunk, and extremities a. Mumps b. Rubella c. Varicella d. Infectious mononucleosis e. Rotavirus infection

b. Rubella

Impetigo is caused by a. C. difficile b. S. aureus c. C. diphtheria d. K. pneumoniae e. P. aeruginosa

b. S. aureus Brennkopper: a highly contagious bacterial skin infection most common among pre-school children It is primarily caused by Staphylococcus aureus, and sometimes by Streptococcus pyogenes. Both bullous and nonbullous are primarily caused by Staphylococcus aureus, with Streptococcus also commonly being involved in the nonbullous form

Which of the following statements is true? a. The lymph node metastasis is usually in the area of the sarcoma b. Sarcomas metastasize predominantly by the blood c. Hematogeneous metastases usually arise from the spread of nodal metastasis d. All tumor emboli give rise to metastasis e. due to the presence of blood-brain barrier the brain metastases are extremely rare

b. Sarcomas metastasize predominantly by the blood

Which of the following statements about steatosis is correct a. Steatosis and lipidosis are synonyms b. The most common hereditary defect of lipid metabolism are lysosomal enzymopathies c. Deficiency of HDL can cause steatosis d. Alcoholic steatosis of the liver is caused by mitochondrial dysfunction e. It`s a special manifestation of lipomatosis

b. The most common hereditary defect of lipid metabolism are lysosomal enzymopathies c. Deficiency of HDL can cause steatosis ? I would believe that e should not be correct ? Maybe d could be correct..

Which of the following applies for staphylococcal infections: a. They often have a phlegmonous character b. They often cause abscesses c. They originate endogenously as opportunistic infections d. They are often seen as secondary infections e. They typically cause lobar pneumonia when affecting the lung

b. They often cause abscesses d. They are often seen as secondary infections

Which of the following infectious agents can cause myositis a. Aspergillus fumigatus b. Trichinella spiralis c. Strongyloides stercoralis d. Candida albicans e. Ascaris lumbricoides

b. Trichinella spiralis Myositis is a general term for inflammation of the muscles. Many such conditions are considered likely to be caused by autoimmune conditions, rather than directly due to infection (although autoimmune conditions can be activated or exacerbated by infections.)

Which of the following infections can manifest with apical aneurysm of the heart, megaesophagus, and megacolon a. Schistosomiasis b. Trypanosoma cruzi c. Entamoeba histolytica d. Visceral leishmaniasis e. Enterobius vermicularis

b. Trypanosoma cruzi

Which of the following pathogens don't invade into the mucosa, but cause disease via their enterotoxin: a. Neisseria gonorrhea b. Vibrio cholera c. Campylobacter jejuni d. Yersinia enterocolitis e. Bacillus anthracis

b. Vibrio cholera

Which vitamin deficiency manifests with demyelination of the anterior horns of the spinal cord: a. Vitamin B1 (Thiamine) deficiency b. Vitamin B12 deficiency c. Vitamin B2 (Riboflavin) deficiency d. Vitamin B6 deficiency e. Vitamin E deficiency

b. Vitamin B12 deficiency

Which vitamin deficiency is seen with anemia from atrophic gastritis: a. Vitamin B1 (Thiamine) deficiency b. Vitamin B12 deficiency c. Vitamin B2 (Riboflavin) deficiency d. Pyridoxine deficiency e. E deficiency

b. Vitamin B12 deficiency (b12 is absorbed in jejunum)

Which of the vitamin deficiencies are associated with rachitis/rickets: a. Vitamin B1 (Thiamine) deficiency b. Vitamin D deficiency c. Vitamin B2 (Riboflavin) deficiency d. Pyridoxine (Vitamin B6) deficiency e. Vitamin C deficiency

b. Vitamin D deficiency Deficiency results in impaired bone mineralization and leads to bone softening diseases including rickets in children and osteomalacia and osteoporosis in adults

What malignant lymphoma is derived from T lymphocytes? a) Burkitt malignant lymphoma b) multiple myeloma c) mycosis fungiodes d) malignant lymphogranuloma (Hodgkin disease????)

c) Mycosis fungoides -Burkitt malignant lymphoma: a cancer of the lymphatic system, particularly B lymphocytes found in the germinal center - Multiple myeloma: is a cancer of plasma cells, a type of white blood cell normally responsible for producing antibodies - Mycosis fungiodes: the most common form of cutaneous T-cell lymphoma --> a class of non-Hodgkin's lymphoma, which is a type of cancer of the immune system. Unlike most non-Hodgkin's lymphomas (which are generally B-cell related), CTCL is caused by a mutation of T cells. The malignant T cells in the body initially migrate to the skin, causing various lesions to appear. -Malignant lymphogranuloma:

Which of the following type of giant cells has tumor origin: a) Langhans type of giant cell b) osteoclast c) RS (Reed-Sternberg) cell d) myogenic giant cell e) Foreign body type giant cell

c) RS (Reed-Sternberg) cell

Hereditary tumor genetically characterized by deletion of the short arm of chromosome13 (13q14p) is called: a) Neuroblastoma b) Wilms tumor c) Retinoblastoma d) Angiosarcoma e) Neurofibroma

c) Retinoblastoma

Which of the following cancers belong among germ cell tumors? a) Embryonal rhadbomyosarcoma b) Wilms tumor (nephroblastoma) c) Teratoma d) Medulloblastoma e) Hamartoma

c) Teratoma

Which of the following causes of edema is associated with the nephrotic syndrome? a) Reduced hydrostatic pressure b) Increased oncotic pressure c) Water and sodium retention d) Decreased plasma oncotic pressure e) Increased hydrostatic pressure

c) Water and sodium retention d) Decreased plasma oncotic pressure

Which of the following tumor types has an autosomal dominant mode of inheritance? a) Squamous cell carcinoma in lungs b) Colloid colon cancer c) Wilms tumor d) Pleomorphic adenoma e) Teratoma

c) Wilms tumor

Which statement describes the correct sequence of events: a) active hyperemia, transudation, passive hyperemia b) passive hyperemia, exudation, active hyperemia c) active hyperemia, passive hyperemia, exudation d) cell infiltration, passive hyperemia, inflammatory edema

c) active hyperemia, passive hyperemia, exudation

Reye's syndrome is a) effect of chronic alcoholism b) prion encephalopathy c) acute encephalopathy associated with liver steatosis d) complications of herpes encephalitis

c) acute encephalopathy associated with liver steatosis Reye's (Ryes) syndrome is a rare but serious condition that causes swelling in the liver and brain. Reye's syndrome most often affects children and teenagers recovering from a viral infection, most commonly the flu or chickenpox

What type of shock is associated with the action of superantigens a) cardiogenic shock b) hypovolemic shock c) anaphylactic shock d) neurogenic shock e) posthemorrhagic shock

c) anaphylactic shock

Hyaline droplets: a) are found only in epithelial cells b) are located in cardiomyocytes in myocardial infarction c) are located in plasma cells in chronic inflammation d) are located in macrophages in chronic purulent inflammation

c) are located in plasma cells in chronic inflammation Here they are talking about Russel bodies -Hyaline droplets: apical cytoplasmic vesicles containing an accumulation of electron-dense amorphous materials surrounded by a unit membrane. Hyaline droplets may originate from apical vesicles after conversion to osmotic vesicles and loss of internally lined glycocalyx. They are found in the proximal tubular epithelium in biopsies from patients with renal diseases. Hmmmm.....

For secondary systemic AA amyloidosis does not apply: a) accompanies chronic suppurative processes b) accompanies chronic TB c) arises from a pathological immunoglobulin production d) amyloid is deposited extracellularly

c) arises from a pathological immunoglobulin productio

Which is true of toxoplasmosis: a) is a viral disease b) is not transferred through placenta c) can cause ...opthalmia d) affects lymph nodes

c) can cause ...opthalmia d) affects lymph nodes - A parasitic disease caused by the protozoan Toxoplasma gondii. -Animals are infected by eating infected meat, by ingestion of feces of a cat that has itself recently been infected, and by transmission from mother to fetus -During acute toxoplasmosis, symptoms are often influenza-like: swollen lymph nodes, or muscle aches and pains that last for a month or more -The parasite can cause encephalitis (inflammation of the brain) and neurological diseases, and can affect the heart, liver, inner ears, and eyes (chorioretinitis).

Neuroectoderm tumors do NOT include: a) malignant melanoma b) medulloblastoma c) carcinoma from C cells d) neurofibromas

c) carcinoma from C cells - Neuroectoderm tumor: a tumor of the central or peripheral nervous system. C-cell carcinoma - neuroendocrine tumo

The retinoblastoma gene regulates: a) DNA transcription b) signal transmission in cells c) cell cycle progression d) apoptosis e) repair DNA

c) cell cycle progression

Which of the following is/are not hematogenic pigments? a) hematoidin b) hemosderin c) ceroid d) bilirubin

c) ceroid

Which of the following may not always be a germ cell tumor? a) yolk sac tumor b) dysgerminoma c) choriocarcinoma d) teratoma

c) choriocarcinoma

Which of the following tumors are commonly associated with mutations in the APC gene? a) bronchogenic carcinoma b) adenocarcinoma of the uterine cervix c) colonic carcinoma d) squamous cell carcinoma of the cervix

c) colonic carcinoma

A 49 year old female presents with a lump in her left breast.. Biopsy showed invasive ductal carcinoma of the mammary gland. There is increased fibrosis in the vicinity of the tumor. This finding is an example of: a) anaplasia b) carcinoma in situ c) desmoplasia d) dysplasia e) metaplasia

c) desmoplasia (the growth of fibrous or connective tissue)

The staphylococcal infection does not cover: a) purulent bronchopneumonia b) carbuncle c) erysipel d) impetigo

c) erysipel

Which of the following is/are true of the RB gene: a) has no relationship to oncogenesis b) is one of the proto-oncogenes c) familial retinoblastoma inheritance requires one copy of the mutated RB gene c) for sporadic retinoblastoma both mutated copies of the RB gene are inherited

c) familial retinoblastoma inheritance requires one copy of the mutated RB gene

Multiple myeloma is not characterized by: a) skeletal involvement b) hypercalcemia c) the proliferation of lymphoblasts d) the proliferation of malignant plasma cells

c) the proliferation of lymphoblasts Multiple myeloma, also known as plasma cell myeloma or Kahler's disease is a cancer of plasma cells, a type of white blood cell normally responsible for producing antibodies. In multiple myeloma, collections of abnormal plasma cells accumulate in the bone marrow, where they interfere with the production of normal blood cells. Most cases of multiple myeloma also feature the production of a paraprotein—an abnormal antibody which can cause kidney problems. Bone lesions and hypercalcemia (high blood calcium levels) are also often encountered.

Paradoxial embolism: a) occurs in systemic hypertension b) is directed from the left heart into the right side c) the requirments is an open foramen ovale and increased pressure in the right heart d) occurs only in bacterial endocarditis

c) the requirments is an open foramen ovale and increased pressure in the right heart A paradoxical embolism, also called a crossed embolism, is a kind of stroke or other form of arterial thrombosis caused by embolism of a thrombus (blood clot) of venous origin through a lateral opening in the heart, such as a patent foramen ovale. The opening is typically an atrial septal defect, but can also be a ventricular septal defect.

Which of the following is/are not characteristic of glioblastoma? a) it is most common among people older than 50 years b) frequently palisading necrosis is present c) the tumor border is sharp d) mitotic activity is high

c) the tumor border is sharp

For Buerger disease does not apply: a) it affects the limbs b) it affects heavy smokers c) they are necrotizing vasculitis d) it is preceded by phlebitis migrans

c) they are necrotizing vasculitis Thromboangiitis obliterans (or presenile gangrene) is a recurring progressive inflammation and thrombosis (clotting) of small and medium arteries and veins of the hands and feet. It is strongly associated with use of tobacco products, primarily from smoking! Buerger's disease (thromboangiitis obliterans) is a rare disease of the arteries and veins in the arms and legs. In Buerger's disease, your blood vessels become inflamed, swell and can become blocked with blood clots (thrombi). This eventually damages or destroys skin tissues and may lead to infection and gangrene. Thrombophlebitis is phlebitis (vein inflammation) related to a thrombus (blood clot). When it occurs repeatedly in different locations, it is known as "Thrombophlebitis migrans", "migrating thrombophlebitis" or Trousseau's syndrome

Which of these characters is least characteristic of fresh pulmonary infarcts? a) fibrinous pleuritis b) they are more common in patients with heart failure c) they have a pale color d) have a wedge shape

c) they have a pale color They are hemorrhagic -RED

Abnormalities in DCC (Deleted in colon cancer) gene are related to: a) abnormal chromosome distribution between cells during cell division b) changes in cell cycle c) to changes in cell adhesion d) changes in DNA repair e) DNA transcription

c) to changes in cell adhesion

General marker of mesenchymal cell origin is: a) cytokeratin b) tubulin c) vimentin d) glial fibrillary protein e) S 100 protein

c) vimentin

All of the following characteristics of tumors are associated with invasion ability EXCEPT: a) type IV collagenase production b) expression of receptors for extracellular matrix components c) viral infection of tumor cells d) paracrine interactions e) angiogenesis

c) viral infection of tumor cells

Which of the following statements is true of paraneoplastic syndrome: a) manifested only in squamous cell carcinoma b) manifested only in upper respiratory tract tumors c) we find him often in small cell lung cancer d) manifested only in the veins e) only condition is the presence of liver metastases

c) we find him often in small cell lung cancer

Hyperplasia is: a) Biceps muscle enlargement in bodybuilders b) Myocardial thickening in patients with hypertension c) Proliferation of hepatocytes after partial hepatectomy d) Reaction of the bronchial mucosa to irritation by tobacco smoking

c,

Choose the statement(s) valid for Hodgkin lymphoma: a) The tumour cells predominate in the histologic section b) It is always possible to find typical Reed Sternberg cells c) It develops most often in the mediastinal lymph nodes d) Coughing may be the first sign of disease

c, d

Choose the statement(s) valid for meningioma: a) Occurs only in the skull b) Has no malignant variants c) Most commonly develops on convexity of hemispheres d) It originates from cells of pia arachnoidea

c, d

Choose the valid statement(s) for osteosarcoma. a) Metastasizes often to lymph nodes b) Arises in the diaphysis of long bones c) Higher risk for its development is described in Paget's disease d) Typically metastasizes to the lungs

c, d

Conventional renal cell carcinoma a) Arises from the epithelium of the renal pelvis b) Lymphatic spread with infiltration of draining lymph nodes is typically described c) In the tumor we typically observe areas of haemorrhage d) It can grow in the lumen of renal blood vessel

c, d

DLBCL: a) ALK fusion gene is typically described b) It always originates from lymph node tissue c) The connection with EB virus infection is known d) May also occur due to the dedifferentiation of other lymphomas

c, d

Metastatic calcification: a. Affects especially the brain b. Occurs in normocalcemia c. Extracellular deposition of calcium hydroxyapatite is described d. Calcium deposits are found in the mucosa of the stomach

c, d

Reye's syndrome is: a. An effect of chronic alchoholism b. A prion encephalopathy c. Acute encephalopathy associated with hepatic steatosis d. Associated with intake of acetylsalicylic acid

c, d

Select the valid statement(s): a. N. meningitidis belongs among yeasts b. Giardia lamblia belongs among viruses c. The causative agent of malaria belongs among protozoa d. The causative agent of syphilis belongs among bacteria

c, d

Which of the following are NOT risk factor(s) for atherosclerosis: a) Diabetes mellitus b) Nicotinism c) Elevated HDL cholesterol levels in the blood d) Arterial hypotension

c, d

Exposure to sunlight plays an important etiological role in the development of all the following pathologies EXCEPT: a. Squamous cell carcinoma b. Basal cell carcinoma c. Adenocarcinoma d. Melanoma e. Actinic keratoses

c. Adenocarcinoma

Which of the following statements is true about glomangioma? a. Glomangiomas arise from specialized structures associated with the regulation of pressure b. Are typically located near the proximal interphalangeal joint c. Are usually painful d. Are usually malignant e. Glomangiomas usually surround nerves

c. Are usually painful A glomus tumor (also known as a "solitary glomus tumor," or glomangioma) is a rare benign neoplasm arising from the glomus body (a component of the dermis layer of the skin, involved in body temperature regulation) and mainly found under the nail, on the fingertip or in the foot

Which cell type are most markedly decreased in HIV patients: a. Cells of the monocyte-macrophage cell line b. Cytotoxic/Suppressor T lymphocytes c. CD4+ (Helper/Inducer) lymphocytes d. Mature plasma cells e. Pre-B lymphocytes

c. CD4+ (Helper/Inducer) lymphocytes

Pneumocystis carinii - Which statements are correct: a. Cause atypical pneumonia in all affected individuals b. Usually cause encephalitis leading to death c. Cause atypical pneumonia in immuncompromised patients d. Can cause atypical pneumonia in premature children e. Can manifest as a foamy mass in the alveoli

c. Cause atypical pneumonia in immuncompromised patients d. Can cause atypical pneumonia in premature children e. Can manifest as a foamy mass in the alveoli Pneumocystis jiroveci pneumonia (PJP), formerly known as Pneumocystis carinii pneumonia (PCP), is the most common opportunistic infection in persons with HIV infection.

Which of the following infectious agents cause botulism a. Streptococcus pyogenes b. Clostridium perfringens c. Clostridium botulinum d. Clostridium tetani e. Clostridium difficile

c. Clostridium botulinum

Which of the following gene is among tumor suppressor genes? a. Ras b. Neu c. DCC d. Abl e. Ret

c. DCC - Deleted in Colorectal Carcinoma (gene on chrom. 18)

Which of the following statements about calcification are correct a. Metastatic calcification is most commonly caused by hypervitaminosis D b. Dystrophic calcification never originates from neoplasms c. Dystrophic calcification often originates from caseous necrosis d. Metastatic calcification affects the tunica media of muscular arteries e. Calcification never affects the cornea

c. Dystrophic calcification often originates from caseous necrosis

Enzymatic digestion of tissue with formation of chalky soap are characteristic of a. Coagulation necrosis b. Caseous necrosis c. Fat necrosis d. Liquefactive necrosis e. Fibrinoid necrosis

c. Fat necrosis

In the tissue of patients with Tay-Sachs disease we find increased content of a. Sphingomyelin b. Hexosaminidase A c. GM2 gangliosides d. Cerebrosides e. Keratin sulfate

c. GM2 gangliosides

What is the most common cause of genital ulcers in the western industrialized world a. Chancroid b. Chondylomata c. Genital herpes d. Primary syphilis e. Granuloma inguinale

c. Genital herpes

So-called pemzové (''spongious'') lungs are most commonly found in patients with a. Lipidosis b. Mucoviscidosis c. Osteolytic metastases d. Connective tissue defects e. Kidney failure on dialysis

c. Osteolytic metastases

A 40-year-old HIV-positive man with a CD4+ T-lymphocyte count of 25/mm3 complains of progressive memory impairment, confusion and problems with incontinence. MRI shows mild cerebral atrophy. The patient later dies from disseminated aspergillosis. During autopsy of the brain, cerebral atrophy is confirmed macroscopically. Histologically in the cerebral tissue there are found mulitiple lympho-histiocytic infiltrates with numerous microglial nodes/foci and dispersed large multinuclear cells. Which of the following conditions would most likely cause the patients complaints: a. Aspergillosis b. CMV encephalitis c. HIV encephalitis d. Mycobacterial infection e. Vacuolar myelopathy

c. HIV encephalitis

Which complication is the least severe for patients with extensive second and third degree burns: a. Fluid loss b. Infection c. Hyperthermia d. Scars e. Pain

c. Hyperthermia

All of the following diseases are connected with the herpes viridae except a. Herpes zoster b. Varicella c. Influenza d. CMV e. Infectious mononucleosis

c. Influenza

Which of the following statements about mucoviscidosis(CF) are not correct a. Cystic pancreatofibrosis is a synonym b. We find a defect of the permeability of Cl in the apical membrane of epithelial cells c. It occurs during loss of water and ions in the GIT d. It cause infertility e. Development of cor pulmonale is a complication of this disease

c. It occurs during loss of water and ions in the GIT

Which of the following statements about hyaline droplets are correct a. It`s ethopathogenically a intracellular process b. It`s a irreversible process c. It occurs in the liver during dysfunction of alpha-1-antitrypsin d. It don`t occur in the kidneys e. It manifests as albumin accumulation in the proximal renal tubules

c. It occurs in the liver during dysfunction of alpha-1-antitrypsin e. It manifests as albumin accumulation in the proximal renal tubules

.

c. Lead

Which of the following metals cause the formation of metaphysic lines in growing bones: a. Arsenic b. Barium c. Lead d. Nickel e. Zinc

c. Lead

In which organ do we find characteristic pathological changes in yellow fever a. Brain b. Lungs c. Liver d. Spleen e. Stomach

c. Liver

Which manifestation are most characteristic for staphylococcal infection a. Diffuse scarring b. Acute interstitial inflammation c. Multiple abscesses d. Mononuclear inflammatory infiltrate e. Lobar involvement of the lung

c. Multiple abscesses

Which of the following organisms are obligate intracellular parasites a. Mycobacterium tuberculosis b. Mycobacterium kansasii c. Mycobacterium leprae d. Histoplasma capsulatum e. Legionella pneumophila

c. Mycobacterium leprae Obligate parasite, means it depends on host for survival

What is the typical complications of diphtheria a. Peritonitis b. Meningitis c. Myocarditis d. Hepatitis e. Nephritis

c. Myocarditis

Which finding can we expect in the stool of patients with infectious enterotoxigenic E. coli a. Multiple basophils b. Multiple mononuclear cells c. None or only isolated leukocytes d. Multiple polymorphonuclear cells e. Multiple giant multinuclear cells

c. None or only isolated leukocytes

Which findings can we expect in the stool of patients with cholera a. Multiple basophils b. Multiple mononuclear cells c. None or only isolated leukocytes d. Multiple polymorphonuclear cells e. Multiple giant multinuclear cells

c. None or only isolated leukocytes

Erysipelas is an infectious disease characterized by which type of inflammation a. Serous b. Fibrinous c. Phlegmonous d. Abscess e. Empyema

c. Phlegmonous It is an acute infection typically with a skin rash, usually on any of the legs and toes, face, arms and fingers. It is an infection of the upper dermis and superficial lymphatics, usually caused by Beta-hemolytic group A streptococcus bacteria on scratches or otherwise infected areas

Which of the following condition is caused by infection with Epstein-Barr virus? a. Chromosomal translocation b. T-cell necrosis c. Polyclonal proliferation of B-cells d. Monoclonal proliferation of B-cells e. Polyclonal proliferation of T-cells

c. Polyclonal proliferation of B-cells

Which of the following substances is NOT a tumor marker? a. Alfa-fetoprotein (AFP) b. Prostate-specific antigen (PSE) c. Prolactin d. Human chorionic gonadotropin (hCG) e. Carcinoembryonic antigen (CEA)

c. Prolactin

In which of the following Rickettsial infections are vasculitis not present: a. Rocky Mountain Spotted fever b. Tsutsugamushi fever c. Q fever d. Epidemic typhus (louse) e. Murine (endemic) typhus (Flea)

c. Q fever - Caused by the bacteria coxiella burnetii: it was originally identified as a species of Rickettsia, but it is no longer regarded as closely related to Rickettsiae

A two-year old boy with a malignant tumor will most likely have: a. Hamartoma b. Adenocarcinoma c. Retinoblastoma d. Squamous cell carcinoma e. Leiomyosarcoma

c. Retinoblastoma A rapidly developing cancer that develops from the immature cells of a retina, the light-detecting tissue of the eye and is the most common malignant tumor of the eye in children

On autopsy of the lung of a child of who dies of acute pneumonia there is found numerous Warthin-Finkeldey cells with more than 100 nuclei. What is the most likely causative agent a. EBV b. CMV c. Rubella d. Mumps e. HIV

c. Rubella - measles

Which of the following complications can one encounter in a patient with mononucleosis a. Burkitt's lymphoma b. Brain abscesses c. Ruptured spleen d. Liver cirrhosis e. Gonadal atrophy

c. Ruptured spleen

Homogenous eosinophilic cytoplasmic inclusions in plasma cells are called a. Dutcher bodies b. Corpora amylacea c. Russel bodies d. Hyaline nuclei e. Glycogen nuclei

c. Russel bodies

What is the tumor called consisting of mutually intertwined elongated bundles, unequally sized spindle cells with large, atypical nuclei and numerous hyperchromatic mitoses? a. Squamous cell carcinoma b. Adenocarcinoma c. Sarcoma d. Lymphoma e. Leiomyoma

c. Sarcoma

Grading and staging is a process which aims to assess the degree of differentiation and aggressiveness of the tumor. Which of the following is correct? a. Grading a tumor has greater clinical significance then staging b. Tumor staging shall be based on cytological differentiation of tumor cells and the number of mitoses c. Staging shall be based on the size of the primary tumor, the extent of tumor spread to lymph nodes and the presence or absence of metastases d. Grading is based on the degree of local invasion of primary tumor e. Staging of tumors do not have - unlike grading - any importance in determining the patient's prognosis

c. Staging shall be based on the size of the primary tumor, the extent of tumor spread to lymph nodes and the presence or absence of metastases

Which of the following terms are not connected with progressive lysosomal dysfunction a. Granulocytes in the CNS b. Sphingomyelinosis c. Strawberry gallbladder d. Tigered effect on heart e. Post-inflammatory pseudoxanthoma

c. Strawberry gallbladder(?) D - tiger effect on heart

The relationship between the human host and the normal bacterial flora can best be described as a. Saprophytic (organisms which obtain nutrients from dead organic matter) b. Commensal (a symbiotic relationship in which one species is benefited while the other is unaffected) c. Symbiotic d. Parasitic (a non-mutual symbiotic relationship between species, where one species, the parasite, benefits at the expense of the other) e. Facultative

c. Symbiotic

Which of the following statements about anthrax are correct a. It is a mild acute gastrointestinal disease b. It has a serious skin form and a mild pulmonary form c. The contagion consist of spores of Bacillus anthracis d. Patients die from hemorrhagic-necrotizing pneumonia e. The skin form appears like malignant pustules

c. The contagion consist of spores of Bacillus anthracis d. Patients die from hemorrhagic-necrotizing pneumonia e. The skin form appears like malignant pustules

Basilar meningitis: a) Is caused by the diplococcus Neisseria meningitidis b) The inflammatory infiltrate consists mainly of neutrophils c) The course of the disease is usually hyperacute d) The spread of the infection into the meninges is usually haematogenous

d

Choose the statement(s) that apply for amyloid: a) After removing the underlying cause it is partially resorbed; with the exception of an amyloidosis resulting from transthyretin b) In HE staining it exhibits dichroism under polarized light c) B2 macroglobulin amyloid usually accumulates in the heart d) Certain types of amyloidosis can cause accelerated development of chronic renal insufficiency

d

Choose the statement(s) valid for cholecystolithiasis: a) Men are more frequently affected b) Cholesterol crystals are always part of the stones c) A possible complication is chronic pancreatitis d) The concretions cause chronic fibroproductive inflammation of the gallbladder

d

Hepatomegaly, growth retardation, and severe hypoglycemia are features MOST consistent with which one of the following disease(s)? a) Hurler's disease b) Pompe's disease c) Niemann-Pick disease d) von Gierke's disease

d

In which of the following disease(s) no fibrinous infection is described: a) Diphtheria b) Uraemia c) Rheumatic fever d) Scleroderma

d

Mark the statement(s) valid for gas gangrene: a) Today it does not occur in developed countries, the cases are imported from abroad b) Is caused by secondary changes in atrophic tissue c) Is accompanied by septic pseudotumorous enlargement of the draining lymph nodes d) It may be the cause of death of the patient

d

Primary tuberculosis complex: a) Always arises in the lung b) Is called Simon's complex c) Undergoes metastatic calcification d) Consists of a primary focus, lymphangiitis of its draining lymphatics and lymphadenitis

d

Select the correct statement(s): A. Pathological calcifications are always metastatic b. Dystrophic calcification never occurs in TBC c. Caseous calcification belongs among the features of tuberculosis d. In the so-called pumice lungs metastatic calcification is described

d

Select the valid statement(s): a. The causative agent of kala azar disease belongs among bacteria b. The causative agent of measles belongs among bacteria c. Brucellas cause infections in rabbits d. Brucella infection can lead to abortions in humans

d

The causative agent of scurvy is: a. A bacterium b. A virus c. A fungus d. None of the listed statements is correct

d

The most common primary malignant tumour of the small intestine is: a) Adenocarcinoma b) Mesenchymal tumours c) Gastrointestinal stromal tumour d) None of the previous options

d

The most common type of tumour in the liver is: a) Hepatoceflular carcinoma b) Cavernous haernargioma c) Cholangiocarcinorna d) None of the above

d

Which of the following cells is the least sensitive towards anoxia: a) Cells of the proximal renal tubules b) Cardiomyocytes c) Epithelial cells of the small intestine d) Fibroblasts

d

Which of the following is/are true of tumors of striated muscle? a) rhabdomyoma is a frequent tumor in limb muscles b) rhabdomyosarcoma occur in the myocardium of children c) rhabdomyosarcoma can be derived into embryonic juvenile and adult d) embryonic rhaobdymyosarcoma is frequently seen in the pelvic floor

d) embryonic rhaobdymyosarcoma is frequently seen in the pelvic floor (B --> Rhabdomyoma (benign variant) in heart) Common sites of RMS include: - Head and neck (such as near the eye, inside the nasal sinuses or throat, or near the spine in the neck) - Urinary and reproductive organs (bladder, prostate gland, or any of the female organs) - Arms and legs - Trunk (chest and abdomen Types: Embryonal rhabdomyosarcoma --> ERMS tends to occur in the head and neck area, bladder, vagina, or in or around the prostate and testicles. Subtypes: Botryoid and spindle cell rhabdomyosarcomas Alveolar rhabdomyosarcoma ARMS most often occurs in large muscles of the trunk, arms, and legs. Anaplastic rhabdomyosarcoma (formerly called pleomorphic rhabdomyosarcoma) is an uncommon type that occurs in adults but is very rare in children. Most rhabdomyosarcomas develop in children, but they can also occur in adults.

Dermatitis herpetiformis is associated with which one of these: a) bechterew disease b) diverticulitis c) emphysema d) gluten sensitive enteropathy

d) gluten sensitive enteropathy

The basic pathogenetic types of amyloid does not cover a) AA amyloid b) AL amyliod c) tranthyertin type d) histamine type

d) histamine type

With chronic bronchitis is not related: a) smoking b) bronchial asthma c) productive cough d) hyaline membrane

d) hyaline membrane Hyaline membrane disease: A respiratory disease of the newborn, especially the premature infant, in which a membrane composed of proteins and dead cells lines the alveoli (the tiny air sacs in the lung), making gas exchange difficult or impossible

The causes of necrosis do not cover: a) trauma b) pathological immune response c) microbial infection d) hypothyroidism

d) hypothyroidism

Which of the following properties are characteristic of the tumor cells? a) proliferation requires external growth factor b) Increased intercellular adhesion of tumor cells c) contact inhibition of growth in vitro d) include years of evolution transformation changes / tumor progression e) terminal differentiation of tumor cells

d) include years of evolution transformation changes / tumor progression

What causes acute pulmonary edema in left-sided heart failure? a) increased arterial hydrostatic pressure b) decreased colloid osmotic pressure of plasma c) increased vascular permeability d) increased capillary hydrostatic pressure e) combination of C and D is correct

d) increased capillary hydrostatic pressure

Which of the following assertion is true for CML: a) moderate enlargement of the spleen b) mainly affects adolescents c) characterized by RS cells d) infiltrates in the liver is within sinusoids e) bone marrow is not affected

d) infiltrates in the liver is within sinusoids

Ulcerative colitis is: a) granulomatous inflammation b) transmural inflammation of the intestinal wall c) associated with formation of fissural ulcers d) inflammation with formation of inflammatory pseudopolyps

d) inflammation with formation of inflammatory pseudopolyps

For lobar pneumonia following applies: a) it is a granulomatous inflammation b) it is an interstitial inflammation c) there are presented multinucleated cells d) it affects the entire lung lobe

d) it affects the entire lung lobe Lobar pneumonia usually has an acute progression. Classically, the disease has four stages: - Congestion in the first 24 hours: This stage is characterized histologically by vascular engorgement, intra-alveolar fluid, small numbers of neutrophils, often numerous bacteria. Grossly, the lung is heavy and hyperemic - Red hepatization or consolidation : Vascular congestion persists, with extravasation of red cells into alveolar spaces, along with increased numbers of neutrophils and fibrin. The filling of airspaces by the exudate leads to a gross appearance of solidification, or consolidation, of the alveolar parenchyma. This appearance has been likened to that of the liver, hence the term "hepatization". - Grey hepatization : Red cells disintegrate, with persistence of the neutrophils and fibrin. The alveoli still appear consolidated, but grossly the color is paler and the cut surface is drier. - Resolution (complete recovery):The exudate is digested by enzymatic activity, and cleared by macrophages or by cough mechanism

What is true of atrophy: a) a rare change b) has a productive form c) is never generalized state d) may be combined with tumor

d) may be combined with tumor

What is true of necrosis: a) caseous necrosis is of ischemic origin b) liquefactive necrosis occurs only in the CNS c) type of necrosis depends only on the causative agents d) necrosis is of endogenous and exogenous origin

d) necrosis is of endogenous and exogenous origin

Which of the following is/are NOT true of plasmacytoma? a) is often accompanied by amyloid deposition b) is often accompanied by paraproteinuria c) it is B-NHL d) numerous osteoplastic metastasis are commonly present

d) numerous osteoplastic metastasis are commonly present

Supraclavicular Lymph node biopsy of a 60 year old man revealed metastasis. What is probably the tumor? a) leiomyosarcoma b) Kidney Wilms tumor c) humerus osteosarcoma d) pancreatic cancer e) embryonic carcinoma

d) pancreatic cancer

For which primary tumor is characterized for osteoplastic metastasis? a) hepatocellular carcinoma b) Basal cell skin cancer c) colorectal cancer d) prostate cancer e) Clear cell carcinoma of kidney

d) prostate cancer

The best method for detecting the origin of clonal T cell lymphoma is: a) in situ hybridization of cells. b) immunohistochemical T cell marker c) investigate cytological nuclear irregularities d) prúkazem rearrangementu DNA e) ELISA test

d) prúkazem rearrangementu DNA

W of the following items can be linked to lung cancer: a) aflatoxin b) chronic pneumonia c) benzen d) radon e) cadmium

d) radon

The characteristics of a malignant tumor do not cover: a) clonal proliferation b) invasive growth c) cell immortalisation d) reduced mitotic time

d) reduced mitotic time

Which of the following risk factors is not a risk factor of atherosclerosis? a) smoking cigarettes b) hypertension c) hypercholesterolemia d) renal disease in the family history

d) renal disease in the family history

Which of the following infectious agents cause tetanus: a. S. pyogenes b. C. perfringens c. C. botulinum d. C. tetani e. C. difficile

d. C. tetani

Which of the following metal`s vapour cause acute pulmonary edema: a. Arsenic b. Nickel c. Lead d. Cadmium e. Mercury

d. Cadmium

Which of the following infectious agents can cause thrush in children a. Aspergillus fumigatus b. Histoplasma capsulatum c. Clonorchis sinensis d. Candida albicans e. Giardia lamblia

d. Candida albicans

A 30-year old man was diagnosed with and had removal of testicular tumors. Which of the following facts are evidence of a malignant lesion? a. The size of the tumor is more than 3 cm b. More than 2 mitoses per 10 fields of vision at a magnification of 40x c. The tumor has a different sized nuclei some of which are markedly hyperchromatic d. Chest X-ray reveals two spherical deposits in the right lung field, which in previous examinations were not present e. The patient is infertile

d. Chest X-ray reveals two spherical deposits in the right lung field, which in previous examinations were not present

Which of the following infectious agents cause blindness a. Coxiella burnetti b. Rickettsia ricketsii c. Chlamydia psittaci d. Chlamydia trachomatis e. Rickettsia prowazekii

d. Chlamydia trachomatis

Which of the following susbstances typically produce hlatě nonpainful brouskovité krystaly (Vernerova couldn't give a english translation as they apparently didn't use this description in english litterature anymore a. Urate sodium b. Calcium oxalate c. Cysteine d. Cholesterol e. Oxalate

d. Cholesterol

Which of the following statements is NOT true of plasmacytoma? a. Often is accompanied by a deposit of amyloid b. Often is accompanied by paraproteinuria c. Belongs to non-Hodgkin's B-cell lymphoma d. Creates numerous osteoplastic metastases e. It affects mainly older men

d. Creates numerous osteoplastic metastases Plasmacytoma refers to a malignant plasma cell tumor growing within soft tissue or within the axial skeleton (but does it belong to non-hodgkins???? source please)

What's the most important difference between benign and malignant tumors? a. Tumor size b. Number of mitoses c. Dystrophic calcification d. Distant metastases e. Necrosis

d. Distant metastases

Which of the following properties is the most characteristic of malignant tumors? a. Autonomous proliferation of cells b. Necrosis in the center of the tumor c. Lack of encapsulation d. Distant metastasis e. Painless manifestation

d. Distant metastasis

Which of the following is a protein encoded by human papillomavirus? a. Bcl-2 b. Bax c. Tax d. E7 e. P53

d. E7 Some of the HPV "early" genes, such as E6 and E7, are known to act as oncogenes that promote tumor growth and malignant transformation. The two primary oncoproteins of high risk HPV types are E6 and E7. The "E" indicates that these two proteins are expressed early in the HPV life cycle, while the "L" indicates late expression. The HPV genome is composed of six early (E1, E2, E4, E5, E6, and E7) ORFs, two late (L1 and L2) ORFs, and a non-coding long control region (LCR)

All of the following oncogenic virus are correctly paired with caused tumors EXCEPT: a. HTLV-1 - Adult T cell leukemia/lymphoma b. Hepatitis C virus (HCV) - Hepatocellular carcinoma c. Human papilloma virus (HPV) - laryngeal papillomatosis d. Epstein-Barr virus - cervical cancer e. EBV - Burkitt's lymphoma

d. Epstein-Barr virus - cervical cancer Also called human herpesvirus 4, is a virus of the herpes family, and is one of the most common viruses in humans. It is best known as the cause of infectious mononucleosis (glandular fever). It is also associated with particular forms of cancer, such as Hodgkin's lymphoma, Burkitt's lymphoma, nasopharyngeal carcinoma, and conditions associated with human immunodeficiency virus (HIV), such as hairy leukoplakia and central nervous system lymphomas.

In the active invasion and bacteremic phases of typhus we can find Salmonella typhi in all of the following organs except: a. Terminal ileum b. Lymph nodes in the mesentery c. Liver d. Esophagus e. Blood

d. Esophagus

What is the most common form of the primary infection of Herpes simplex virus: a. Keratitis b. Conjunctivitis c. Encephalitis d. Gingivostomatitis e. Hepatitis

d. Gingivostomatitis Also known as primary herpetic gingivostomatitis or orolabial herpes: is a combination of gingivitis and stomatitis (inflam.of mouth and lips), or an inflammation of the oral mucosa and gingiva. Herpetic gingivostomatitis is often the initial presentation during the first ("primary") herpes simplex infection. It is of greater severity than herpes labialis (cold sores) which is often the subsequent presentations. Primary herpetic gingivostomatitis is the most common viral infection of the mouth.

Which of the following statements about CMV infections are not true a. Most of the adult population are seropositive b. CMV can be transmitted across the placenta to the fetus c. Most congenital infections are asymptomatic d. In patients with AIDS it most often cause encephalitis e. Infected cells are strikingly enlarged

d. In patients with AIDS it most often cause encephalitis

Which of the following statements about steatosis are correct a. Every metabolic defect related to lipid metabolism can manifest as steatosis b. Steatosis is defined as accumulation of fat in any type of cell c. The occurrence of lipid in cells is always a pathologic condition d. It`s an accumulation of lipids intracellularly in the form of lipid-drop aggregates e. Lipomatosis of the myocardium is an example of steatosis

d. It`s an accumulation of lipids intracellularly in the form of lipid-drop aggregates

Which of the following tumor is NOT malignant? a. Glioblastoma b. Lymphoma c. Melanoma d. Leiomyoma e. Medulloblastoma

d. Leiomyoma

Which finding can we expect to find in the stool of patients with shigellosis a. Multiple basophils b. Multiple mononuclear cells c. None or only sporadic leukocytes d. Multiple polymorphonuclear cells e. Multiple giant multinuclear cells

d. Multiple polymorphonuclear cells

Which vitamin deficiency presents with dementia, skin manifestations, and diarrhea: a. Vitamin B1 (Thiamine) deficiency b. Vitamin B12 deficiency c. Folic acid deficiency d. Niacin deficiency e. Pyridoxine deficiency

d. Niacin deficiency = Vit.B3

Biphasic benign tumor often present in the parotid gland is: a. Adenocarcinoma b. Papilloma c. Cystadenoma d. Pleomorphic adenoma e. Chondroma

d. Pleomorphic adenoma

Which infectious disease are not connected with a granulomatous reaction a. TBC b. Coccidioidomycosis c. Schistosomiasis d. Rabies e. Tuberculoid type of leprosy

d. Rabies

Which of the following diseases are not caused by bacteria: a. Tularemia b. Tetanus c. Plague d. Rabies e. Bacillary dysentery

d. Rabies Rabies is a viral disease that causes acute inflammation of the brain in humans and other warm-blooded animals. Early symptoms can include fever and tingling at the site of exposure. These symptoms are followed by one or more of the following symptoms: violent movements, uncontrolled excitement, fear of water, an inability to move parts of the body, confusion, and loss of consciousness

The clinical manifestations of patients with Niemann-Pick disease don`t include a. Mental retardation b. Lung involvement c. Liver involvement d. Renal involvement e. Muscle defects

d. Renal involvement

All tumors underwent the following sequence of events metaplasia-dysplasia-malignant tumor EXCEPT: a. Squamous cell carcinoma of cervix b. Squamous cell carcinoma of urinary bladder c. Squamous cell carcinoma of lung d. Squamous cell carcinoma of esophagus e. Squamous cell carcinoma of paranasal sinus

d. Squamous cell carcinoma of esophagus

Which of the following statements about poliomyelitis are true: a. The infection belongs to anthropozoonosis b. The virus enters the organism through the upper respiratory tract c. The virus enters the CNS via the peripheral nerve fibers d. The region most affected are the frontal horns of the spinal cord e. Outbreak of the disease can be stopped by T cell response

d. The region most affected are the frontal horns of the spinal cord Poliomyelitis: infectious disease caused by the poliovirus. Approximately 90% to 95% of infections cause no symptoms. Another 5 to 10% of people have minor symptoms such as: fever, headache, vomiting, diarrhea, neck stiffness and pains in the arms and legs. These people are usually back to normal within one or two weeks. In about 0.5% of cases there is muscle weakness resulting in an inability to move. Poliovirus is usually spread from person to person through infected feces entering the mouth. It may also be spread by food or water containing human feces and less commonly from infected saliva

Which of the following is the LEAST important factor in the etiology of cancer? a. Age b. Solar radiation c. Virus infection d. Traumatic injury e. Inheritance

d. Traumatic injury

Which of the following conditions in cancerogenesis/neoplastic progression is reversible? a. Invasion b. Clonal expansion c. Distant metastasis d. Tumor promotion e. Initiation

d. Tumor promotion A process in carcinogenesis by which various factors permit the descendents of a single initiated cell to survive and expand in number, i.e. to resist apoptosis and to undergo clonal growth. This is a step toward tumor progression

In which of the following locations does metastatic calcification not occur a. Renal tubules b. Glands of the fundus of the stomach c. Lung alveoli d. Tunica media in Monckeberg medial calcification e. Cornea

d. Tunica media in Monckeberg medial calcification

Which of the following vitamin deficiency manifests as xerophthalmia: a. Vitamin B1 (Thiamine) deficiency b. Vitamin B12 deficiency c. Vitamin D deficiency d. Vitamin A deficiency e. Vitamin E deficiency

d. Vitamin A deficiency Xerophthalmia (dry eyes) is a medical condition in which the eye fails to produce tears. It may be caused by a deficiency in vitamin A and is sometimes used to describe that lack, although there may be other causes

Which of the following statements about ionizing radiation are correct a. Cells in the G1 phase of the cell cycle are most sensitive b. Irradiated cells don't precede to mitosis/There is no mitosis in irradiated cells c. Cells exposed to hypoxia are particularly sensitive to radiation injury d. With radiation injuries there is delayed production of granulation tissue e. Undifferentiated neoplasms are more sensitive to irradiation than differentiated neoplasms

d. With radiation injuries there is delayed production of granulation tissue e. Undifferentiated neoplasms are more sensitive to irradiation than differentiated neoplasms

Which of the following microorganisms cause clinical symptoms resembling acute appendicitis a. Enteropathogenic E. coli b. Enterobius vermicularis c. Trichomonas hominis d. Yersinia enterocolitis e. Bacillus anthracis

d. Yersinia enterocolitis

Which of the following infectious agents most frequently cause asymptomatic helminthic infections: a. Aspergillus fumigatus b. Trichinella spiralis c. Cryptosporidium d. Candida albicans e. Ascaris lumbricoides

e. Ascaris lumbricoides - Ascaris lumbricoides: giant roundworm of humans, growing to a length of up to 35 cm - Helminthic infections: parasitic worms - They are often referred to as intestinal worms even though not all helminths reside in the intestines; for example Schistosomes (causing the disease schistosomiasis) are not intestinal worms, but rather a helminth which resides in blood vessels. Helminths belong to the group of intestinal parasites (the other type of intestinal parasite are the protozoa)

Which of the following pathogens cause infection by transmitting spores to the skin and lungs a. Neisseria gonorrhea b. Vibrio cholera c. Campylobacter jejuni d. Yersinia entercolitica e. Bacillus anthracis

e. Bacillus anthracis

Clostridium can cause all of the following manifestations except: a. Necrotizing enteritis b. Necrosis of skeletal muscle c. Generalized muscle spasms d. Cranial nerve palsies e. Cerebral abscesses

e. Cerebral abscesses

Which of the following infectious agents cause pseudomembranous enterocolitis: a. Streptococcus pyogenes b. Clostridium perfringens c. Clostridium botulinum d. Clostridium tetani e. Clostridium difficile

e. Clostridium difficile

A 3-year old boy is presented with a left sided neck swelling. Histological examination shows a cystic lesion lined with flattened endothelial cells. There is no blood present. What is the most likely diagnosis? a. Kaposi's sarcoma b. Metastatic prostate cancer c. Hemangioma d. Angiosarcoma e. Cystic hygroma

e. Cystic hygroma Also known as cystic lymphangioma and macrocystic lymphatic malformation, is often a congenital multiloculated lymphatic lesion

Perisplenitis cartilaginea is an example of a. Fibrinoid transformation of connective tissue b. Transparent transformation of connective tissue c. Myxoid transformation of connective tissue d. Hyaline transformation of connective tissue e. Fibrous transformation of connective tissue

e. Fibrous transformation of connective tissue

Deficiency of which vitamin can cause megaloblastic anemia during administration of antineoplastic drugs or antiepileptic drugs: a. B1/Thiamine b. B12 c. B2/Riboflavin d. Pyridoxine e. Folic acid

e. Folic acid An anemia (of macrocytic classification) that results from inhibition of DNA synthesis during red blood cell production. When DNA synthesis is impaired, the cell cycle cannot progress from the G2 growth stage to the mitosis (M) stage. This leads to continuing cell growth without division, which presents as macrocytosis. The defect in red cell DNA synthesis is most often due to hypovitaminosis, specifically a deficiency of vitamin B12 and/or folic acid.

Anemia in a pregnant female is most often caused by deficiency of which of the following vitamins: a. Vitamin B1 (Thiamine) deficiency b. Vitamin B12 deficiency c. Vitamin B2 (Riboflavin) deficiency d. Niacin deficiency e. Folic acid deficiency

e. Folic acid deficiency

Which of the following diseases are not usually transmitted by bites of arthropods or other insect vectors a. Cutaneous leishmaniasis b. Infection of Plasmodium falciparum c. African trypanosomiasis d. American trypanosomiasis e. Giardiasis

e. Giardiasis

Which of the following findings don't belong to the clinical picture of congenital syphilis: a. Pneumonia alba b. ''Flint stone'' liver c. Osteochondritis d. Hutschinson triad e. Gumma

e. Gumma A gumma is a soft, non-cancerous growth resulting from the tertiary stage of syphilis. It is a form of granuloma. Gummas are most commonly found in the liver (gumma hepatis), but can also be found in brain, heart, skin, bone, testis, and other tissues, leading to a variety of potential problems including neurological disorders or heart valve disease.

Which of the following microorganisms dont belong among the protozoa: a. Giardia lamblia b. Isospora belli c. Trichomonas vaginalis d. Plasmodium malariae e. Histoplasma capsulatum

e. Histoplasma capsulatum - dimorphic fungi!

Among paraneoplastic syndromes include all the following states, EXCEPT: a. Dermatomyositis b. Syndrome of inappropriate antidiuretic hormone (SIADH) c. Hypercalcemia d. Peripheral neuropathy e. Hormonal effect of carcinoembryonic antigen (CEA)

e. Hormonal effect of carcinoembryonic antigen (CEA)

Which of the following statements are correct a. Caseous necrosis is a typical cause of tissue ischemia b. Amyloid exhibits light green birefringence after staining with Berlin blue c. Post-mortem autolysis is characteristic for the vital reaction (inflammation) to necrotic cells d. Russel bodies are produced predominantly by ubiquitination e. Hyaline in the dye blue trichrome stains blue, like collagen in connective tissue

e. Hyaline in the dye blue trichrome stains blue, like collagen in connective tissue

In which way does viruses cause cell death by an indirect cytopathic mechanism: a. Injury to DNA b. Injury to RNA c. Injury to the cell membrane d. Influx of potassium to the cell e. Immune response

e. Immune response

Which of the statements about dry gangrene are correct a. It always develops secondary to venous thrombosis/On the base of venous thrombosis b. It affects predominantly the upper extremities c. Its caused by saprophytic bacteria d. It spreads rapidly and uncontrollably to healthy tissue e. It most often originates secondary to trauma Vernerova: Combination of slow vascular occlusion + trauma

e. It most often originates secondary to trauma Vernerova: Combination of slow vascular occlusion + trauma

Which of the following organisms cause chronic rhinitis a. Rotavirus b. Enteropathogenic E. coli c. Shigella dysentery d. Vibrio cholera e. Klebsiella rhinoscleromatis

e. Klebsiella rhinoscleromatis

Which of the following manifestations are not seen in patients with Lymphogranuloma venereum a. Pustular or vescicular lesions on the penis or vagina b. Lymphadenopathy c. Confluent abscesses surrounded by a zone of palisading arrangement of epithelial cells d. Proctitis in homosexuals e. Urethritis

e. Urethritis

Which of the following pairs of cell - disease are not correct a. Mikulicz cells - Rhinoscleroma b. Warthin-Finkeldey cells - Measles c. Virchow cells - Leprosy d. Orthov cells - TBC e. Langerhans cells - TBC

e. Langerhans cells - TBC - HIV, LCH (Langerhans cell histiocytosis) - Langerhans cell histiocytosis (LCH) is a rare disease involving clonal proliferation of Langerhans cells, abnormal cells deriving from bone marrow and capable of migrating from skin to lymph nodes. Clinically, its manifestations range from isolated bone lesions to multisystem disease.

What are the most likely late effects one can expect to find in survivers of atomic bombs: a. Failure of hematopoiesis after about two weeks b. Death due to GIT syndrome within the third day c. Fibrosing pneumonitis d. Brain edema with neural necrosis and death within the first hour e. Leukemia

e. Leukemia

In which of the following venereal diseases are genital ulcers not one of the manifestations: a. Syphilis b. Granuloma inguinale c. HSV-2 infection d. Chancroid e. Lymphogranuloma venereum

e. Lymphogranuloma venereum

Which of the following metals affect the kidneys predominantly in its inorgic form, while the organic form mainly injure the brain: a. Copper b. Iron c. Lead d. Nickel e. Mercury

e. Mercury - Organic compounds of mercury are often extremely toxic and have been implicated in causing brain and liver damage - Mercury occurs inorganically as salts such as mercury(II) chloride. Mercury salts affect primarily the gastrointestinal tract and the kidneys, and can cause severe kidney damage.

Which of the following infectious agents present with acute rhinitis progressing to symptoms of meningoencephalitis: a. Cryptosporidium b. Histoplasma capsulatum c. Trichinella spiralis d. Schistosoma hematobium e. Naegleria fowleri

e. Naegleria fowleri

In which of the following locations can one find the latent form of the virus in people infected with HSV type 2: a. Mucosa of the external genitalia b. Epithelium of internal genital organs c. Germ cells d. Connective tissue of the penis or vagina e. Neural ganglia of the sacral region

e. Neural ganglia of the sacral region

Which of the following are not ocular complications of Chlamydia trachomatis: a. Conjunctival congestion/hyperemia b. Pannus formation (an abnormal layer of fibrovascular tissue or granulation tissue) c. Lymphocytic infiltrate d. Scarring e. Retinal detachment

e. Retinal detachment Serotypes/serovars Ab, B, Ba, or C — cause trachoma --> infection of the eyes, which can lead to blindness

What is the most common complication of Gonorrhea in young females: a. Myocarditis b. Encephalitis c. Meningitis d. Pancreatitis e. Salpingitis

e. Salpingitis An infection and inflammation in the fallopian tubes.

All the following statements are valid about oncogene products, except: a. May be a nuclear protein b. Some act as growth factors c. Some act as receptors for growth factors d. Some has tyrosine kinase activity e. Some facilitates the process of apoptosis

e. Some facilitates the process of apoptosis

Which of the following are not examples of 'hyaline droplets' deposits a. Mallory bodies b. Russel bodies c. Crooke hyaline degeneration/change of the basophilic hypophysis d. Deposits of alpha-1-antitrypsin in the liver e. Splendore-Hoeppli bodies/ phenomenon

e. Splendore-Hoeppli bodies/ phenomenon

The liver is not injured during a. Glycogenosis type IV/Andersen disease b. Galactosemia c. Niemann-Pick disease d. Wilson`s disease e. Tay-Sachs disease

e. Tay-Sachs disease

Which of the following infectious agents can cause superficial skin mycosis also affecting hairs: a. Aspergillus fumigatus b. Histoplasma capsulatum c. Clonorchis sinensis d. Candida albicans e. Trichophyton

e. Trichophyton


Ensembles d'études connexes

Completing the Application, Underwriting, and Delivering the Policy

View Set

Visual Element - Perspective Online Content

View Set

Trivia (1-15) - BAC: Texas French Symposium

View Set

Ch.32 The Great Depression and The New Deal

View Set

BIBLE 25.5 Elijah Boldly Confronts Evil

View Set

Chapter 21: postpartum complications

View Set

Life Insurance Underwriting and Policy Issue

View Set